You are on page 1of 117

Master (comprehensive) budget

Fact pattern: Farfora Corporation manufactures and sells two products, Thing one and Thing two,
Farfora’s budget department gathered the following data to project sales and budget requirements:

Projected sales

Product Units Price


Thing one 60,000 $70
Thing two 40,000 $100

Projected inventories – in units


Product Expected January ,1 Desired December ,31
Thing one 20,000 25,000
Thing two 8,000 9,000

To produce one unit of thing one and thing two, the following raw materials are used:
Raw material Thing one Thing two
(A) 1b 4 5
(B) 1b 2 3
(C) Inch - 1

Projected data for the year with respect to raw materials are as follows:
Raw material Anticipated Expected beg Desired End
Purchase price Inventories Inventories
A $8 32,000 1b 36,000 1b.
B $5 29,000 1b 32,000 1b.
C $3 6,000 Inch 7,000 Inch

Projected direct labor requirements and rates are as follows:


Thing one  Two hours per unit at $3 per hour
Thing two  Three hours per unit at $4 per hour
Overhead is applied at the rate of $2 per direct labor hour.
What are projected gross profit and the estimated value of ending inventories?
Item of D.M D.M usage Q budget Ending Beg Raw Q of required raw
Raw materials materials material purchases
A (65 × 4)) + (41 × 5) = 465,000 36,000 (32,000) 469,000
B (65 × 2) + (41 × 3) = 253,000 32,000 (29,000) 256,000
C (65 × 0) + (41 × 1) = 41,000 7,000 (6,000) 42,000

Item of raw Q of required raw M purchases × price per unit Cost of raw M. purchases
materials
A 469,000 units × $8 3,752,000
B 256,000 units × $5 1,280,000
C 42,000 units × $3 126,000
Total $ 5,158,000

Item Production Quantity DLH Total DLH


Thing one 65,000 units 2 130,000
Ting two 41,000 units 3 123,000
Total DLH required to satisfy production Q budget 253,000

Production item Total DLH required DLH rate DL budget in $


Thing one 130,000 $3 $390,000
Thing two 123,000 $4 $492,000
Total $882,000

Production item Factory O.H. per unit


Thing one 2 DLH × $ 2 = $4
Thing two 3 DLH × $ 2 = $6

Cost per unit of finished goods


Description Thing one Thing two

Direct material
A 4 units × $8 = 32 5 units × $8 = 40
B 2 units × $ 5 = 10 3 units × $5 = 15
C zero units × $3 = 0 1 unit × $3 = 3
D.M cost per unit $ 42 $ 58
D.L. cost per unit 2 DLH × 3 = $6 3 DLH × $4 = $12

O.H. cost per unit 2 DLH × $2 = 4 3 DLH × 2 = $6


Total manufacturing cost per unit $52 $76

Projected gross profit


Description Thing one Thing two Total
Sales 60,000 × 70 = 4,200,000 40,000 × 100 = 4,000,000 8,200,000
Cost of goods sold 60,000 ×52 = 3,120,000 40,000 × 76 = 3,040,000 6,160,000
Gross profit $1,080,000 $ 960,000 2,040,000

Ending inventories costs


Item Quantity Costs per unit Cost of ending inv.
Thing one 25,000 units of FG $52 $1,300,000
Thing two 9,000 units of FG $76 $684,000
A 36,000 units of R.M $8 $288,000
B 32,000 units of R.M $5 $160,000
C 7,000 units of R.M $3 $21,000
Total $2,453,000

Master budget is a compilation of all of the separate operating and financial budget schedules of
the organization.

Operating budgets Financial budgets


(1) Sales budgets (Q and $) is the first budget to be developed. (9) Capital expenditure budget.
(2) Production Q budget is based on forecasted sales volume (expenditures for productive capacity
and desired ending finished goods inventory and E.U of WIP are a function of long-term estimates
Production Q = forecasted sales Q Minus budgeted Beg. F.G of demand for the firm’s products)
and EU in BWIP Plus ending F.G and EU in EWIP (10) Working capital budgets
(3) D.M. to be used budget (Q and $) (11) Cash budget (last budget to be
(4) D.M. to be purchased budget (Q and $) prepared)
Q of D.M. to be purchased = Q of D.M. to be used Minus
budgeted Beg. D.M inventory Plus ending D.M inventory
(5) D.L. budgets
(6) Estimated factory O.H. budget.
(7) Selling and administrative expenses budgets

Operating budgets: Consist of all budgets that concern


normal operating activities

Operating Pro-forma Financial Pro-forma


(8) Pro-forma income statement (12) Pro-forma balance sheet
(13) Pro forma statement of changes
in owner's equity.
(14) Pro-forma statement of cash
flow (last pro-forma to be prepared)

Master budget (comprehensive budget)

1. Strategic planning drives all other planning and budgeting activities within the firm thus, strategic
planning must completed first before master budget preparation.

2. Master budget may be prepared by a for-profit entity or non for profit organizations, public company
and governmental unit. Also service company, manufacturing company, wholesale or retail Company
3. The master budget provides a comprehensive summation of all of an entity’s budgets and its plans
for the operating activities of its subunits. It is the place where everything must add up, where strategy
and long-term plans meet up with short-term objectives and current realities. The master budget is
basically made up of financial projections of many different budgets for a company on an annual basis,
although other short-duration time periods are also used.

4. Master budget is a formal quantitative model recognizing the organization’s objectives, inputs, and
outputs and contains both operating budgets and financial budgets. Thus master budget is a compilation
of all the separate operational and financial budget schedules of the organization for a specified period of
time and for the planned level of activity (master budget is a static budget).

5. A master or comprehensive budget consolidates all budgets into an overall planning and control
document for the organization.

6. The master budget reflects all applicable expected costs and revenues, whether or not controllable by
individual managers. Because master budget coordinate the various activities of a firm

7. Sales forecast is the first step in sales budget and the sales budget is the foundation of the master
budget.

8. The usual starting point in master budgeting process is to forecast sales demand (Q) and revenues ($)
for most companies, the level of sales is the most important constraint in business management .Once a
firm can estimate sales preferably based on expected value, the next step is to decide how much to
produce or purchase. Thus production Q budget is based on assumptions appearing in the sales budget,
once the production Q budget has been completed, the next step is to prepare the raw material, direct
labor, and manufacturing O.H budgets.

9. Sales budget defines the capacity needed throughout the company.

10. The level of manufacturing (production) capacity is fixed in the short run. In the long run, it may be
adjusted in accordance with long term projected sales.

11. Capital budget is independent component of the master budget that concerns satisfying the long run
product demand.

12. The master budget can be broken down into two major components.

A. The operating budget makes up the bulk of the master budget and it includes the sales budgets, the
production Q budget, the D.M usage and D.M purchases budgets, the D.L budgets, the O.H budgets, and the
selling and administrative expense budgets. All of these budgets culminate in the formation of the pro-
forma (or budgeted) income statement.

B. The financial budget includes the capital expenditure budget, R& D budget working capital budget, cash
budget, and the pro- forma (or budgeted) statement of changes in owner's equity, pro- forma balance sheet
and pro forma statement of cash flows.

13. Master budget can be used to determine variances if it segregates costs to fixed and variable
components and thus flexible budget can be adapted.

14. The most important factors in budgeting manufacturing O.H. are:


A. Production volume: because V.C of manufacturing O.H. depends on production volume.

B. Management judgment: because O.H. is applied based on predetermined rates thus the nature and
amount of O.H. costs expected to be incurred is subject to judgment. Hence accurate judgment is
important.

True or false

The usual starting point in budgeting is to forecast net income.


Answer is false. The usual starting point in budgeting is to forecast sales demand and revenues.

Q1: The master budget


A. shows forecasted and actual results.
B. Reflects controllable costs only
C. Can be used to determine manufacturing cost variances
D. contains the operating budget.
Answer (A) is incorrect because the master budget does not contain actual results.
Answer (B) is incorrect because the master budget reflects all applicable expected costs, whether or not
controllable by individual managers.
Answer (C) is incorrect because the master budget is not structured to allow determination of
manufacturing cost variances, which requires using the flexible budget and actual results.
Answer (D) is correct. All other budgets are subsets of the master budget. Thus, quantified estimates by
management from all functional areas are contained in the master budget. These results are then
combined in a formal quantitative model recognizing the organization’s objectives, inputs, and outputs.

Q2: In an organization that plans by using comprehensive budgeting, the master budget is:
A. a compilation of all the separate operational and financial budget schedules of the organization.
B. The booklet containing budget guidelines, policies, and forms to use in the budgeting process.
C. The current budget updated for operations for part of the current year.
D. A budget of a not-for-profit organization after it is approved by the appropriate authoritative body.
Answer (A) is correct. Budgets coordinate the various activities of a firm. A company’s overall budget,
often called the master or comprehensive budget encompasses the organization’s operating and financial
plans for a specified period, ordinarily a year. Thus, all other budgets are subsets of the master budget.
The operating budget is the part of the master budget that consists of the pro forma income statement and
related budgets. Its emphasis is on obtaining and using resources. The financial budget is the part of
the master budget that includes the cash budget, capital budget, pro forma balance sheet, and pro forma
statement of cash flows. Its emphasis is on obtaining the funds needed to purchase operating assets.
Answer (B) is incorrect because the booklet containing budget guidelines, policies, and forms to use in the
budgeting process is the budget manual.
Answer (C) is incorrect because the current budget updated for operations for part of the current year is a
continuous budget.
Answer (D) is incorrect because a master budget may be prepared by a for-profit entity.

Q3: In the budgeting and planning process for a firm, which one of the following should be completed
first?
A. Sales budget. B. Financial budget. C. Cost management plan. D. Strategic plan.
Answer (D) is correct: Strategic planning drives all other planning and budgeting activities within the
firm.
Q4: After the goals of the company have been established and communicated, the next step in the
planning process would be the development of the
A. Production budget B. Sales forecast
C. Selling and administrative budget D. Direct materials budget
The correct answer is (B) : After the goals of the company have been established and communicated,
the budgeting process begins The first step in the budgeting process is the completion of a sales
forecast.

Q5: Which of the following is the element that if inaccurate will throw off all of the other master
budget elements?
A. Sales forecast B. Production forecast C. Cash budget D. Capital budget
The correct answer is (A). Without an accurate sales forecast, all other budget elements will be
inaccurate because production levels, material purchases, etc., are set to match expected sales.

Q6: The master budget process usually begins with the


A. Production budget. B. Operating budget. C. Financial budget. D. Sales budget.
Answer (A) is incorrect because the production budget normally can't be prepared until the expected
sales are determined.
Answer (B) is incorrect because the operating budget is another term for the budget used on a day-to-
day basis for managing operations. It can't be prepared until after the sales budget is prepared.
Answer (C) is incorrect because preparation of the sales budget is the first step in the overall budgeting
process.
Answer (D) is correct. A master or comprehensive budget consolidates all budgets into an overall
planning and control document for the organization. The preparation of a master budget usually
takes several months. The sales budget is the first budget prepared because it is the basis for all
subsequent budgets. Once a firm can estimate sales, the next step is to decide how much to produce or
purchase.

Q7: Which of the following budgets is the first operating budget prepared because it defines needed
capacity for operations?
A. Direct labor budget B. Production/inventory budget
C. Overhead budget D. Sales budget
Answer (D) is Correct. The sales forecast and the resulting sales budget are prepared before other
budgets and define the capacity needed throughout the company.

Q8: Which one of the following items should be done first when developing a comprehensive budget for a
manufacturing company?
A. Determination of the advertising budget. B. Development of a sales budget.
C. Development of the capital budget. D. Preparation of a pro forma income statement.
Answer (A) is incorrect because the amount of advertising cost depends on the desired level of sales.
Answer (B) is correct. The sales budget is usually the first to be prepared because all other elements of a
comprehensive budget depend on projected sales. For example, the production budget is based on an
estimate of unit sales and desired inventory levels. Thus, sales volume affects purchasing levels, operating
expenses, and cash flow.
Answer (C) is incorrect because expenditures for productive capacity are a function of long-term
estimates of demand for the firm’s products.
Answer (D) is incorrect because preparation of a pro forma income statement is one of the final steps in the
budgetary process. It can't be prepared until after all sales, production, and expense budgets are finished.
Q9: When preparing the series of annual operating budgets, management usually starts the process with
the
A. Cash budget. B. Balance sheet. C. Capital budget. D. Sales budget.
Answer (A) and (B) are incorrect because Cash budget and Balance sheet can't be prepared until the
sales budget has been determined.
Answer (C) is incorrect because Capital budget is independent of the coming year forecasted sales.
Answer (D) is correct. A company usually begins with the sales budget and then proceeds to the
production budget. Once the production budget is complete, then the raw materials, direct labor, and
overhead budgets can be prepared. Next, the capital budget can be prepared, followed by a cash budget.
The final step is the preparation of the pro forma financial statements.

Q10: In developing a comprehensive budget for a manufacturing company, which one of the following
items should be done first?
A. Development of a sales plan. B. Determination of manufacturing capacity.
C. Development of the capital budget. D. Determination of the advertising budget.
Answer (A) is correct. Normally, the preparation of a sales budget is the first step in the comprehensive
budget process. For most companies, the level of sales is the most important constraint in business
management. Once sales have been determined, related budgets can be prepared, e.g., the production
budget, capital budget, expense budgets, and pro forma financial statements.
Answer (B) is incorrect because the level of manufacturing (production) capacity is fixed in the short
run. In the long run, it may be adjusted in accordance with projected sales.
Answer (C) is incorrect because a capital budget is independent component of the master budget that
concerns satisfying the long run product demand.
Answer (D) is incorrect because expense budgets are not prepared until the level of operating activity is
known.

Q11: The necessary input for production budget process is


A. Direct labor budget. B. Direct materials budget.
C. Manufacturing overhead budget. D. Sales budget.
Answer (A) is incorrect because the direct labor budget can't be prepared until after unit production
figures have been compiled.
Answer (B) is incorrect because the direct materials budget can't be prepared until after the unit
production figures have been compiled.
Answer (C) is incorrect because the manufacturing O.H budget can't be prepared until after the unit
production figures have been compiled.
Answer (D) is correct. A production budget and its components can't be prepared until after the sales
budget has been completed. Once a firm determines its expected sales, production quantity can be
estimated. The production budget is based on assumptions appearing in the sales budget; thus, the sales
budget is the first step in the preparation of a production budget.

Q12: Individual budget schedules are prepared to develop an annual comprehensive or master budget.
The budget schedule that would provide the necessary input data for the direct labor budget would be
the
A. Sales forecast. B. Raw materials purchases budget.
C. Schedule of cash receipts and disbursements. D. Production budget.
Answer (A) is incorrect because the sales forecast is insufficient for completion of the direct labor
budget.
Answer (B) is incorrect because the raw material purchases budget is not needed to prepare a direct
labor budget.
Answer (C) is incorrect because the schedule of cash receipts and disbursements can't be prepared until
after the direct labor budget has been completed.
Answer (D) is correct. A master budget typically begins with the preparation of a sales budget. The
next step is to prepare a production quantity budget. Once the production budget has been completed,
the next step is to prepare the direct labor, raw material, and manufacturing O.H budgets. Thus, the
production budget provides the data for the completion of the direct labor budget.

Q13: There are various budgets within the master budget cycle. One of these budgets is the production
budget. Which one of the following best describes the production budget?
A. It summarizes all discretionary costs.
B. It includes required direct labor hours.
C. It includes required material purchases.
D. It is calculated from the desired ending inventory and the sales forecast.
Answer (A) is incorrect because a production budget is usually prepared in terms of units of output rather
than costs.
Answer (B) is incorrect because the D.L budget is prepared after the production budget.
Answer (C) is incorrect because the materials purchases budget is prepared after the production budget.
Answer (D) is correct. A production budget is based on sales forecasts, in units, with adjustments for
beginning and ending inventories. It is used to plan when items will be produced. After the production
budget has been completed, it is used to prepare materials purchases, D.L, and factory O.H budgets

Q14: Which of the following (internal) pro forms financial statements is usually the last budget
prepared at the end of a period?
A. Pro forma balance sheet B. Pro forma income statement
C. Pro forma cash budget D. Pro forma statement of stockholder’s equity

The correct answer is (A): The pro forma or budgeted balance sheet is based in part on the current
period’s budget balance sheet and in part on the other budgets so it is usually the last item prepared in a
master budget among the stated items.

Q15: Which of the following correctly orders the breakdown of budgets from most general to most
specific?
A. Operating, sales, O.H, production B. Production, operating, O.H, sales
C. Operating, sales, production, O.H D. Operating, Production, sales, O.H
The correct answer is (C) Operating budgets include sales budgets. A sales budget is followed by
production budgets and selling and administrative expense budgets. Production budgets are broken
down into D.M, D.L, O.H, and COGS budgets

Q16: The cash budget must be prepared before you can complete the
A. Production budget. B. Forecasted balance sheet
C. Forecasted income statement. D. Capital expenditure budget

the correct answer is: (B): The cash budget determines the projected borrowings. Repayments, investments,
interest received and interest paid. All of this information is needed to forecast the balance sheet.

Q17: Which one of the following items is the last schedule to be prepared in the normal budget
preparation process?
A. Cash budget B. Cost of goods sold budget
C. Selling expense budget D. Manufacturing overhead budget
The correct answer is (A): The cash budget is the last schedule to be prepared in the normal budget
preparation process. All of the other budgets listed are needed in order to determine the receipts and
disbursements required to prepare the cash budget

Q18: The preparation of a comprehensive master budget culminates with the preparation of the
A. Production budget. B. Capital investment budget.
C. Cash management and working capital budget. D. Strategic budget.
Answer (A) is incorrect because the production budget must precede the capital investment and cash
budgets.
Answer (B) is incorrect because a capital investment budget is prepared before a cash budget.
Answer (C) is correct. The comprehensive master budget begins with the preparation of the sales
budget and proceeds to the production budget. The production budget is followed by D.M usage and
purchases, labor, and O.H budgets and departmental expense budgets. A capital investment budget
will follow next. Finally, a cash budget and working capital budget will culminate the process.
Answer (D) is incorrect because a strategic budget is a long-range planning tool that is prepared
before the master budget.

Q19: Pro forma F/S are part of the budgeting process. Normally, the last pro forma statement prepared
is the
A. Capital expenditure plan. B. Income statement.
C. Statement of COGS. D. Statement of cash flows.
Answer (A) is incorrect because the capital expenditure plan must be prepared before the cash budget.
Cash may be needed to pay for capital purchases.
Answer (B) is incorrect because the income statement must be prepared before the statement of cash
flows, which reconciles net income and net operating cash flows.
Answer (C) is incorrect because COGS is included in the income statement, which is an input to the
statement of cash flows.
Answer (D) is correct. The statement of cash flows is usually the last of the listed items prepared. All
other elements of the budget process must be completed before it can be developed. The statement of
cash flows reconciles net income with net operating cash flow, a process that requires balance sheet data
(e.g., changes in receivables, payables, and inventories) as well as net income.

Q20: Which one of the following statements regarding selling and administrative budgets is most
accurate?
A. Selling and administrative budgets are usually optional.
B. Selling and administrative budgets are fixed in nature.
C. Selling and administrative budgets are difficult to allocate by month and are best presented as one
number for the entire year.
D. Selling and administrative budgets need to be detailed in order that the key assumptions can be better
understood.
E. Selling and administrative budgets should be a certain percentage of sales and should be developed
using a bottom-up approach.
Answer (A) is incorrect because selling and administrative budgets are no more optional than any other
component of the master budget.
Answer (B) is incorrect because selling and administrative budgets have both variable and fixed
components.
Answer (C) is incorrect because selling and administrative budgets should be prepared on the same basis
as the remainder of the budget, typically on at least a monthly basis.
Answer (D) is correct. Sales and administrative budgets are prepared after the sales budget. They are
among the components of the operating budget process, which culminates in a budgeted (pro forma)
income statement. Like the other budgets, they constitute prospective information based on the preparer’s
assumptions about conditions expected to exist and actions expected to be taken. Selling and
administrative budgets need to be detailed in order to determine the relevant sales and administrative
expense items. The relevant detail provides for more enhanced communication of goals and objectives,
coordination of actions, and control of activities.

Q21: All of the following are considered operating budgets except the
A. Sales budget. B. Materials budget. C. Production budget. D. Capital budget.
Answer (D) is correct. The operating budget consists of all budgets that concern normal operating
activities, including the sales budget, production budget, materials budgets, D.L budget, and factory O.H
budget. The capital expenditures budget, which outlines needs for new capital investment, is not a part of
normal operations. The capital expenditures budget is sometimes prepared more than a year in advance to
allow sufficient time to secure financing for these major expenditures. The long lead time is also necessary
to allow sufficient time for custom orders of specialized equipment and buildings.

Q22: Which of the following is normally included in the operating budget?


A. Capital budget. B. Cash budget.
C. Selling expense budget. D. Budgeted balance sheet.
Answer (A) is incorrect because the capital budget is included in the financial budget.
Answer (B) is incorrect because the cash budget is included in the financial budget.
Answer (C) is correct. An operating budget normally includes sales, production, selling and
administrative, and budgeted income statement components.
Answer (D) is incorrect because the budgeted balance sheet is included in the financial budget.

Q23: Which of the following is normally included in the financial budget of a firm?
A. Direct materials budget. B. Selling expense budget.
C. Budgeted balance sheet. D. Sales budget.
Answer (A) is incorrect because the direct materials budget is included in the production budget.
Answer (B) is incorrect because the selling expense budget is included in the operating budget.
Answer (C) is correct. The financial budget normally includes the capital budget, the cash budget, the
budgeted balance sheet, and the budgeted statement of cash flows.
Answer (D) is incorrect because the sales budget is included in the operating budget.

Q24: The financial budget process includes


A. The cash budget. B. The capital budget.
C. The budgeted statement of cash flows. D. All of the answers are correct.

Answer (D) is correct. The financial budget process includes all mentioned elements

Q25: Many companies use comprehensive budgeting in planning for the next year’s activities.
When both an operating budget and a financial budget are prepared, which one of the following
is correct concerning the financial budget?

Choice Included in the financial budget


Capital budget Pro-forma balance sheet Cash budget
A Yes No Yes
B No Yes No
C Yes Yes Yes
D No No No

Answer (C) is correct


Q26: A company that manufactures furniture is establishing its budget for the upcoming year. All of
the following items would appear in its O.H budget except for the
A. Overtime paid to the workers who perform production scheduling.
B. Cost of glue used to secure the attachment of the legs to the tables.
C. Fringe benefits paid to the production supervisor.
D. Freight charges paid for the delivery of raw materials to the company.
Answer (D) is correct Freight charges paid for the delivery of raw materials to the company is
considered as part of D.M cost.

Q27: Which of the following is the correct order of preparing operating budgets?
A. Sales budget, production budget, product cost budgets (including D.M, D.L, and factory O.H budgets),
and COGS budget
B. Sales budget, COGS budget, production budget, and product costs budgets (including D.M, D.L, and
factory COGS budgets)
C. Production budget, product cost budget (including D.M, D.L, and factory COGS budgets), sales
budget, and COGS budget
D. Production budget, product cost budget (including D.M, D.L, and factory COGS budgets), COGS
budget, and sales budget
Answer (A) is correct The sales forecast leads to the production budget and from there the product costs
budgets are prepared. After product cost budgets are complete, the firm creates a COGS budget.

Q28: Wilson Company uses a comprehensive planning and budgeting system. The proper order
for Wilson to prepare certain budget schedules would be
A. COGS, balance sheet, income statement, and statement of cash flows.
B. Income statement, balance sheet, statement of cash flows, and COGS.
C. Statement of cash flows; cost of goods sold, income statement, and balance sheet.
D. COGS, income statement, balance sheet, and statement of cash flows.
Answer (A) is incorrect because the balance sheet should not precede the income statement.
Answer (B) is incorrect because the income statement cannot precede cost of goods sold.
Answer (C) is incorrect because the statement of cash flows cannot precede the cost of goods
sold. The latter is an input of the former.
Answer (D) is correct. The pro forma COGS must be prepared before the pro forma income
statement because it is a component of the income statement. Also, the income statement must be
prepared before the pro forma balance sheet because net income is a necessary part of preparing
the stockholders’ equity section of the balance sheet. In turn, the income statement and the
balance sheet are necessary for estimating cash flows. If the statement of cash flows is prepared
using the indirect method, balance sheet data, e.g., the changes in A/R, inventory, and A/P, must
be available to determine the adjustments needed to reconcile net income to net cash flow.

Q29: Which one of the following schedules would be the last item to be prepared in the normal budget
preparation process?
A. Direct labor budget. B. Cash budget.
C. COGS budget. D. Manufacturing O.H budget.
Answer (A) is incorrect because a direct labor budget must be prepared before the cash budget.
Answer (B) is correct. The budget process begins with the sales budget, proceeds to the production and
expense budgets, and eventually the cash budget. The cash budget can’t be prepared until the end of the
process because all other budgets provide inputs to the cash budget.
Answer (C) is incorrect because a cost of goods sold budget must be prepared before the cash budget.
Answer (D) is incorrect because a manufacturing overhead budget must be prepared before the cash
budget.

Q30: Which one of the following best describes the order in which budgets should be prepared
when developing the annual master operating budget?
A. Production budget, D.M budget, revenue budget.
B. Production budget, revenue budget, D.M budget.
C. Revenue budget, production budget, D.M budget.
D. Revenue budget, D.M budget, production budget
Answer (C) is correct The correct chronological order in budget preparation always starts with
the sales budget, production budget, purchases budget (including D.M), COGS budget, and
administrative budget.

Q31: What would be the correct chronological order of preparation for the following budgets?
I. COGS budget. II. Production budget.
III. Purchases budget. IV. Administrative budget.
A. III, II, IV, I. B. IV, II, III, I. C. I, II, III, IV. D. II, III, I, IV
Answer (D) is correct The correct chronological order in budget preparation always starts with
the sales budget (not inclusive in this problem), production budget, purchases budget, COGS
budget, and administrative budget.

Q32: Which of the following refers to a quantitative expression of proposed management actions for a
set period of time?
A. Budgets B. COGS statements
C. COGM statements D. Financial statements
Answer (A) is correct A budget is a plan, expressed in financial terms, and is for a set period of time.

Q33: The starting point for creating a master budget for a proprietary secretarial school would be:
A. Preparing a capital expenditure budget. B. Preparing the student recruiting budget.
C. Forecasting enrollment. D. Estimating salaries of the instructors.
The correct answer is (C): The master budget always begins with the forecast of sales revenue,
Tuition from students is the revenue source in a proprietary school.

Q34: The process of creating a formal plan and translating goals into a quantitative format is
A. Process costing. B. Job order costing.
C. Activity-based costing D. Budgeting
The correct answer is (D): A budget is a plan expressed in dollar terms.

Q35: Which one of the following factors may influence financial forecasts in the future?
A. Last year’s production level B. Like-kind exchange of equipment today
C. Current year’s wages D. Pending litigation against the organization
The correct answer is (D): In preparing a financial forecast, it is important to include the items
that are most likely to occur so the organization can effectively anticipate any cash flow changes
in the future

Q36: Which of the following best describes how a master budget is used?
A. It compiles the operational budgets of all the separate operational and financial groups in the organization.
B. It is the projected schedule of costs and revenues against which each department is measured.
C. It is followed exactly for up to a year, but it may be adjusted after one year.
D. It deals only with those categories that will appear on future financial statements.
Answer (A) is correct:

Q37: An entity has excess capacity in production-related property, plant, and equipment. If in a
given year these assets are being used to only 80% of capacity and the sales level in that year is 2
million, the full capacity sales level is
A. 2,500,000 B. 10,000,000 C. 2,000,000 D. 1,600,000
Answer (A) is correct: If actual sales level is 2,000,000 and the plant is operating at 80%, then
full capacity sales level is 2,500,000 (2,000,000 / 80%)

Q38: A company has budgeted sales of 24,000 finished units for the forthcoming 6-month
period. It takes 4 pounds of D.M to make one finished unit. Given the following:

Description Finished units Direct materials (Pounds)


Beginning inventory 14,000 44,000
Target ending inventory 12,000 48,000

How many pounds of D.M should be budgeted for purchase during the 6-month period?
A. 48,000 B. 88,000 C. 96,000 D. 92,000
Answer (D) is correct:
1. Forecasted production of F.G = 24,000 + 12,000 – 14,000 = 22,000 Units
2. Budgeted purchases of D.M = (22,000 × 4) + 48,000 – 44,000 = 92,000

Capital budgeting

♥Project budgeting (capital budgeting) are normally associated with capturing the costs of a particular
endeavor (task) from beginning to end.

♥Project budgeting (capital budgeting) may be extended to more than one year; thus capital
budgeting is a long term planning process for long-term investments.

EX: - Construction of a building.

- Development of a product

♥The first step in planning capital expenditures is to identify capital additions projects (identifying
capital needs)

Then:

 Analyzing capital addition proposals.


 Evaluating all promising alternatives.
 Ranking capital expenditure proposals based on selected financial criteria (ROI, NPV,
IRR, payback period…) and non financial criteria (safety, less pollution. …).
 Selecting specific project or projects based on quantitative and qualitative criteria.
 Finance the project (undertakings).
 Implementation of the project
 Monitoring and feedback.

♥Capital expenditure budget: Outlines needs for new capital investment, capital expenditure
budget is prepared for more than one year in advance to allow sufficient time to secure financing
for these major expenditures. Capital budget is considered an independent item in the preparation
of the master budget, because it may be prepared more than a year in advance. Thus capital
budgeting is a long planning process for investments.

Q1: Which one of the following is the best characteristic concerning the capital budget? The capital
budget is a (n)
A. Plan to insure that there are sufficient funds available for the operating needs of the company.
B. Exercise that sets the long-range goals of the company including the consideration of external
influences caused by others in the market.
C. Plan that result in the cash requirements during the operating cycle.
D. Plan that assesses the long-term needs of the company for plant and equipment purchases.
Answer (A) is incorrect because capital budgeting involves long-term investment needs, not immediate
operating needs. The choice relates to cash budget
Answer (B) is incorrect because establishing long-term goals in the context of relevant factors in the
firm’s environment is strategic planning.
Answer (C) is incorrect because cash budgeting determines operating cash flows. Capital budgeting
evaluates the rate of return on specific investment alternatives.
Answer (D) is correct. Capital budgeting is the process of planning expenditures for long-lived assets.
It involves choosing among investment proposals using a ranking procedure. Evaluations are based on
various measures involving the rate of ROl.

Q2: Which one of the following may be considered an independent item in the preparation of the
master budget?
A. Ending inventory budget. B. Capital investment budget.
C. Pro forma income statement. D. Pro forma statement of financial position.
Answer (A) is incorrect because the ending inventory budget is based on the current production budget.
Answer (B) is correct. The capital investment budget may be prepared more than a year in advance,
unlike the other elements of the master budget. Because of the long-term commitments that must be
made for some types of capital investments, planning must be done far in advance and is based on
needs in future years as opposite the current year’s needs.
Answer (C) is incorrect because the pro forma income statement is based on the sales budget, expense
budgets, and all other elements of the current master budget.
Answer (D) is incorrect because the pro forma balance sheet is based on the other elements of the
current master budget.

Q3: All of the following are part of a capital investment budget except?

A. Factory machine purchase. B. Purchasing a business.


C. Research and development costs. D. Cost of new product development unit.
The correct answer is (C). Research and development costs are period costs that must be expensed in the
period incurred and are therefore not part of a capital budget.

Q4: Which of the following is an integral part of long-term planning?

A. Operational budgeting B. Financial budgeting


C. Activity analysis D. Capital budgeting

Answer (D) is Correct. Capital budgeting is the process of allocating resources to an entity's long-term
projects

Q5: Capital investment projects include proposals for all of the following except
A. The acquisition of government mandated pollution control equipment.
B. The expansion of existing product offerings
C. Additional research and development facilities
D. Refinancing existing working capital agreements
Answer (D) is correct

Q6: Which of the following items is not an example of a capital expenditure?


A. Heating, ventilation, and air-conditioning system upgrade for EPA compliance
B. Purchase of a new assembly machine that will cut labor and maintenance costs
C. Purchase of a new computer server for the research and development group
D. Project bonuses paid to salaried employees
Answer (D) is correct

Q7: Capital budgeting is best described as


A. Decision making related to current expenditures.
B. The process of making long-term investment decisions
C. The process of planning for short-term investments
D. Decision making to support operating efficiency
Answer (B) is correct

Q8: Capital budgeting is used for the decision analysis of


A. Adding product lines or facilities. B. Multiple profitable alternatives.
C. Lease-or-buy decisions. D. All of the answers are correct.
Answer (A) is incorrect because Capital budgeting may also be used for analysis of multiple profitable
alternatives and of lease-or-buy decisions.
Answer (B) is incorrect because Capital budgeting permits analysis of adding or discontinuing product
lines or facilities and of lease-or-buy decisions.
Answer (C) is incorrect because the lease-or-buy decision is just one specific example of an appropriate
use of capital budgeting techniques.
Answer (D) is correct. The capital budgeting process is a method of planning the efficient expenditure
of the firm’s resources on capital projects. Such planning is essential in view of the rising costs of scarce
resources.

Q9: The budget that describes the long-term position, goals, and objectives of an entity within its
environment is the
A. Capital budget. B. Operating budget.
C. Cash management budget. D. Strategic budget.
Answer (A) is incorrect because capital budgeting involves evaluating specific long-term investment
decisions.
Answer (B) is incorrect because the operating budget is a short-range management tool.
Answer (C) is incorrect because cash management is a short-range consideration related to liquidity.
Answer (D) is correct. Strategic budgeting is a form of long-range planning based on identifying and
specifying organizational goals and objectives. The strengths and weaknesses of the organization are
evaluated and risk levels are assessed. The influences of environmental factors are forecast to derive the
best strategy for reaching the organization’s objectives.

Q10: A company is in the process of identifying, evaluating, and selecting projects that require a large
commitment of funds and will generate benefits well into the future. The company will look at the
budget over the life of the projects and review many different options. This is an example of
A. Rolling budget since they want information for multiple periods
B. Flexible budget as they can prepare it using several options
C. Capital budget to help with the evaluation and identification
D. Activity-based budget to evaluate all activities for each project
Answer (C) is correct:
Operational planning

Examples that are covered in tactical planning:


-Annual budgets
-Target product mix for the next year
- Outsourcing decisions.
-Production schedules
- Material procurement

1. Sales forecast and sales budget

A sales forecast is a subjective estimate of the entity’s future sales for the upcoming period. Without an
accurate sales forecast, all other budget elements will be inaccurate. Forecasters consider not only
historical trends for sales, but also economic and industry conditions and indicators, competitors’ actions,
rising costs, policies on pricing and extending credit, the amount of advertising and marketing
expenditures, the number of unfilled back orders, as well as sales in the sales pipeline (unsigned prospects
i.e customer purchase orders that are not approved yet by credit manager) and also production capacity.

Sales forecasts should use statistical analysis techniques such as regression analysis and time series
analysis and rely on sales managers’ knowledge about their market and customer needs. Once a company
has determined its forecasted sales level, based on its long- and short-term objectives, it forms a sales
budget to accomplish its goals. The two key components of the sales budget are the projected number of
units of sales and the projected selling prices for the upcoming periods. The sales budget basically drives
the operating budget because it helps define the formation of the other budgets.

Q1: Which of the following is not an important step in forming a sales budget?
A. Studying economic forecasts
B. Creating a projected income statement
C. Analyzing long- and short-term company objectives
D. Analyzing production capacity
The correct answer is (B) The D.M, D.L, and O.H budgets required to create a projected or pro
forma income statement are made after the sales budget is formed
Q2: Which of the following is not a required step prior to forming a sales budget?
A. Forecasted sales B. Production capacity
C. Labor, materials, and O.H budget D. Long- and short-term objectives
The correct answer is (C). The D.M, D.L, and O.H budgets are created after the sales budget is
formed.

Q3: The budget that is usually the most difficult to forecast is the
A. Production budget. B. Expense budget.
C. Sales budget. D. Manufacturing O.H budget.
Answer (A) is incorrect because the production budget is based on the sales budget.
Answer (B) is incorrect because expense budgets are based on sales and production budgets.
Answer (C) is correct. The budgeting process normally begins with the sales budget. Following the
preparation of the sales budget, all other budgets are prepared based on the assumptions used in the
sales budget. For this reason, the sales budget is the most difficult to prepare because there are many
external factors affect sales forecast. Sales are greatly based on the desires of consumers,
competition and the current business climate.
Answer (D) is incorrect because the manufacturing O.H budget is based on the production budget

Q4: A large manufacturer’s forecast of total sales revenues for a year is least likely to be influenced by
A. The seasonal pattern of sales revenues throughout the year.
B. Anticipated interest rates and unemployment rates.
C. Expected shortages of key raw materials.
D. Input from sales personnel.
Answer (A) is correct. A large manufacturing firm will be well familiar with the seasonal patterns to
which demand for its products is subject. The forecast is being prepared for an entire year, so the cyclical
effects of varying demand will be taken into account.
Answer (B) is incorrect because interest rates and unemployment levels will have a direct impact on a
manufacturer’s operating and financing budgets.
Answer (C) is incorrect because if shortages of key raw materials are expected, this will have a major
impact on a manufacturer’s budgeting process.

Answer (D) is incorrect because input from all levels of the organization is important to a successful
budget.

Essay: Marvel Products manufactures and wholesales several different lines of luggage in two basic
types; soft-side and molded. Each luggage line consists of several different pieces all of which are in
different sizes. At least one line is a complete set of luggage designed to be used by both men and
women, but some lines and styles are designed specifically for men or women. Some lines also have
matching briefcases. Luggage lines are discontinued and introduced as tastes change or as product
improvements are developed.
Marvel Products also manufactures luggage for large retail companies according to each company’s
specifications. This luggage is marketed under the retail companies’ own private labels rather than the
Marvel label. Marvel has been manufacturing several lines of luggage under its own label and private
lines for one or more retail companies for the last ten years.

REQUIRED:
Identify and discuss the factors Marvel Products needs to consider when developing its sales
component of the annual budget.

Answer
Factors that Marvel Products needs to consider when developing the sales component of its annual
budget include the pricing strategy, size of the market and Marvel's market share and the relationship to
its competitors, sales-mix of products such that contribution can be maximized, available production
capacity, effect of advertising on sales volume, and national and international economic conditions.

See Gleim's MCQ # 8,


15,115,118,135,147,148,149,151,153,154,155,158,160,163,164,165,174,188,189,190,191,192,193,199,

221,222,236,239,242,267,271

2. Production Q budget
The production budget is a plan for acquiring resources and combining them to meet sales goals and
maintain a certain level of inventory.
Note: The decision on how much ending inventory a company should keep is driven by a variety of
factors such as
1. The level of safety stock the company is comfortable with,
2. The sophistication of its inventory management system,
3. The Company's relationship with its suppliers
As a general guideline the inventory levels should be as low as possible without restricting sales.

Quantity to be produced = Projected sales + Desired ending inventory of finished goods + expected
EWIP in E.U – expected beginning inventory of finished goods - expected BWIP in E.U

Note: In purchase Q budget for merchandising firm no WIP exists in the formula

Q1: Nick Klaus is a divisional manager for Tot Toys. He has been assigned the task of creating a
production budget for his division, which produces the company’s most popular stuffed animal
Budgeted sales for this toy for the next year have been set at 650,000 units, desired ending finished
goods inventory is 200,000 units, and Klaus would like there to be 100,000 equivalent units in ending
work-in-process inventory. The starting finished goods inventory for the next year will be 300,000
units, with 75,000 equivalent units in beginning work-in-process inventory. How many equivalent
units should Klaus plan for his division to produce?
A. 550,000 B. 575,000 C. 725,000 D. 925,000
Answer (B) is correct.

Description Quantities
Projected sales –units 650,000
Less: beginning F.G inventory – units (300,000)
Less: beginning WIP inventory – equivalent units (75,000)
Add: ending F.G inventory – units 200,000
Add: ending WIP inventory – equivalent units 100,000
Projected production quantity 575,000

Q2: Lena Company budget contains the following information:


Description Units
Beginning finished goods inventory 85
BWIP in equivalent units 10
Desired ending finished goods inventory 100
Desired EWIP in equivalent units 40
Projected sales 1,800
How many equivalent units should Lena plan to produce?
Budgeted quantity to be produced = Sales - Beg-inventories + Ending inventories
Budgeted quantity to be produced = 1800 - (85 + 10) + (100 + 40) = 1,845 units
Q3: When budgeting, the items to be considered by a manufacturing firm in going from a sales
quantity budget to a production quantity budget would consider
A. Expected change in the quantity of WIP inventories.
B. Expected change in the quantity of finished goods and WIP inventories.
C. Expected change in the quantity of finished goods and raw material inventories.
D. Expected change in the availability of raw material without regard to inventory levels.
Answer (A) is incorrect because finished goods inventories can't be ignored.
Answer (B) is correct. Production quantities are not identical to sales because of changes in inventory
levels. Both finished goods and WIP inventories may change during a period, thus necessitating an
analysis of both inventory levels before the production budget can be set.
Answer (C) is incorrect because work-in-process inventory should be considered.
Answer (D) is incorrect because existing inventories determine production levels.

Q4: Swan Company is a maker of men’s shirts. The company would like to maintain 20,000 yards of
fabric in ending inventory. The beginning fabric inventory is expected to contain 25,000 yards. The
expected yards of fabric needed for sales is 90,000. Compute the yards of fabric that Swan needs to
purchase.
A. 85,000 B. 90,000 C. 95,000 D. 135,000
Answer (A) is correct: Purchases = 90,000 +20,000 -25,000 = 85,000 yards

Q5: Savior Corporation assembles backup tape drive systems for home microcomputers. For the first
quarter, the budget for sales is 67,500 units. Savior will finish the fourth quarter of last year with an
inventory of 3,500 units, of which 200 are obsolete. The target ending inventory is 10 days of sales
(based upon 360 days). What is the budgeted production for the first quarter?
A. 75,000 B. 71,700 C. 71,500 D. 64,350

Answer (B) is correct: The target ending inventory of the first quarter = (67,500 ÷ 90 days) ×10
days = 7,500 units
Budgeted production for the first quarter = 67,500 +7,500 -3,300 =71,700 units

Q6: Ming Company has budgeted sales at 6,300 units for the next fiscal year, and desires to have 590
good units on hand at the end of that year. Beginning inventory is 470 units. Ming has found from past
experience that 10% of all units produced do not pass final inspection, and must therefore be destroyed.
How many units should Ming plan to produce in the next fiscal year?
A. 6,890 B. 7,062 C. 7,133 D. 7,186

Philip answer: Answer (C) is correct:


budgeted.Sales.6,300.Units  Desired .Ending.Inventory.590.Units  beg.Inv.470
= 7,133
90%
Board answer: Answer (D) is correct: The expected production for the next fiscal year would include
both good units and those that fail inspection. Since all the beginning units are good units and there is a
spoilage rate of 10% on new production, we must take the total sales units of 6,300 and divide by 1 -
spoilage rate, or 90%, to figure how many units we need to produce to cover the projected sales. We must
also take that total desired ending inventory of 590 units and divide by 1 - spoilage rate, or 90%, to figure
how many units need to be produced to cover the desired ending inventory.

budgeted.Sales.6,300.Units  Desired .Ending.Inventory.590.Units


Expected production = -
90%
beginning inventory 470 Good units = 7,186

Q7: The shocker company sales budget shows projected quarterly sales for the next year as follows:

Description Q1 Q2 Q3 Q4 Q1 Y2
Projected sales 10,000 8,000 12,000 14,000 15,000

The company policy is to have a finished goods inventory at the end of each quarter equal to 20% of the
next quarter’s sales thus the estimated beginning and ending inventory of finished goods would be as
follows.

Description Q1 Q2 Q3 Q4 Totals
Projected sales 10,000 8,000 12,000 14,000 44,000
Less beg. Inv (2,000) (1,600) (2,400) (2,800) (2,000)
Add Ending inv. 1,600 2,400 2,800 3,000 3,000
Budgeted production 9,600 8,800 12,400 14,200 45,000

Note: Beg. Inventory of the first quarter (Ending inventory of the quarter previous to the first quarter)
= Projected sales of the first quarter 10,000 units × 20% = 2,000 units.

Q8: When presenting a production budget aggregated to the annual level, which of the following is true?
A. Sales, beginning, and ending inventories are not totaled, but the last Quarter's data is used for the
annual totals.
B. Sales are totaled, and beginning and ending inventories are averaged across the quarters.
C. Sales, beginning, and ending inventories are totaled for each of the four quarters.
D. Sales are totaled for the four quarters, but ending inventory counts only the last quarter and beginning
inventory counts only the first quarter.
The correct answer is (D). The sales totals are summed for each quarter, but beginning is taken from first
quarter while ending inventory is for the last quarter.

Q9: Streeter Company produces plastic microwave turntables. Sales for the next year are expected to
be 65,000 units in the first quarter, 72,000 units in the second quarter, 84,000 units in the third quarter
and 66,000 units in the fourth quarter. Streeter maintains a finished goods inventory at the end of each
quarter equal to one half of the units expected to be sold in the next quarter. How many units should
Streeter produce in the second quarter?
A. 72,000 units. B. 75,000 units. C. 78,000 units. D. 84,000 units.

Description Q1 Q2 Q3 Q4
Sales - units 65,000 72,000 84,000 66,000
Less Beginning inventory 32,500 36,000 42,000 33,000
Add Ending inventory 36,000 42,000 33,000 ?
Production 68,500 78,000 75,000 ?
Production = Sales units + Ending inventory - Beginning inventory

Answer (C) is correct:

Q10: From previous example assume that due to a work stoppage, the finished goods inventory at the
end of the first quarter is 8,000 units less than it should be. How many units should Streeter produce in
the second quarter?
A. 75,000 units B. 78,000 units C. 80,000 units D. 86,000 units

Description First quarter Second quarter Third quarter Fourth quarter


Sales - units 65,000 72,000 84,000 66,000
Beg inventory 32,500 28,000 42,000 33,000
End inventory 28,000 42,000 33,000 ?
Production 60,500 86,000 75,000 ?

Production = Sales units + Ending inventory - Beginning inventory

Answer (D) is correct:

Q11: A firm has budgeted sales for the next two periods of 50,000 units and 55,000 units, respectively
the firm maintains a policy that the beginning-of-period inventory is 20% of the following period's
forecasted sales. The policy is currently met. What is the budgeted production for period one?
A. 51,000 units B. 61,000 units C. 50,000 units D. 49,000 units

The correct answer is (A): The budgeted production is calculated by adding budgeted sales
(50,000) to desired ending inventory 55,000 × 20%) minus the beginning inventory (50,000 ×
20%) = 51,000 units

Q12: Daffy tunes manufacture a toy rabbit with moving parts and a built in voice box. Projected
sales in units for the next 5 months are as follows

Description January February March April May


Projected sales in units 30,000 36,000 33,000 40,000 29,000

Each rabbit requires basic materials that Daffy purchases from a single supplier at $3.50 per rabbit.
Voice boxes are purchased from another supplier at $1 each. Assembly labor cost is $2 per rabbit, and
variable OH cost is $0.50 per rabbit, fixed manufacturing O.H applicable to rabbit production is
$12,000 per month. Duffy’s policy is to manufacture 1.50 times the coming month's projected sales
every other month, starting with January (i.e., odd-numbered months) and to manufacture 0.50
times the coming month's projected sales in alternate months (i.e. even-numbered months). This
allows daffy to allocate limited manufacturing resources to other products as needed during the even-
numbered months.
Production Budget in units
Description January February March April May
Projected sales in units 30,000 36,000 33,000 40,000 29,000
Projected production 36× 1½ = 33 × ½ = 40×1½ = 29× ½ = ?
54,000 16,500 60,000 14,500

Costs structure
A) V.C. per unit = (D.M. $ 3.50 + $1) + (D.L. $ 2) + (V.O.H. $ 0.50) = $7
B) Monthly Fixed O.H. $12,000
C) Total monthly production cost Y = $12,000 + $7 X
The dollar production budget for toy rabbits for February is
A. $113,500 B. $390,000 C. $327,000 D. $127,500
Answer (D) is correct. (16,500 × $7) + $12,000 = $127,500
Q13: Assume from previous example that beginning inventory in January is 2,000 units calculate
ending inventory in February
Answer 6,500 units (prove)
Q14: When sales volume is seasonal in nature, certain items in the budget must be coordinated. The three
most significant items to coordinate in budgeting seasonal sales volume are
A. D.L.H, WIP inventory, and sales volume.
B. Production volume, finished goods inventory, and sales volume.
C. Raw material inventory, D.L.H, and manufacturing O.H costs.
D. Raw material inventory, WIPinventory, and production volume.
Answer (A) is incorrect because D.L and WIP are less directly significant to the desired coordination.
Answer (B) is correct. The most important items that need to be coordinated in a seasonal business are
sales volume and production. The sales budget is the basis for other budgets. The sales projection
determines how much needs to be purchased or produced. In turn, projected sales and production (or
purchases) must be coordinated with existing quantities on hand (inventory) and with amounts to be held
in the future. If an enterprise faces sharp variations in demand, this coordination becomes especially
crucial.
Answer (C) is incorrect because D.L, raw materials, and O.H are less directly significant to the desired
coordination.
Answer (D) is incorrect because raw materials and WIP are less directly significant to the desired
coordination.
Q15: Hannon Retailing Company prices its products by adding 30% to its cost. Hannon anticipates sales
of $715,000 in July, $728,000 in August, and $624,000 in September. Hannon’s policy is to have on hand
enough inventories at the end of the month to cover 25% of the next month’s sales. What will be the
budgeted cost of purchase in August?
A. $509,600 B. $540,000 C. $560,000 D. $680,000

Description June July August September


Sales 715,000 728,000 624,000
Cost of sales 550,000 560,000 480,000
Beginning inventory ? 137,500 140,000 120,000
Ending inventory 137,500 140,000 120,000 ?
Purchases 552,500 540,000 ?

Purchases = Cost of sales + Ending inventory - Beginning inventory

Answer (B) is correct:


Q16: Manoli Gift Shop maintains a 35% gross profit percentage on sales, and carries an ending
inventory balance each month sufficient to support 30% of the next month expected sales.
Anticipated sales for the fourth quarter are as follows.

Month October November December


Anticipated sales $42,000 58,000 74,000

What amount of goods should Manoli Gift Shop plan to purchase during the month of November?
A. $40,820 B. $51,220 C. $52,130 D. $62,800

Description October November December


Sales $42,000 58,000 74,000
COGS (65%) 27,300 37,700 48,100
Beginning inventory 8,190 11,310 14,430
Ending inventory 11,310 14,430 ?
Purchases 30,420 40,820 ?

Answer (A) is correct:

Q17: Maker Distributors has a policy of maintaining inventory at 15% of the next month’s forecasted
sales. The cost of Maker’s merchandise averages 60% of the selling price. The inventory balance as of
May 31 is $63,000, and the forecasted dollar sales for the last seven months of the year are as follows.

Month June July August September October November December


Sales $700,000 600,000 650,000 800,000 850,000 900,000 840,000

What is the budgeted dollar amount of Maker’s purchases for July?


A. $355,500 B. $360,000 C. $364,500 D. $399,000

Description June July August September October November December


Sales $700,000 600,000 650,000 800,000 850,000 900,000 840,000
Cost of sales (60 %) 420,000 360,000 390,000 480,000 510,000 540,000 504,000
Beginning inventory 63,000 54,000 58,500 72,000 76,500 81,000 75,600
Ending inventory 54,000 58,500 72,000 76,500 81,000 75,600 ?
Purchases 411,000 364,500 403,500 484,500 514,500 534,600 ?

Answer (C) is correct:

Q18: Tidwell Corporation sells a single product for $20 per unit. All sales are on account, with
60% collected in the month of sale and 40% collected in the following month. A partial schedule
of cash collections for January through March of the coming year reveals the following receipts
for the period.

Description Cash receipts


January February March
December receivables $32,000
From January sales 54,000 $36,000
From February sales 66,000 $44,000

Other information includes the following.


• Inventories are maintained at 30% of the following month’s sales.
• Assume that March sales total $150,000.
The number of units to be purchased in February is
A. 3,850 units. B. 4,900 units. C. 6,100 units. D. 7,750 units.

Description January February March


Sales $ 54,000+$36,000= 90,000 66,000 +$44,000=110,000 $150,000
Sales Q 4,500 5,500 7,500
Beginning inventory-units 4,500×30%=1,350 1,650 2,250
Ending inventory-units 5,500×30%=1,650 7,500×30%=2,250
Purchases – units 4,800 6,100

Answer (C) is correct:


Notes:
December sales $ = $32,000 / 0.40 = $80,000

December sales Q = $80,000 ÷ $20 = 4,000 units

3. Direct materials budgets


The production budget specifies only the number of units to be produced; Then D.M usage budgets
are to be prepared after determining the required materials Quality level ; D.M usage budgets
specifies the amount and cost of materials needed for the production and the amount and cost of
materials that must be purchased to meet the production requirement.
Q1: In preparing the D.M purchases budget for next quarter, the plant controller has the following
information available.
Budgeted unit sales 2,000
Pounds of materials per unit 4
Cost of materials per pound $3
Pounds of materials on hand 400
finished units on hand 250
Target ending units inventory 325
Target ending inventory of pounds of materials 800
how many pounds of materials must be purchased?
A. 2,475 B. 7,900 C. 8,700 D. 9,300

♠Production quantity of finished product =2,000 - 250 + 325 = 2,075 units


♠Pounds of materials needed for production = 2,075 unit sales × 4 Pounds = 8,300 Pounds
♠Pounds of materials to be purchased = 8,300 - 400 + 800 = 8,700 Pounds
Answer (C) is correct:

Q2: Playtime Toys estimates that it will sell 200,000 dolls during the coming year. The beginning
inventory is 12,000 dolls; the target ending inventory is 15,000 dolls. Each doll requires two shoes
which are purchased from an outside supplier. The beginning inventory of shoes is 20,000; the target
ending inventory is 18,000 shoes. The number of shoes that should be purchased during the year is
A. 396,000 shoes. B. 398,000 shoes C. 402, 000 shoes D. 404,000 shoes
♠Dolls to be produced =200,000 - 12,000 +15,000 = 203,000 Dolls
♠Shoes required for production budget = 203,000 Dolls × 2 shoes = 406,000 shoes
♠Shoes to be purchased = 406,000 -20,000 +18,000 = 404,000 shoes
Answer (D) is correct:
Q3: The Jung Corporation’s budget calls for the following production:
Description Q1 Q2 Q3 Q4
Projected production 45,000 38,000 34,000 48,000

Each unit of product requires three pounds of direct material. The company’s policy is to begin each
quarter with an inventory of direct D.M equal to 30% of that quarter’s D.M requirements.
Budgeted D.M purchases for the each of the first three quarters would be.

Description Q1 Q2 Q3 Q4
Projected production 45,000 38,000 34,000 48,000
Projected D.M requirements 135,000 114,000 102,000 144,000
Beg. D.M. inv. (40,500) (34,200) (30,600) (43,200)
Ending D.M. inv 34,200 30,600 43,200 ?
Projected R.M. purchases 128,700 110,400 114,600 ?

Q4: A company has budgeted sales for the upcoming quarter as follows:
Description January February March
Budgeted sales - Units 15,000 18,000 16,500

The ending finished goods inventory for each month equals 50% of the next month’s budgeted
sales. Additionally, 3 pounds of raw materials are required for each finished unit produced. The
ending raw materials inventory for each month equals 200% of the next month’s production
requirements. If the raw materials cost $4 per pound and must be paid for in the month purchased,
what is the budgeted raw materials purchase (in dollars) for January?

Description January February March


Budgeted sales -Units 15,000 18,000 16,500
Less beg. Inv F.G (7,500) (9,000) (8,250)
Add Ending inv. F.G 9,000 8,250 ?
Projected production 16,500 17,250 ?
D.M usage Q budget (3 pounds) 49,500 51,750 ?
Less Beg. Inv. Raw materials (99,000) 103,500 ?
Add Ending inv. Raw materials 103,500 ? ?
Budgeted R.M. purchases-Units 54,000 ? ?
Raw materials cost per unit $4 ? ?
Raw materials budgeted cost $ 216,000 ? ?

Q5: Maximilian Computer Company uses a comprehensive budgeting system in planning its annual
operations. Which of the following best describes the information needed in order to determine the
budgeted cost of circuit boards to be purchased for use in building its laptop computer? Assume
one circuit board is used in each laptop.
A. Begin with budgeted laptop sales in units, add the desired ending inventory of circuit boards,
deduct the expected beginning inventory of circuit boards, and multiply the resulting amount by the
budgeted purchase cost per circuit board.
B. Begin with budgeted laptop sales in units, deduct the desired ending inventory of circuit boards,
add the expected beginning inventory of circuit boards, and multiply the resulting amount by the
purchase cost per circuit board.
C. Begin with budgeted laptop production in units, deduct the desired ending inventory of circuit
boards, add the expected beginning inventory of circuit boards, and multiply the resulting amount
by the purchase cost per circuit board.
D. Begin with budgeted laptop production in units, add the desired ending inventory of circuit
boards, deduct the expected beginning inventory of circuit boards, and multiply the resulting
amount by the budgeted purchase cost per circuit board.

Correct answer is (D): To determine the budgeted cost of purchases, the number of units to be produced
should be adjusted by the change in inventory and then multiplied by the budgeted purchase cost.

Q6: The direct materials budget is often broken down into


A. D.M yield budget and D.M usage budget. B. D.M mix budget and D.M yield budget.
C. D.M usage budget and D.M purchase budget. D. D.M mix budget and D.M purchase budget.
The correct answer is (C).
Q7: Stevens Company manufactures electronic components used in automobile manufacturing. Each
component uses two raw materials, Geo and Clio. Standard usage of the two materials required to
produce one finished electronic component, as well as the current inventory, are shown below.

Material Standard Per Unit Price Current Inventory


Geo 2 pounds $15/lb. 5,000 pounds
Clio 1.50 pounds $10/lb. 7,500 pounds

Stevens's forecasts production of 20,000 components for the next two production periods.
Company policy dictates that 25% of the raw materials needed to produce the next period’s projected
sales be maintained in ending direct materials inventory.
Based on this information, the budgeted D.M purchases for the coming period would be:

Choice Geo Clio


A $450,000 $450,000
B $675,000 $300,000
C 675,000 $400,000
D $825,000 $450,000

Description Geo Clio


Raw materials requirements for production budget (20,000 units) 40,000 pounds 30,000 pounds
Beginning direct materials inventory 5,000 7,500
Ending direct materials inventory 10,000 7,500
Direct materials purchases - units 45,000 pounds 30,000 pounds
Price per pound $15 $10
Total purchases cost $675,000 $300,000

Answer (B) is correct:

Q8: A company produces a product that requires 2 pounds of a raw material. The company forecasts
that there will be 6,000 pounds of raw material on hand at the end of June. At the end of any given
month the company wishes to have 30% of next month's raw material requirements on hand. The
company has budgeted production of the product for July, August, September, and October to be 10,000,
12,000, 13,000, and 11,000 units, respectively. As of June 1, the raw material sells for $1per pound. In
the month of September, raw material purchases and ending inventory, respectively, will be (in pounds):
A. 24,800 and 6,600 B. 28,600 and 6,600
C. 32,600 and 6,600 D. 13,000 and 3,900

Answer (A) is correct:


1. Beg D.M inventory for September = September production 13,000 Units × 2 Units of D.M ×
30% = 7,800 Units
2. Ending D.M inventory for September = October production 11,000 Units × 2 Units of D.M
× 30% = 6,600 Units
3. D.M purchases during September = September production 13,000 Units × 2 + Ending D.M
inventory 6,600 Units - Beg D.M inventory 7,800 Units = 24,800 Units

Q9: A company produces a product that requires 2 pounds of a raw material. The company forecasts
that there will be 6,000 pounds of raw material on hand at the end of June. At the end of any given
month the company wishes to have 30% of next month's raw material requirements on hand. The
company has budgeted production of the product for July, August, September, and October to be
10,000, 12,000, 13,000, and 11,000 units, respectively. As of June 1, the raw material sells for $1 per
pound. The cost of inventory is determined using the last-in-first-out (LIFO) method. If the price of raw
material increases 10% as of June 30, what will be the effect of this increase on the cost of purchases
from July to September?
A. $3,230 increase B. $60 increase C. $600 increase D. $7,060 increase
Answer (D) is correct:
1. Beginning inventory July 6,000 Units (Given)
2. Ending inventory September = 11,000 × 30% × 2 = 6, 600 Units
3. D.M to be used from July to September = (10,000 + 12,000 + 13,000) × 2 = 70,000 Units
4. D.M purchases for the period from July to September = 70,000 + 6,600 – 6,000 = 70,600 Units
5. Increase in cost paid for D.M purchases = 70,600 Units × $1 × 10% = $7,060

Q10: Werner Company buys raw materials from several suppliers, and makes payments according to
the following schedule.
In the month of purchase 25%
In the month after purchase 60%
In the second month after purchase 15%
In preparing the master budget for the fourth quarter of the year, Werner assumed that total purchases
for the quarter would be spread evenly over the three months. In its pro forma balance sheet, Werner
anticipated a December 31 A/P balance of $207,000. What amount of purchase did Werner anticipate
for the fourth quarter of the year?
A. $496,800 B. $558,900 C. $621,000 D. $690,000
Answer (D) is correct:
The A/P balance at year end composed of
75% from December purchases
15% from November purchases
Werner assumed that total purchases for the quarter would be spread evenly over the three months; this
means that purchases of each month are equal thus
$207,000
Total purchases during the quarter = × 3 Months = $690,000
90%

Q11: Barolo Company plans to sell 200,000 units of finished product in July and anticipates a growth
rate in sales of 5% per month. The desired monthly ending units of finished product are 80% of the
next month’s estimated sales. There are 150,000 finished units in inventory on June 30. Each unit of
finished product requires 4 pounds of D.M at a cost of $ 1.20 per pound. There are 800,000 pounds of
D.M in inventory on June 30.
Description July August September Total
Estimated sales – units 200,000 210,000 220,500 630,500
Less beg. Inv -F.G (150,000) Given (168,000) (176,400) ------
Add Ending inv.- F.G 168,000 176,400 185,220 ------
Projected production– units 218,000 218,400 229,320 665,720
Raw materials requirements (4 pounds) 872,000 873,600 917,280 2,662,480

Assume Barolo Company plans to produce 600,000 units of finished product in the three month
period ending September, 30, and to have D.M inventory on hand at the end of the three month
period equal to 25% of the D.M use in the same period. Calculate the estimated cost of direct
material purchases for the three month period ending September, 30
Description Amount
Production requirement of raw material 600,000 × 4 2,400,000 units
Beginning inventory - direct materials (given) (800,000)
Ending inventory 25% of the use 2,400,000 × 25% 600,000
Budgeted raw material purchases-Units during the period Estimated 2,200,000 units
cost of D.M. purchases during the period $1.20 × 2,200,000 units $2,640,000

Q12: A firm desires a finished goods ending inventory equal to 25% of the following month’s
budgeted sales, January sales are budgeted at 10,000 units and February sales are budgeted at 12,000
units, each unit requires 2 pounds of material X that cost $4 per pound. The company has a just-in-
time system and materials are delivered daily just prior to use, so no raw materials inventories are
maintained. Materials are paid for in the month following purchases. In February, what amount
should the company expect to pay as cash outflow for raw materials?

Description Jan Feb


Budgeted sales - units 10,000 12,000
Beginning inventory - units (25% of Jan budgeted sales) 2,500 3,000
Ending inventory units -25% of the following month’s budgeted sales 3,000 ?
Budgeted production - units 10,500

D.M requirements for January production schedule


= (10,500 units × 2 pounds of material X) = 21,000 pounds
Costs of material (X) to be paid in Feb. = 21,000 × 4 = $84,000

Fact pattern: Superflite expects April sales of its deluxe model airplane, the C-14, to be 402,000 units
at $11 each. Each C-14 requires three purchased components shown below.

Part No Purchase Cost Number Needed for each C- 14 Unit


A-9 $0.50 1
B-6 0.25 2
D-28 1 3
Factory D.L and variable O.H per unit of C-14 totals $3. Fixed factory O.H is $1 per unit at a
production level of 500,000 units. Superflite plans the following beginning and ending inventories for
the month of April and uses standard absorption costing for valuing inventory.

Part No Units at April 1 Units at April 30


C-14 12,000 10,000
A-9 21,000 9,000
B-6 32,000 10,000
D-28 14,000 6,000

Q13: Superflite’s C-14 production budget for April should be based on the manufacture of
A. 390,000 units. B. 400,000 units. C. 402,000 units. D. 424,000 units.
Answer (B) is correct.

Projected April sales units of C-14 402,000 units


Less beginning inventory of C-14 (12,000) units
Add ending inventory of C-14 10,000 units
Projected April production of C-14 400,000 units

Q14: Assume Superflite plans to manufacture 400,000 units in April. Superflite's April budget
for the purchase of A-9 should be
A. 379,000 units. B. 388,000 units. C. 402,000 units. D. 412,000 units.
Answer (B) is correct

Description A-9 B-6 D-28


Production requirements of D.M 400,000 800,000 1,200,000
less beginning inventory of D.M (21,000) (32,000) (14,000)
Add ending inventory of D.M 9,000 10,000 6,000
D.M purchases quantities 388,000 778,000 1,192,000
Per unit D.M purchase cost $0.50 0.25 1
D.M purchases cost budget $194,000 $194,500 $1,192,000

Q15: Assume Superflite plans to manufacture 400,000 units in April then

Choice Total fixed factory O.H Per unit fixed factory O.H
A $400,000 $1
B $500,000 $1
C $500,000 $1.25
D $400,000 $1.25

Answer (C) is correct: fixed factory O.H is constant in total and varies per unit

Q16: Assume Superflite plans to manufacture 400,000 units in April. What is total manufacturing
cost per unit of C-14?
A. $8 B. $8.25 C. $7.25 D. $11
Answer (B) is correct.
D.M = [(1 × $0.50) + (2 × $0.25) + (3 × $1)] = $4

D.L + Variable factory O.H = $3


Fixed factory overhead = $500,000/400,000 units = $1.25

Total manufacturing cost per unit = $8.25

Q17: Assume Superflite plans to manufacture 400,000 units in April. The total April budget for all
purchased components should be
A. $1,596,500 B. $1,608,500 C. $1,600,000 D. $1,580,500

Answer (D) is correct.

To solve these question we should use the formula for the physical flow of inventory to
calculate the amount to be purchased for each component (A-9, B-6 and D-28), then use the
price of each component to calculate the total amount to be budgeted for April's production.
Beginning Inventory + Units Purchased- Units Used in Production = Ending Inventory
For A-9: one unit of A-9 is necessary for each unit of finished product. Plugging numbers for
A-9 into the formula we will get the following: 21,000 + Units Purchased - (400,000 × 1) =
9,000. Solving for Units Purchased, we get Units Purchased = 388,000. The cost to purchase
388,000 units of A-9 $194,000 ($0.50 ×388,000)
For B-6: two units of B-6 are necessary for each unit of finished product. Plugging numbers
for B-6 into the formula we will get the following: 32,000 + Units Purchased � (400,000 × 2)
= 10,000. Solving for Units Purchased, we get Units Purchased = 778,000. The cost to
purchase 778,000 units of B-6 is $194,500 ($0.25 × 778,000).
For D-28: three units of D-28 are necessary for each unit of finished product. Plugging
numbers for D-28 into the formula we get the following: 14,000 + Units Purchased � (400,000
× 3) = 6,000. Solving for Units Purchased, we get Units Purchased = 1,192,000. The cost to
purchase 1,192,000 units of D-28 is $1,192,000 ($1.00 × 1,192,000).
Adding the costs to purchase all of the components together, we get $1,580,500 ($194,000 +
$194,500 + $1,192,000)

Q18: Which of the following correctly describes the materials purchase budget calculation?
A. (Sales forecast (in units) × D.M per unit × D.M cost) + beginning D.M inventory - desired ending
D.M inventory
B. (Sales forecast (in units) × D.M per unit × D.M cost) + desired ending D.M inventory - beginning
D.M inventory
C. (Number of units to be produced × D.M per unit × D.M cost) + beginning D.M inventory - desired
ending D.M inventory
D. (Number of units to be produced × D.M per unit × D.M cost) + desired ending D.M inventory -
beginning D.M inventory

The correct answer is (D): The direct materials usage budget specifies the direct materials needed to
meet production demands Therefore, one must begin with the number of units to be produced (not the
number of units forecast in sales) Since beginning inventory is already on hand, it is subtracted while
desired ending inventory must be met with production needs.

Q: A company has budgeted sales of 24,000 finished units for the forthcoming 6-month period.
It takes 4 pounds of D.M to make one finished unit. Given the following:

Description Finished units Direct materials (pounds)


Beginning inventory 14,000 44,000
Target ending inventory 12,000 48,000

How many pounds of direct materials should be budgeted for purchase during the 6-month
period?
A. 48,000 B. 88,000 C. 96,000 D. 92,000
Answer (D) is correct:
1. Forecasted production of F.G = 24,000 + 12,000 – 14,000 = 22,000 Units
2. Budgeted purchases of D.M = (22,000 × 4) + 48,000 – 44,000 = 92,000

4. Direct labor budget

The D.L budget, is prepared by the production manager and human resources, it specifies the D.L
requirement needed to meet the production needs. The D.L budget can help firms plan production
processes to smooth out production over a year and keep a consistent workforce size throughout the
year. Labor budgets are usually broken down into categories such as unskilled, semi-skilled, and skilled.

Q1: A firm with union labor is contractually required to give several months' notice before any layoffs
can occur. Which of the following would best allow the company to make such an assessment?

A. Direct labor budget B. Actual labor results


C. Production budget D. Prior year's labor results

The correct answer is (A). A direct labor budget allows firms to predict their staffing needs in
advance, so that any required union negotiations can occur within a reasonable amount of time.

Q2: Which of the following is not a quality (feature) of a D.L budget?

A. D.L budget can smooth out production over the year to keep work force size consistent.
B. D.L budget allows firms with unions to notify the union before changes are needed.
C. D.L budget can be broken down by categories such as semiskilled, unskilled, and skilled.
D. D.L budget can be broken down into fixed and variable D.L.

Answer (D) is Correct. D.L budgets are not broken down into fixed and variable D.L because
all D.L is assumed to be variable.

Q3: The pro forma statement of employee benefit costs, a budget schedule that is prepared as part
of an organization’s annual profit plan, would include costs related to
A. Employees’ gross wages, salaries and the related company-paid benefits.
B. Employees’ net wages, salaries and the related company-paid benefits.
C. All payroll related deductions withheld from employees and company-paid benefits.
D. Company-paid benefits and company-paid payroll taxes.
Answer (D) is correct. Using a pro forma statement of employee benefit costs allows a company
to view the cost associated with company-paid benefits and company-paid payroll taxes.

Q4: Petersons Planters Inc. budget includes the following amounts for the coming year.
Beginning inventory, finished goods $10,000
COGS $400,000
D.M to be used in production $100,000
Ending inventory, finished goods $25,000
Beginning and ending WIP inventory Zero
O.H is estimated to be two times the amount of D.L dollars. The amount that should be
budgeted for direct labor for the coming year is
A. $315,000 B. $210,000 C. $157,500 D. $105,000

DR Finished goods inventory CR


$10,000 Beginning inventory COGS $400,000
415,000 Cost of goods manufactured Ending inventory 25,000
425,000 Total Total 425,000

DR WIP CR
0 BWIP COGM $415,000
100,000 D.M (Given)
105,000 D.L (415,000 -100,000)×1/3 EWIP 0
210,000 O.H(415,000 -100,000)×2/3
415,000 Total Total 415,000

Answer (D) is correct:

Q5: A firm has projected sales of 20,000 and 22,000 units for the next two periods (periods 1 and 2,
respectively). The firm maintains an ending finished goods inventory equal to 15% of the next period's
projected sales. Each unit requires 2 D.L.H at a cost of $15 per hour. Assuming that all inventory
standards are currently met, what is the D.L Dollar budget for period 1?
A. $609,000 B. $600,000 C. $699,000 D. $690,000
Answer (A) is correct:
Number of units that must be produced in period =20,300
D.L budget = 20,300 × 2 hours × $15 = $609,000.

Q6: Data regarding Rombus Company’s budget are shown below.


Planned sales 4,000 units
Material cost $2.50 per pound
D.L 3 hours per unit
D.L rate $7 per hour
Finished goods beginning inventory 900 units
Finished goods ending inventory 600 units
D.M beginning inventory 4,300 units
D.M ending inventory 4,500 units
Materials used per unit 6 pounds
Rombus Company’s production budget will show total units to be produced of
A. 3,700 B. 4,000 C. 4,300 D. 4,600
Answer (A) is correct: Production budget = 4,000 + 600 -900 = 3,700 units
From previous example calculate the following
1. D.M usage Q ►3,700 unit's × 6 pounds = 22,200 pounds
2. D.M purchase Q ►22,200 pounds + 4,500 pounds -4,300 pounds = 22,400 pounds
3. D.M usage $ ►22,200 pounds × $2.5 = $55,500
4. D.M purchase $ ►22,400 pounds × $2.5 = $56,000
5. D.L hour's budget►3,700 unit's × 3 hours per unit =11,100 hours
6. D.L cost budget ►11,100 hour's × $7 per hour = $77,700
7. Budgeted prime cost per unit ► (6 pounds× $2.5) + (3 hours per unit × $7 per hour) = $36
Fact pattern: Mountain Corporation manufactures cabinets but outsourcers the handles. Eight
handles are needed for a cabinet, with assembly requiring 30 minutes of D.L per unit. Ending finished
goods inventory is planned to consist of 50% of projected unit sales for the next month and ending
handles inventory is planned to be 80% of the requirement for the next month’s projected unit output
of finished goods.

Month October November December January


Projected unit sales 4,600 5,000 4,200 6,000

Mountain’s ending inventories in units at September 30:


Finished goods 3,800
Handles 16,000

Q7: The number of units that Mountain finished during December is


A. 3,000 B. 5,100 C. 4,200 D. 5,000
Answer (B) is correct.

Q8: The number of handles, Mountain should purchase in October is


A. 39,840 B. 76,800 C. 40,000 D. 36,800
Answer (A) is correct.

Description October November December January


Projected unit sales 4,600 5,000 4,200 6,000
Beginning inventory – F.G (3,800) (2,500) (2,100) (3,000)
Ending inventory – F.G 2,500 2,100 3,000
Projected production quantity 3,300 4,600 5,100
D.M Q required for production 26,400 36,800 40,800
Beginning D.M inventory (16,000) (29,440) (32,640)
Ending D.M inventory 29,440 32,640 ?
D.M purchases quantity 39,840 40,000

Q9: Given that a full-time employee works 160 hours per month, no overtime is allowed, and part-
time employees may be used, how many full-time equivalent employees does Mountain need to
assemble the output of finished units in November?
A. 14.375 B. 28.75 C. 15.625 D. 31.25
Answer (A) is correct.
-November Projected production quantity = 4,600 units
-Assembly time needed for November Projected production quantity = 4,600 units ÷ ½ DLH =2,300
DLH
- Number of full-time equivalent employees needed = 2,300 DLH ÷160 hours per month = 14.375
equivalent employees

Fact pattern: Rocket Corporation is a manufacturer of tables sold to schools, restaurants, hotels, and
other institutions. The table tops are manufactured by Rocket, but the table legs are purchased from an
outside supplier. The Assembly Department takes a manufactured table top and attaches the four
purchased table legs. It takes 20 minutes of labor to assemble a table. The company follows a policy of
producing enough tables to ensure that 40% of next month’s sales are in the finished goods inventory.
Rocket also purchases sufficient raw materials to ensure that D.M inventory is 60% of the following
month’s scheduled production.
Rocket's sales budget in units for the next quarter is as follows:
Month July August September
Projected sales quantity 2,300 2,500 2,100

Rocket's ending inventories in units for June 30 are


Finished goods 1,900
Direct materials (legs) 4,000

Q10: The number of tables to be produced by Rocket during August is


A. 1,400 tables. B. 2,340 tables. C. 1,440 tables. D. 1,900 tables.
Answer (B) is correct.

Q11: Assume Rocket's required production for August and September is 1,600 and 1,800 units,
respectively, and the July 31 D.M inventory is 4,200 units. The number of table legs to be purchased
in August is
A. 6,520 legs. B. 9,400 legs. C. 2,200 legs. D. 6,400 legs.

Answer (A) is correct.

Month July August September


Projected sales quantity 2,300 2,500 2100
Beginning inventory – F.G (1,900) (1,000) (840)
Ending inventory – F.G 1,000 840 (2,340)
Projected production quantity 1,400 2,340 ?
D.M Q required for production 5,600 9,360 ?
Beginning D.M inventory (4,000) (5,616)
Ending D.M inventory 5,616 ?
D.M purchases quantity 7,216

Description August September


Required production (assumed) 1,600 1,800
D.M Q required for production 6,400 7,200
Beginning D.M inventory –assumed 4,200
Ending D.M inventory 4,320
D.M purchases quantity 6,520

Q12: Assume that Rocket Corporation will produce 1,800 units in the month of September. How
many employees will be required for the Assembly Department? (Fractional employees are
acceptable since employees can be hired on a part-time basis. Assume a 40-hour week and a 4-
week month.)
A. 15 employees. B. 3.75 employees. C. 60 employees. D. 600 employees.
Answer (B) is correct
- September Projected production quantity = 1,800 units
-Assembly time needed for September Projected production quantity = [(1,800 units × 20 minutes)] ÷
[60 minutes] = 600 DLH
- Number of full-time equivalent employees needed = 600 DLH ÷ (40 hours per week × 4 weeks) =
3.75 equivalent employees
Fact pattern: Jordan Auto has developed the following production plan:

Month January February March April


Units 10,000 8,000 9,000 12,000

Each unit contains 3 pounds of D.M. The desired D.M ending inventory each month is 120% of the
next month’s production, plus 500 pounds. (The beginning inventory meets this requirement.) Jordan
has developed the following D.L standards for production of these units

Description Department 1 Department 2


Hours per unit 2 0.50
Hourly rate $6.75 $12

Q13: How much D.M should Jordan Auto purchase in March?


A. 27,000 pounds B. 32,900 pounds C. 36,000 pounds D. 37,800 pounds
Answer (D) is correct.

Q14: Jordan Auto’s total budgeted D.L dollars for February usage should be
A. $156,000 B. $165,750 C. $175,500 D. $210,600
Answer (A) is correct.

Month January February March April


Production Units 10,000 8,000 9,000 12,000
D.M Q required for production 30,000 24,000 27,000 36,000
Beginning D.M inventory (36,500) (29,300) (32,900) (43,700)
Ending D.M inventory 29,300 32,900 43,700 ?
D.M purchases quantity 22,800 27,600 37,800 ?

Month January February March April


Production Units 10,000 8,000 9,000 12,000
Hours per unit- Department 1 20,000 16,000 18,000 24,000
Cost - Department 1 $135,000 $108,000 $121,500 $162,000
Hours per unit- Department 2 5,000 4,000 4,500 6,000
Cost - Department 2 $60,000 $48,000 $54,000 $72,000
Total D.L cost $195,000 $156,000 $175,500 $234,000

5. Overhead budget (Factory O.H budget)

All other production costs that are not in the D.M and D.L budgets are in the O.H budget, sometimes
called a fixed costs budget because most of the costs in this category do not vary with the rise and fall
of production. Factory and related facilities, Rent and insurance, for instance, remain stable even if
production goes up or down. However, there are some O.H costs that do vary with production;
variable costs such as batch setup costs or the costs of electricity and other utilities. Fixed costs are
easy to budget, but the variable costs require forecasting the number of units to be produced, the
production methods used, and other external factors.
Q: Warren Company uses departmental rates to assign O.H to its two products. Budgeted data for the
next year are shown below.

Description Department (1) Department (2)


O.H costs $5,000,000 $7,000,000
Cost driver M.H Labor hours
M.H 500,000 120,000
Labor hours 100,000 1,400,000

Warren expects to manufacture 400,000 units of Product A during the year. Each unit of Product A
requires 0.50 M.H in Department 1 and 1.50 labor hours in Department 2. The budgeted O.H cost
for one unit of Product A is
A. $11.03 B. $12.50 C. $12.73 D. $15
Answer (B) is correct: $5 + $7.50 = $12.50

Description Department (1) Department (1)


O.H costs $5,000,000 $7,000,000
Cost driver 500,000 M.H 1,400,000 D.L.H
O.H rate $10 per M.H $5 per DLH
O.H Allocated $10 × 0.50 M.H = $5 $5 × 1.50 DLH = $7.50

6. COGM budget

The information contained in this budget must directly relate to BWIP, D.M used D.L., O.H.
applied and EWIP.

Example

DR WIP CR
BWIP $10,000 COGM $125,000
D.M. used $50,000
D.L. $40,000 EWIP $35,000
O.H. applied $60,000
TMC $160,000 Total $160,000

COGM budget
BWIP $10,000
Add: D.M. to be used 50,000
Add: D.L. 40,000
Add: O.H. to be applied 60,000
TMC 160,000
Less: EWIP 35,000
COGM 125,000

Q1: Adams Manufacturing, Inc. produces farm tractors. The details of its budgeted cost of goods
manufactured schedule should come from which of the following schedules?
A. Cost of goods sold plus or minus the change planned in finished goods.
B. D.M to be used, D.L, manufacturing O.H, and WIP.
C. Purchases, D.L, manufacturing O.H, finished goods, and WIP.
D. Purchases, raw material, WIP, and finished goods.
Answer (A) is incorrect because COGS equals the COGM adjusted for the change in finished goods.
Also, finished goods are not a part of the COGM calculation.
Answer (B) is correct. COGM equals all manufacturing costs incurred during the period, plus
BWIP, minus EWIP. The COGM schedule therefore includes D.M used; D.L, factory O.H applied,
and changes in WIP inventories.
Answer (C) is incorrect because a purchase is a component of the raw materials budget, not COGM
schedule, and finished goods are not included in COGM.
Answer (D) is incorrect because COGM includes D.M used, not purchases or finished goods.

Q2: The information contained in a COGM budget most directly relates to the
A. Materials used, D.L, factory overhead applied, and ending WIP budgets.
B. Materials used D.L, factory O.H applied, and WIP inventories budgets.
C. Materials used D.L; factory O.H applied, WIP inventories, and F.G inventories budgets.
D. Materials used D.L; factory O.H applied, and F.G inventories budgets.
Answer (A) is incorrect because both beginning and ending work-in-process must be included.
Answer (B) is correct. COGM is equivalent to a retailer’s purchases. It equals all
manufacturing costs incurred during the period, plus BWIP, minus EWIP. COGM budget is
therefore based on D.M used, D.L, factory O.H applied, and WIP inventories.
Answer (C) is incorrect because finished goods are excluded. They are the end product of the
manufacturing process.
Answer (D) is incorrect because finished goods are excluded. They are the end product of the
manufacturing process.

Q3: Krouse Company is in the process of developing its operating budget for the coming year. Given
below are selected data regarding the company’s two products, forged putter heads and laminated
putter heads that are sold through specialty golf shops?

Description Putter Heads


Forged Laminated
Raw materials
Steel 2 pounds @ $5/lb. 1 pound @ $5/lb.
Copper None 1 pound @ $15/lb.
Direct labor 1/4 hour @ $20/hr 1 hour @ $22/hr
Expected sales (units) 8,200 2,000
Selling price per unit $30 $80
Ending inventory target (units) 100 60
Beginning inventory (units) 300 60
Beginning inventory (cost) $5,250 $3,000

Manufacturing O.H is applied to units produced on the basis of D.L hours. Variable manufacturing O.H is
projected to be $25,000, and fixed manufacturing O.H is expected to be $15,000.
The estimated cost to produce one unit of the laminated putter head is

A. $42 B. $46 C. $52 D. $62


Description Forged Laminated

Expected sales (units) 8,200 2,000

Add: Ending inventory target (units) 100 60

Less: Beginning inventory (units) 300 60

Units to be produced 8,000 2,000

Direct labor hours per unit 1/4 hour 1 hour

Total Direct labor hours needed 2,000 2,000

Predetermined variable O.H allocation rate =$25,000/4,000 DLH = $6.25 per DLH

Predetermined fixed O.H allocation rate =$15,000/4,000 DLH = $3.75 per DLH

Description Forged Laminated

Raw materials (2 × 5) = $10 (1×5) + (1×15) = $20

Direct labor 1/4× $20 =$5 (1 × 22) = $22

variable O.H $6.25 ×1/4 = $1.5625 $6.25 ×1= $6.25

Fixed O.H $3.75 × 1/4 = 0.9375 $3.75 × 1= $3.75


Total manufacturing cost per unit $17.50 $52

Answer (C) is correct:

Q4: A company has the following budget data:


Beginning finished goods inventory 40,000 units
Sales 70,000 units
Ending finished goods inventory 30,000 units
Direct materials $10 per unit
Direct labor $20 per unit
Variable factory O.H $5 per unit
Selling costs $2 per unit
Fixed factory O.H $80,000
What will be the total budgeted production costs?
A. $2,100,000 B. $2,300,000 C. $2,180,000 D. $2,220,000
Answer (C) is correct: production quantity = sales Q 70,000 + Ending F.G 30,000 –
Beginning F.G 40,000 = 60,000 Units
Budgeted production costs = (60,000 Units × $35) + $80,000 = $2,180,000

7. Cost of goods sold budget


The cost of goods sold budget indicates the total cost of producing the product sold for a
period. This budget is sometimes called the cost of goods manufactured and sold budget,
because it often also includes items budgeted to be in inventory. This budget is created only
after the production, D.M, D.L, and factory O.H budgets are formed, because it is basically a
summary of these budgets.
DR Finished Goods CR
Beginning F.G inventory $60,000 COGS $145,000
Cost of goods manufactured $125,000 Ending F.G inventory $40,000
Cost of goods available for sale $185,000 Total $185,000

COGS budget
Beginning F.G inventory $60,000
Cost of goods manufactured 125,000
Cost of goods available for sale 185,000
Less Ending F.G inventory 40,000
Budgeted COGS 145,000

Q1: A COGS budget for a manufacturing firm contains which of the following primary components?

A. Cost of goods manufactured and finished goods inventory balances

B. Budgeted D.M to be used and budgeted D.L

C. Budgeted D.M purchases, budgeted D.L, and budgeted factory O.H

D. Cost of goods manufactured and D.M inventory balances

The correct answer is (A). COGS for a manufacturing firm is defined as beginning F.G inventory plus
COGM less ending F.G inventory. The COGM may be expanded in this report, where COGM is defined
as budgeted D.M used (not purchased), budgeted D.L, and budgeted factory O.H.

Q2: Which of the following is true of the COGS budget?

A. the COGS budget includes a category for summing up the selling and administrative expenses.
B. the COGS budget sometimes includes items budgeted to be in inventory and not sold.
C. the COGS budget usually shows per unit cost for D.L and D.M.
D. the COGS budget is created before the D.M, D.L, and O.H budgets.

The correct answer is (B). The COGS budget is sometimes called the cost of goods manufactured
and sold budget, since this budget often also includes items budgeted to be in inventory.

Q3: Which of the following would a merchandising company use in place of cost of goods
manufactured on a statement of cost of goods sold?

A. Purchases B. Prime costs C. Materials inventory D. Conversion costs


The correct answer is (A). The income statement for a merchandising company looks similar
to that of a manufacturing company, except that it uses purchases instead of cost of goods
manufactured.
Fact pattern: Wellfleet Company manufactures Tricycles and Bicycles and prepares annual
operational budgets for each department the purchasing department is finalizing plans for the fiscal
year ending June 30 year two and has gathered this information regarding two of the components
used in both tricycles and bicycles. Wellfleet uses the first in first out inventory method.

Description A 19 B12 Tricycles Bicycles


Beg inv. July 1, year one 3,500 1,200 800 2,150
End inv. June 30, year two 2,000 1,800 1,000 900
Unit cost/ price $1.20 $4.50 $54.50 $89.60
Projected fiscal year units sales -- -- 96,000 130,000
Components usage

Tricycles 2/units 1/unit


Bicycles 2/units 4/units

Budgeted production quantity budget


Description Tricycles Bicycles
Projected sales 96,000 130,000
Less Beg – inv. (800) (2,150)
Add End – inv. 1,000 900
Q of budgeted production 96,200 128,750

Raw materials requirements budget


Description A 19 B12
Tricycles requirements (2,1) 96,200 × 2 = 192,400 96,200 ×1 = 96,200
Bicycles requirements (2,4) 128,750 × 2 = 257,500 128,750 ×4 = 515,000
Production requirements of raw materials 449,900 units 611,200 units
Less Beg – inventory of raw materials (3,500) (1,200)
Add End– inventory of raw materials 2,000 1,800
Estimated purchases of raw materials – units 448,400 611,800
Estimated purchase price per unit of raw materials $ 1.20 $4.50
Budgeted cost of raw materials purchases $538,080 $2,753,100
Budgeted cost of raw materials purchases $3,291,180

From previous fact pattern; If the economic order quantity of component B12 is 70,000 units, the
number of times that Wellfleet Company should purchase this component during the fiscal year
611,800 .units(annual. purchases)
ended June 30,Year two = = 9 times
70 ,000 .units. per.order
Using the information from Exhibit A, what is budgeted COGM for the three-month period of
January, February, and March?
A. $2,184,525 B. $2,194,500 C. $2,183,100 D. $2,208,750
Answer (D) is correct. The number of units to be sold in this three-month period is 15,400 units,
calculated as the sum of the sales for the period (5,000 units + 5,100 units + 5,300 units).
To determine the number of units to be produced, adjust the number of units to be sold (15,400) by
adding desired ending finished goods inventory for March of 600 (6,000 units × 10%) and
subtracting beginning finished goods inventory for January of 500 (5,000 units × 10%), leaving
15,500 units to be produced in the three-month period.

Budgeted cost of goods manufactured is 15,500 × $142.50 = $2,208,750


See Gleim's MCQ's # 150, 152, 156, 157, 159, 167, 170, 171, 172, 173, 175, 176, 177, 178, 179,
196, 197, 208, 209, 210, 211, 212, 213, 214, 215, 216, 217, 218, 219, 220, 223, 224, 225, 227,

8. Selling and administrative expenses budget

Non-manufacturing expenses are often grouped into a single budget called a selling and
administrative expenses budget or non-manufacturing costs budget. The selling expense components of
this budget include salaries and commissions for the sales department, travel and entertainment,
advertising expenditures, shipping supplies, postage and stationery (related to sales), etc. Sales
expenses are included in this category because they are not allowed to be allocated to production
processes, but must be expensed in the period in which they are incurred. The administrative
expenses components of this budget include management salaries, legal and professional services,
utilities, insurance expense, non-sales related stationery, supplies, postage, etc.
Just as with overhead expenses, selling and administrative expenses can be categorized into fixed costs
and variable costs. In general, selling expenses are made up of both fixed and variable cost components,
whereas the administrative expenses tend to include more fixed costs.
♠The costs in this budget usually satisfy long-term goals, such as customer service, so it is not easy
to make cuts in these expense items.

Q1: For the month of December, Crystal Clear Bottling Expects to sell 12,500 cases of Cranberry
Sparkling Water at $24.80 per case and 33,100 cases of Lemon Dream Cola at $32 per case. Sales
personnel receive 6% commission on each case of Cranberry Sparkling Water and 8% commission on
each case of Lemon Dream Cola. In order to receive a commission on a product, the sales personnel
team must meet the individual product revenue quota. The sales quota for Cranberry Sparkling Water
is $500,000, and the sales quota for Lemon Dream Cola is $1,000,000. How much sales commission
that should be budgeted for December?

Item Commission % Expected Sales ($) Sales quota


CSW 6% 12,500 × $24.80 = $310,000 $500,000
LDC 8% 33,100 × $32 = $1,059,200 $1,000,000

Budgeted commission for December = $1,059,200 × 8% = $84,736

Q2: Karma, Inc. pays out sales commissions to its team in the month the company receives cash for
sales. These commissions equal 5% of total (monthly) cash inflows as a result of sales. Karma has
budgeted sales of $300,000 for August, $400,000 for September, and $200,000 for October.
Approximately half of all sales are on credit, and the other half is for cash sales. Experience indicates
that 70% of the budgeted credit sales will be collected in the month following the salts, 20% the month
after that, and 10% of the sales will be uncollectible. Based on this information, what should be the
total estimated amount of sales commissions paid out by Karma in the month of October?

Description August September October


Budgeted sales $300,000 $400,000 $200,000
Estimated cash collection same month (50%) $150,000 $200,000 $100,000
Next month collection (50% × 70%) = 35% ? $105,000 $140,000
Third month collection (50% × 20%) = 10% ? ? $30,000
Total estimated collections during month ? ? $270,000
Estimated amount of sales commission (5%) ? ? $13,500

Q3: Tut Company selling and administrative costs for the month of August when it sold 20,000 units
were as follows.
Description Costs Per Unit Total Costs
Variable costs $18.60 $372,000
Step costs 4.25 85,000
Fixed costs 8.80 176,000
Total selling and administrative costs $ 31.65 $ 633,000
The variable costs represent sales commissions paid at the rate of 6.20 % of sales the step costs depend
on the number of sales persons employed by the company. In August there were 17 persons on the sales
force However, two members have taken early retirement effective August, 31 it is anticipated that these
positions will remain vacant for several months. Total fixed costs are unchanged within a relevant range
of 15,000 to 30,000 units per month. Tut is planning a sales price cut of 10%. which it expects will
increase sales volume to 24,000 units per month .If Tot implements the sales price reduction, the total
budgeted selling and administrative costs for the month of September would be:
A. $714960 B. $652,760 C. $679,760 D. $759,600
Answer (B) is correct:
Current sales price =$18.6 / 6.2% = $300 per unit
Projected sales price =$300× 90%= $270per unit
Current sales quantity =$372,000 ÷$18.60 per unit = 20,000 units
Budgeted sales quantity =24,000 units

Description Costs Per Unit Total Costs


Variable costs 24,000 × $270 ×6.2% $16.74 $401,760
Step costs ($85,000 ÷ Person 17) ×15 3.125 75,000
Fixed costs 7.33 176,000
Total selling and administrative costs $27.198 $652,760

Q4: The marketing manager of Ames Company has learned the following about a new product that is
being introduced by Ames. Sales of this product are planned at $100,000 for the first year. Sales
commission expense is budgeted at 8% of sales plus the marketing manager's incentive budgeted at an
additional ½%. The preparation of a product brochure will require 20 hours of marketing salaried staff
time at an average rate of $100 per hour, and 10 hours, at $150 per hour, for an outside illustrator's effort.
The variable marketing cost for this new product will be
A. $8,000 B. $8,500 C. $10,000 D. $10,500
Answer (B) is correct: The variable marketing cost would include the 8% sales commission
plus the ½% manager’s incentive, 8.5% x $100,000 = $8,500.
See Gleim's MCQ's # 161, 166, 169,

9. Cash budget (schedule of cash receipts and disbursements)

♥Cash budget can't be prepared unless all operating budget schedules are completed first.
♥Cash budget must be completed before preparing pro-forma B/S and pro-forma statement
of cash flow
♥Cash budget contains hybrid cash flows data (operating and non-operating cash flows data).
But cash budget is considered financial budget.
♥Cash budget preferably prepared on a monthly basis to avoid opportunity costs of excess
non-invested cash and to minimize the cost of interim financing needs.

Maintaining adequate liquidity is a requirement for staying in business, and a cash budget is a plan to
ensure liquidity. Financing can be arranged in an orderly fashion, and investment durations can be
planned so that investments can be liquidated at the time the funds are needed. Cash budget is
commonly formulated for monthly periods, but many companies find it useful to have even finer
divisions, such as by week or even by day. Because cash is needed in all areas of operations, the cash
budget gets data from all parts of the master budget. A cash budget is divided into four sections: the cash
receipts section, the cash disbursements section, the cash surplus or deficient section, and the
financing section.

Reasons for preparing cash budget


1. To predict the cash balance over the period of the budget.
2. To arrange an appropriate overdraft facility with the bank.
3. To plan uses for any anticipated cash surpluses.
4. To plan the timing of expenditure.

According to John Maynard Keynes, the three major motives for holding cash are
A. Transactional purposes as a medium of exchange
B. Precautionary purposes
C. Speculative purposes

Q1: The primary reason to have a cash budget is to


A. Generates a return on investment. B. Sum up all of the other budgets.
C. Locate sources of financing for long-term projects D. Ensure that liquidity is maintained.

The correct answer is (D). A cash budget is a plan to ensure that liquidity is maintained. While generating a
return on investment is desirable; liquidly takes precedence over return goal.

Q2: When managing cash and short-term investments, a corporate treasurer is primarily concerned
with:
A. Maximizing rate of return. B. Minimizing taxes. C. Liquidity and safety.
D. Investing in treasury bonds since they have no default risk.

Answer (A) is incorrect because most companies are not in business to earn high returns on liquid assets
(i.e., they are held to facilitate operations).
Answer (B) is incorrect because the holding of cash and cash-like assets is not a major factor in
controlling taxes.
Answer (C) is correct. Cash and short-term investments are crucial to a firm’s continuing success.
Sufficient liquidity must be available to meet payments as they come due. Liquidity and safety are the
primary concerns of the treasurer when dealing with highly liquid assets. Cash and short-term
investments are held because of their ability to facilitate routine operations of the company. These assets
are not held for purposes of achieving investment returns.

Answer (D) is incorrect because investments in treasury bonds do not have sufficient liquidity to serve as
short-term assets.

Q3: Which of the following budgets is designed to ensure that the company maintains adequate liquidity?
A. Sales budget B. Capital budget
C. Cash budget D. Production/inventory budget
The correct answer is (C). The purpose of the cash budget is to ensure that enough cash is available to
meet current obligations.

Q4: According to John Maynard Keynes, the three major motives for holding cash are for:
A. Transactional, psychological, and social purposes.
B. Speculative, fiduciary, and transactional purposes.
C. Speculative, social, and precautionary purposes.
D. Transactional, precautionary, and speculative purposes.
Answer (D) is correct. John Maynard Keynes, founder of Keynesian economics, concluded that there
were three major motives for holding cash: for transactional purposes as a medium of exchange,
precautionary purposes, and speculative purposes (but only during deflationary periods).
Q5: The cash budget must be prepared before completing the
A. Capital expenditure budget. B. Sales budget.
C. Forecasted balance sheet. D. Production budget.
Answer (A) is incorrect because the capital expenditure budget is an input necessary for the preparation
of a cash budget.
Answer (B) is incorrect because the sales budget is usually the first budget prepared.
Answer (C) is correct. The pro forma balance sheet is the balance sheet for the beginning of the period
updated for projected changes in cash, receivables, inventories, payables, etc. Accordingly, it can't be
prepared until after the cash budget is completed because cash is a current asset reported on the balance
sheet.
Answer (D) is incorrect because a production budget is normally prepared before the cash budget is
started.

Q6: The cash budget must be prepared before you can complete the
A. Production budget. B. Forecasted balance sheet
C. Forecasted income statement. D. Capital expenditure budget
the correct answer is (B): The cash budget determines the projected borrowings, repayments, investments,
interest received and interest paid. All of this information is needed to forecast the balance sheet.

Q7: The cash receipts budget includes


A. Funded depreciation. B. Operating supplies.
C. Extinguishment of debt. D. Loan proceeds.
Answer (A) is incorrect because funded depreciation involves cash outlays.
Answer (B) is incorrect because purchases of supplies involve cash outlays.
Answer (C) is incorrect because the extinguishment of debt involves cash outlays.
Answer (D) is correct. A cash budget may be prepared monthly or even weekly to facilitate cash
planning and control. The purpose is to anticipate cash needs while minimizing the amount of idle cash.
The cash receipts section of the budget includes all sources of cash. One such source is the proceeds of
loans.

Q8: Which one of the following items would have to be included for a company preparing a schedule of
cash receipts and disbursements for Calendar Year one?
A. Purchase order issued in December year one for items to be delivered in February year two.
B. Dividends declared in November year one to be paid in January year two to shareholders of record as
of December year one
C. The amount of uncollectible customer accounts for year one
D. The borrowing of funds from a bank on a note payable taken out in June year one with an agreement
to pay the principal and interest in June year 2
Answer (A) is incorrect because the cash disbursement presumably will not occur until year two.
Answer (B) is incorrect because the cash flow will not occur until dividends are paid in year two.
Answer (C) is incorrect because bad debt expense is a non-cash item.
Answer (D) is correct. A schedule of cash receipts and disbursements (cash budget) should include
all cash inflows and outflows during the period without regard to the accrual accounting treatment of the
transactions. Hence, it should include all checks written and all sources of cash, including borrowings. A
borrowing from a bank in June year one should appear as a cash receipt for year one.

Q9: Myers Company uses a calendar-year and prepares a cash budget for each month of the year.
Which one of the following items should be considered when developing July’s cash budget?
A. Federal income tax and social security tax withheld from employee’s June paychecks to be remitted
to the Internal Revenue Service in July.
B. Quarterly cash dividends scheduled to be declared on July 15 and paid on August 6 to shareholders
of record as of July 25.
C. Property taxes levied in the last calendar year scheduled to be paid quarterly in the coming year
during the last month of each calendar quarter.
D. Recognition that 0.50% of the July sales on account will be uncollectible.
Answer (A) is correct:

Q10: Which of the following would not appear in the cash disbursements section of a cash budget?
A. Payments to supplier's B. Interest expense
C. Depreciation expense D. Salaries and wages expenses
Answer (C) is correct: Depreciation is a non-cash entry and would not appear on a cash budget.

Q11: Which one of the following is least likely to be a factor in determining medium- and long-term
cash forecasts?
A. Forecasts for production fluctuations
B. Current union wage negotiations
C. Potential governmental tax law changes
D. Increases in cost of natural resources
Answer (C) is correct: When determining medium- and long-term cash flows, it is important to consider
changes in any factors that can be reasonably forecast. Changes in production (fixed and variable costs),
wage expenses, and increases in the cost of natural resources can typically be predicted with varying
degrees of accuracy. Since potential governmental tax law changes can't be known until signed into law, it
would be difficult for a business to accurately anticipate these changes.

Q12: Which one of the following best represents a factor that should be considered for medium and
long-term cash forecasting
A. Pre-tax cost of capital projects. B. Current monthly depreciation.
C. Impact of stock split. D. Non-routine property sales.
Answer (D) is correct:

Cash inflow and A/R balance

Q1: Brown Company estimates that monthly sales will be as follows.

Month January February March


Sales forecast $100,000 150,000 180,000

Historical trends indicate that 40% of sales are collected during the month of sale, 50% are collected in
the month following the sale, and 10% are collected two months after the sale. Brown’s A/R balance
as of December 31 totals $80,000 ($72,000 from December’s sales and $8,000 from November’s
sales). The amount of cash Brown can expect to collect during the month of January is
A. $76,800 B. $84,000 C. $108,000 D. $133,000
Correct answer is (C) The amount of cash expected to be collected during the month of January =
8,000 from November sales + [(72,000 /0.60) × 0.50] + (100,000 × 40%) = $108,000
A/R balance at the end of January = (100,000 ×60%) + [(72,000 /0.60) × 0.10] = $72,000

Q2: Granite Company sells products exclusively on account, and has experienced the following collection
pattern. 60% in the month of sale, 25% in the month after sale, and 15% in the second month after sale,
Uncollectible accounts are negligible. Customers who pay in the month of sale are given a 2% discount.
If sales are $220,000 in January, $200,000 in February, $280,000 in March, and $260,000 in April,
Granite’s A/R balance on May 1 will be:
A. $107,120 B. $143,920 C. $146,000 D. $204,000
Correct answer is (C): A/R balance on May 1 = (260,000 × 40%) + (280,000 ×15%) = $146,000.

Q3: Bootstrap Corporation anticipates the following sales during the last six months of the year.

Month July August September October November December


Sales $ 460,000 500,000 525,000 500,000 480,000 450,000

20% of Bootstrap’s sales are for cash. The balance is subject to the collection pattern shown below.
Percentage of balance collected in the month of sale 40%
Percentage of balance collected in the month following sale 30%
Percentage of balance collected in the second month following sale 25%
Percentage of balance uncollectible 5%
What is the planned net A/R balance as of December 31?
A. $279,300 B. $294,000 C. $360,000 D. $367,500
Correct answer is (B): Planned net A/R balance as of December 31 can be determined as follows:

From December sales (450,000 × 44%) + From November sales (480,000 ×20%) = 294,000.

Q4: Projected monthly sales of Wallstead Corporation for January to April are as follows.

Month January February March April


Sales $ 300,000 340,000 370,000 390,000

• The company bills each month’s sales on the last day of the month.
• Receivables are booked gross and credit terms of sale are: 2/10, N130.
• 50% of the billings are collected within the discount period, 30% are collected by the end of the month,
15% are collected by the end of the second month, and 5% become uncollectible.
Budgeted cash collections for Wallstead Company during April would be
A. $343,300 B. $347,000 C. $349,300 D. $353,000

Correct answer is (A): Cash collections during April can be determined as follows:
Collections from March sales (370,000 × 79%) + Collections from February sales (340,000× 15%) =
$343,300.

Q5:Tip-Top Cleaning Supply carries a large number of different items in its inventory, giving the firm a
competitive advantage in its industry. Below is part of Tip-Top’s budget for the first quarter of next year.
Sales $855,000
Cost of goods sold 425,000
Rent and salary expenses 375,000
historically, all of the sales are on account and are made evenly over the quarter. 5% of all sales are
determined to be uncollectible and written off. The balance of the receivables is collected in 50 days. This
sales and collection experience is expected to continue in the first quarter. The projected balance sheet for
the first day of the quarter includes the following account balances.
Cash $10,000
Accounts receivable (net) 450,000
Inventory 900,000
Accounts payable 800,000
how much cash can Tip-Top anticipate collecting in the first quarter (based on a 360-day year)?
A. $811,000 B. $830,000 C. $901,250 D. $902,500
Correct answer is (A): Anticipated cash collections during the quarter can be determined as follows:
Daily sales = $855,000 ÷ 90 days = $9,500
Sales collections = A/R (net) 450,000 + (40 days × $9,500 ×95%) = $811,000

Net A/R balance at the end of first quarter = 50 days × $9,500 ×95% = $451,250

Q6: Prudent Corporation’s budget for the upcoming accounting period reveals total sales of $700,000 in
April and $750,000 in May. The sales cash collection pattern is
20% of each month’s sales are cash sales.
5% of a month’s credit sales are uncollectible.
70% of a month’s credit sales are collected in the month of sale.
25% of a month’s credit sales are collected in the month following the sale.
If Prudent anticipates the cash sale of a piece of old equipment in May for $25,000, May’s total budgeted
cash receipts would be?
A. $560,000 B. $702,500 C. $735,000 D. $737,500
Correct answer is (C): Total budgeted cash receipts during May = $25,000 + (700,000 × 20%) +
(750,000 × 76%) = $735,000

Q7: ANNCO sells products on account, and experiences the following collection schedule.
In the month of sale 10%
in the month after sale 60%
in the second month after sale 30%
At December 31, ANNCO reports A/R of $211,500. Of that amount, $162,000 is due from December
sales, and $49,500 from November sales. ANNCO is budgeting $170,000 of sales for January. If so, what
amount of cash should be collected in January?
A. $129,050 B. $174,500 C. $211,500 D. $228,500
Correct answer is (B): Total budgeted cash receipts during January = ($170,000 × 10%) + (49,500) +
(162,000 /0.90 × 60%) = $174,500

Q8: Trumbull Company budgeted sales on account of $120,000 for July, $211,000 for August, and
$198,000 for September. Collection experience indicates that 60% of the budgeted sales will be collected
the month after the sale, 36% the second month, and 4% will be uncollectible. The cash from A/R that
should be budgeted for September would be
A. $169,800 B. $194,760 C. $197,880 D. $198,600
Answer (A) is correct: ($211,000× 60%) + ($120,000× 36%) = $169,800

Q9: Werner Company buys raw materials from several suppliers, and makes payments according to the
following schedule.
In the month of purchase 25%
In the month after purchase 60%
In the second month after purchase 15%
In preparing the master budget for the fourth quarter of the year, Werner assumed that total purchases will
be sufficient to have enough inventories on hand to fill one month’s worth of production, as has been the
policy for the past several years. The production is forecast as follows, in $000:

September October November December January


170 180 190 220 230

What will be the A/P balance as of December 31?


A. $260,500 B. $230,000 C. $220,000 D. $205,500
Answer (D) is correct:

Description September October November December


Purchases 180 190 220 230
A/P balance
75% from December 172.50
15% from November 33
Total A/P balance 205.50

Q10 (H.W): Health Foods Inc. has decided to start a cash budgeting program to improve overall
cash management. Information gathered from the past year reveals the following cash collection
trends.
40% of sales are on credit
50% of credit sales are collected in month of sale
30% of credit sales are collected first month after sale
15% of credit sales are collected second month after sale
5% of credit sales result in bad debts
Gross sales for the last five months were as follows.

Month January February March April May


Sales $220,000 240,000 250,000 230,000 260,000

Sales for June are projected to be $255,000. Based on this information, the expected cash
receipts for March would be
A. $230,000 B. $237,400 C. $242,000 D. $243,200
Correct answer is (C): Expected cash receipts for March = (250,000×80%) + (240,000
×12%) + (220,000×6%) = $242,000

Q11(H.W): Data regarding Johnsen Inc.’s forecasted dollar sales for the last seven months of the
year and Johnsen’s projected collection patterns are as follows.

Month June July August September October November December


Forecasted sales 700,000 600,000 650,000 800,000 850,000 900,000 840,000

Types of sales
Cash sales 30%
Credit sales 70%
Collection pattern on credit sales
5% determined to be uncollectible
During the month of sale 20%
During the first month following the sale 50%
During the second month following the sale 25%
Johnsen’s budgeted cash receipts from sales and collections on account for September are
A. $635,000 B. $684,500 C. $807,000 D. $827,000
Correct answer is (B): Budgeted cash receipts from sales and collections on account for
September = (800,000×30%) + (800,000 ×14%) + (650,000 ×35%) + (600,000 ×17.5%) =
$684,500

Q12: Historically, pine hill wood products have had no significant bad debt experience with its
customers. Cash sales have accounted for 10% of total sales, and payments for credit sales have been
received as follows:
40% of credit sales in the month of sales
30% of credit sales in the first subsequent month
25% of credit sales in the second subsequent month
5% of credit sales in the third subsequent month
The forecast for both cash and credit sales is as follows:
Description Jan Feb March April May
Forecasted total sales 95,000 65,000 70,000 80,000 85,000

Calculate cash collections during April and May and ending A/R balance at the end of April and May

Description Jan Feb March April May


Forecasted total sales 95,000 65,000 70,000 80,000 85,000
1. Forecasted credit sales (90% of total sales) 85,500 58,500 63,000 72,000 76,500
2. Forecasted cash sales (10% of total sales) 9,500 6,500 7,000 8,000 8,500
Collections
Cash sales (100%) same month 9,500 6,500 7,000 8,000 8,500
Credit sales Month of sales (40%) 34,200 23,400 25,200 28,800 30,600
Credit sales First next month (30%) ? 25,650 17,550 18,900 21,600
Credit sales Second next month (25%) ? ? 21,375 14,625 15,750
Credit sales Third next month (5%) ? ? ? 4,275 2,925
Total cash collections ? ? ? 74,600 79,375

Balance of A/R at end of April and May

Description April May


Same month uncollected sales [(30 % + 25% + 5%)] × 90% = 54% of total 43,200 45,900
Previous month sales [(25% + 5%)] × 90% = 27% of total 18,900 21,600
Two previous months sales [5% ] × 90% = 4.5 % of total 2,925 3,150
Balance of A/R at end of April and May 65,025 70,650

Q13: From previous example due to deteriorating economic conditions, pine Hill wood products has
now decided that its cash forecast should include a bad debt adjustment of 2% of credit sales, beginning
with sales for the month of April. Because of this policy change, the total expected cash inflow
related to sales made in April will.
A. Be unchanged. B. Decrease by $1,260 C. Decrease by $1,440 D. Decrease by $1,530
Correct answer is (C):
Uncollectible bade debt % = 90% × 2% = 1.80%
Uncollectible amount of April sales = $80,000 × 1.80% = $1,440
Q14: From previous example the estimated total cash collections during April and May from sales made
on open account (credit sales) on that period assuming no bad debt $28,800 + 30,600 + 21,600 =
$81,000

Q15: A project with a 4 year life has a cost of acquisition of $400,000 and installation cost of $100,000.
If the effective income tax rate is 40%, what is the cash saving each period due to depreciation expense?
A. $40,000 B. $50,000 C. $60,000 D. $75,000
Answer (B) is correct: Annual reduction of cash outflow in tax expense due to depreciation
expense= [($400 + $100) ÷ 4 years] × depreciation tax shield 40% = $50

Q16: Edberg Company has developed the following sales projections for the months from May to
October
May $100,000 August $160,000
June 120,000 September 150,000
July 140,000 October 130,000

Normal cash collection experience has been that 50% of sales are collected during the month of sales
and 45% in the month following sales. The remaining 5% of sales is never collected. Calculate Edberg’s
budgeted cash collections for the third calendar quarter
Description May June July Aug. Sep.
Projected sales 100,000 120,000 140,000 160,000 150,000
50% during same month 70,000 80,000 75,000
45% one month later 54,000 63,000 72,000
Total collections during the month 124,000 143,000 147,000
Total budgeted collections during the 3rd calendar $414,000
quarter

Q17: The Matthew Nichols Company projected sales of $200,000 for July, $280,000 for August,
$198,000 for September and $200,000 for October. Approximately 75% of sales are on credit; the
remainder is cash sales. Collection experience indicates that 60% of the budgeted credit sales will be
collected the month after the sales, 36% will be collected the second month, and 4% will be uncollectible.
Calculate the cash receipts budgeted for September and October.

Description July Aug. Sep. Oct.


Projected sales 200,000 280,000 198,000 200,000
Collections
1. Collections from cash sales (25%) 50,000 70,000 49,500 50,000
2. Collections from credit sales (75%)
One Month after sales 75% × 60% = 45% ------- 90,000 126,000 89,100
Two months 75% × 36% = 27% ------- ------- 54,000 75,600
Total collections ------- ------- 229,500 214,700
Q18: The Maxwell Company’s cash budget for March includes the following information
concerning its accounts receivable:

Estimated credit sales for March $300,000


Actual credit sales for February 250,000
Estimated collections in March for credit sales in March 30%
Estimated collections in March for credit sales in February 60%
Estimated collections in March for credit sales prior to February 15,000
Estimated write-offs in March for uncollectible credit sales 7,000
Estimated provision for bad debts in March, for credit sales in March 8,000
Calculate estimated cash receipts during March

Description February March (estimated)


Sales (S) 250,000 $300,000
Estimated collections during March for credit sales in March $300,000 × 30% 90,000
Estimated collections during March for credit sales in February $250,000 × 60% 150,000
Estimated collections in March for credit sales prior to February 15,000
Estimated cash receipts during march $255,000

Q19: Cooper Company’s management team is preparing a cash budget for the coming quarter. The
following budgeted information is under review.

Description January February March


Revenue $700,000 $800,000 $500,000
Inventory purchases 350,000 425,000 225,000
Other expenses 150,000 175,000 175,000

The company expects to collect 40% of its monthly sales in the month of sale and 60% in the following
month. 50% of inventory purchases are paid in the month of purchase, and the other 50% in the following
month. All payments for other expenses are made in the month incurred.
Cooper forecasts the following account balances at the beginning of the quarter.
Cash $100,000
Accounts receivable 300,000
Accounts payable (Inventory) 500,000
given the above information, the projected change in cash during the coming quarter will be
A. $412,500 B. $300,000 C. $112,500 D. $133,000

Description January February March


Cash collections from December sales 300,000
Cash collections from January sales 280,000 420,000
Cash collections from February sales 320,000 480,000
Cash collections from March sales 200,000
Total cash collections 580,000 740,000 680,000
Cash payments on December purchases 500,000
Cash payments on January purchases 175,000 175,000
Cash payments on February purchases 212,500 212,500
Cash payments on March purchases 112,500
Other expenses 150,000 175,000 175,000
Total cash payments 825,000 562,500 500,000
Change in Cash during the month (245,000) 177,500 180,000
Beginning Cash balance $100,000 (145,000) 32,500
Ending Cash balance (145,000) 32,500 212,500

Projected change in cash during the coming quarter = Ending Cash balance 212,500 - Beginning Cash
balance $100,000 =112,500

Answer (C) is correct:

Q20: From the previous example assume the account balances at the beginning of the quarter
are as follows.
Cash $200,000
Accounts receivable 300,000
Accounts payable (Inventory) 400,000
given the above information, the projected ending cash balance for February will be
A. $712,500 B. $500,000 C. $232,500 D. $120,000

Description January February March


Collections 300 +280 = 580,000 (700 × 60%) (800×60%) +
+(800×40%) = 740,000 (500×40%) = 680,000
Purchases Payments 400 +175 = 175 + 212,5 = 212,5 +112,5 =
575,000 387,500 325,000
Other expenses 150,000 175,000 175,000
Net changes in cash (145,000) 177,500 180,000
Beginning cash balance 200,000 55,000 232,500
Ending cash balance 55,000 232,500 412,500

Answer (C) is correct:

Q21: As part of the master budget process, a merchandising company begins to prepare the cash budget
for the same period. Which of the following additional information will be most useful to management in
preparing this budget?
A. Sales, credit policies, purchasing terms, and planned capital acquisition.
B. Projected revenues, projected expenses, and intended financing activities.
C. Credit policies, projected expenses, and inventory procurement policies.
D. Planned D.M purchases, planned D.L, and purchasing terms.
Answer (A) is correct

Q22: Holland Company is in the process of projecting its cash position at the end of the second quarter.
Shown below is pertinent information from Holland’s records.
Cash balance at end of 1st quarter $36,000
Cash collections from customers for 2nd quarter 1,300,000
Accounts payable at end of 1st quarter 100,000
Accounts payable at end of 2nd quarter 75,000
All 2nd quarter costs and expenses (accrual basis) 1,200,000
Depreciation (accrued expense included above) 60,000
Purchases of equipment (for cash) 50,000
Gain on sale of asset (for cash) 5,000
Net book value of asset sold 35,000
Repayment of notes payable 66,000
From the data above, determine Holland’s projected cash balance at the end of the second quarter.
A. Zero B. $25,000 C. $60,000 D. $95,000
Answer (D) is correct:
cash available during the second quarter = Beginning cash balance $36,000 + Cash collections
1,300,000 + Cash received from sale of asset 40,000 = $1,376,000.

Cash payments during the second quarter = (Costs and expenses $1,200,000 – Depreciation $60,000) +
Cash purchase of equipment $50,000 + Repayment of notes payable $66,000 + decrease in A/P balance
$25,000 = $1,281,000
Cash balance at the end of the second quarter = $1,376,000 - $1,281,000 = $95,000

Q23: Monroe Products is preparing a cash forecast based on the following information.
• Monthly sales: December $200,000; January $200,000; February $350,000; March $400,000.
• All sales are on credit and collected the month following the sale.
• Purchases are 60% of next month’s sales and are paid for in the month of purchase.
• Other monthly expenses are $25,000, including $5,000 of depreciation.
If the January beginning cash balance is $30,000, and Monroe is required to maintain a minimum cash
balance of $10,000, how much short-term borrowings balance at the end of February?
A. $60,000 B. $70,000 C. $75,000 D. $80,000

Description January February March


Monthly sales- accrual 200,000 350,000 400,000
Collections 200,000 200,000 350,000
Payment for purchases 210,000 240,000
Other monthly cash expenses payments 20,000 20,000 20,000
Change in Cash during the month (30,000) (60,000)
Beginning Cash balance 30,000 10,000
Ending Cash balance 0 (50,000)
Minimum cash balance (borrowing) 10,000 60,000
Adjusted Ending Cash balance 10,000 10,000

Short-term borrowing will be required at the end of February =10,000 during January +60,000 during
February = $70,000
Answer (B) is correct:

Q24: Tidwell Corporation sells a single product for $20 per unit. All sales are on account, with 60% collected
in the month of sale and 40% collected in the following month. A schedule of cash collections for January
through March of the coming year reveals the following receipts for the period.

Description Cash receipts


January February March
December receivables $32,000
From January sales 54,000 $36,000
From February sales 66,000 $44,000
From March sales 72,000

Other information includes the following.


• Inventories are maintained at 30% of the following month’s sales.
• Tidwell desires to keep minimum cash balance of $15,000. Total payments in January are expected to be
$106,500, which excludes $12,000 of depreciation expense. Any required borrowings are in multiples of
$1,000.
• The December 31 balance sheet for the preceding year revealed a cash balance of $24,900.
Ignoring income taxes, the financing needed in January to maintain the firm’s minimum cash balance is
A. $8,000 B. $10,600 C. $11,000 D. $23,000

Description Amount
January collections ($32,000 +54,000) 86,000
Cash payments in January (106,500)
Beginning cash balance 24,900
Ending cash balance 4,400
minimum cash balance 15,000
Required financing in January (in multiples of $1,000) 11,000
Ending cash balance in January 15,400

Answer (C) is correct:

Q25: The Mountain Mule Glove Company is in its first year of business. Mountain Mule had a beginning cash
balance of $85,000 for the quarter. The company has a $50,000 short- term line of credit. The budgeted information
for the first quarter is shown below.

Description January February March


Sales $60,000 $40,000 $50,000
Purchases 35,000 40,000 75,000
Operating costs 25,000 25,000 25,000

All sales are made on credit and are collected in the second month following the sale. Purchases are paid in the
month following the purchase, while operating costs are paid in the month that they are incurred. How much will
Mountain Mule need to borrow at the end of the quarter if the company needs to maintain a minimum cash balance
of $5,000 as required by a loan covenant agreement?
A. $0 B. $5,000 C. $10,000 D. $45,000
Answer (C) is correct:

Description January February March Total


Beginning cash balance 85,000
Collections $60,000 $60,000
Purchases payments 35,000 40,000 75,000
Operating costs 25,000 25,000 25,000 75,000
Ending cash balance (5,000)
minimum cash balance $5,000
Amount needed to be borrowed 10,000

Q26: Arrow Co. master budget was prepared based on the following projections.
Sales $2,400,000
Gross margin % 40%
Decrease in inventories $60,000
Decrease in A/P $100,000
A. What is COGS?
$2,400,000 × 60% = $1,440,000
B. What is the purchases amount during the period?
$1,440,000 – $60,000 = $1,380,000
C. What are estimated cash disbursements during the period?
1,380,000 + $100,000 = $1,480,000

Q27: A company has $10,000 in cash and $150,000 in merchandise inventory on March 31. The desired cash
and merchandise inventory balances on June 30 are $20,000 and $250,000 respectively. Sales for the quarter
are expected to be $300,000, all in cash. Gross margin is 40% of sales. Cash operating expenses are expected
to be $50,000. All merchandise inventory purchases are paid for in cash at the time of purchase. What
amount of financing will the company need during the quarter?
Beginning inventory + purchases = COGS + Ending inventor
$150,000 + purchases = ($300,000 × 60%) + $250,000
Purchases = $280,000

DR. Cash account CR.


Beginning Balance 10,000 Cash expenses 50,000
Cash sales 300,000 Purchases 280,000
Required financing 40,000 Desired Ending Balance 20,000
Total $350,000 Total $350,000

Q28: a company is preparing its cash budget for the coming month. All sales are made on account.
Given the following, Calculate Expected cash balance at the end of the coming month.

Description Beaming balance Budgeted amount


Cash $50,000
Accounts receivable 180,000
Sales $800,000
Cash disbursement 780,000
Depreciation 25,000
Ending accounts receivable balance 210,000

Answer

DR. Accounts receivable account CR.


Beginning balance 180,000 Collections 770,000
Sales 800,000 Ending balance 210,000
Total 980,000 Total 980,000

Dr. Cash account Cr.


Beginning balance 50,000 Cash disbursement 780,000
Cash collections 770,000 Ending balance 40,000
Total $ 820,000 Total $ 820,000

Q29: The following information concerns Montero Corporation.


Description Purchases Sales
January $42,000 $72,000
February 48,000 66,000
March 36,000 60,000
April 54,000 78,000
May 60,000 66,000

Collections from Montero Corporation customers are normally 70% in the month of sale, and 20% and
9% respectively, in the 2 months following the sale. The balance is uncollectible. Montero takes full
advantage of the 2% discount allowed on purchases paid for by the 10th of the following month. Cash
disbursements for expenses are expected to be $14,400 for the month of May Montero’s cash balance at
May, 1, was $22,000
What was cash balance on April, 1, assuming cash disbursements for expenses increased 20%
from April to May what is April beginning cash balance?

Description January February March April May


Sales 72,000 66,000 60,000 78,000 66,000
Purchases 42,000 48,000 36,000 54,000 60,000
Collections
A) Same month 70% 50,400 46,200 42,000 54,600 46,200
B) Next month 20% ? 14,400 13,200 12,000 15,600
C) third month 9% ? ? 6,480 5,940 5,400
Total collections ? ? 61,680 72,540 67,200
Payment on purchases 98% ? 41,160 47,040 35,280 52,920
Other payments 12,000 14,400
Total disbursements 47,280 67,320
Net cash out flows during May (120)
Beg. Cash balance 22,000
Ending cash balance 21,880

Dr. Cash account - April Cr.


Begging Balance ? Total Payments 47,280
Collection during April 72,540 Ending balance 22,000
Total 72,540 Total 72,540

Beg. Cash balance is overdraft of ($3,260)


Q30: Ward Corporation’s current year-end sales totaled $240 million and its ending cash balance was $20
million. Ward anticipates its sales for the upcoming year will be $260 million. On average, 10% of a year’s
sales will be collected during the following year. Assume Ward has no uncollectable accounts. Ward also
anticipates cash expenses of $240 million and depreciation of $5 million. During the next year, Ward
intends to spend $30 million cash for capital improvements. If Ward’s policy is to have a minimum of $10
million cash available at the beginning of each year, its budgeted cash flow projections indicate that it will
need outside financing of
A. $0 B. $2 million C. $7 million D. $26 million

Answer (B) is correct:

Description Amount Notes


Beg cash balance $20
Beg A/R balance (collected) 24 240 × 10%
Amounts collected for year's sales 234 260 × 90%
Cash expenses (240)
Cash for capital improvements (30)
Pre Adjusted ending cash balance 8
Minimum cash balance 10
Outside financing 2 (8-10)
Q31: Werner Company buys raw materials from several suppliers, and makes payments according to the
following schedule.
In the month of purchase 25%
In the month after purchase 60%
In the second month after purchase 15%
In preparing the master budget for the fourth quarter of the year, Werner assumed that total purchases will be
sufficient to have enough inventories on hand to fill one month’s worth of production, as has been the policy
for the past several years. The production is forecast as follows, in $000:

September October November December January


170 180 190 220 230

What will be the A/P balance as of December 31?


A. $260,500 B. $230,000 C. $220,000 D. $205,500
Answer (D) is correct:

Description September October November December


Purchases 180 190 220 230
A/P balance
75% from December 172.50
15% from November 33
Total A/P balance 205.50

Q32: Trumbull Company budgets sales on account of $120,000 for July, $211,000 for August, and $198,000
for September. Collection experience indicates that 60% of the budgeted sales will be collected the month after
the sale, 36% the second month, and 4% will be uncollectible. The cash from A/R that should be budgeted for
September would be
A. $169,800 B. $194,760 C. $197,880 D. $198,600
Answer (A) is correct: ($211,000× 60%) + ($120,000× 36%) = $169,800

Fact pattern (questions from 33 through 35): Information pertaining to Noskey Corporation's sales
revenue is presented in the following table.
Description November Year 1 December Year 1 January Year 2
( Actual ) (Budget ) ( Budget )
Cash sales $80,000 $100,000 $60,000
Credit sales 240,000 360,000 180,000
Total sales $320,000 $460,000 $240,000

Management estimates that 5% of credit sales are uncollectible. Of the credit sales that are collectible,
60% are collected in the month of sale and the remainder in the month following the sale. Purchases of
inventory are equal to next month's sale needs and gross profit margin is 30%. All purchases of
inventory are on account; 25% are paid in the month of purchase, and the remainder is paid in the month
following the purchase.
Q33: Noskey Corporation's budgeted cash collections in December Year 1 from November Year 1
credit sales are
A. $91,200 B. $228,000 C. $136,800 D. $84,000
Answer (A) is correct: November credit sales are $240,000. However, 5% of this total is not collectible.
Therefore, the total November credit sales that will be collectible are only $228,000 ($240,000 × 95%). Of
this amount, 40% of the collectible November credit sales will be collected in December. This is $91,200
($228,000 × 40%).

Q34: Noskey Corporation's budgeted total cash receipts in January Year 2 are
A. $294,000 B. $239,400 C. $240,000 D. $299,400
Answer (D) is correct: $60,000 + ($180,000 × 95% × 60%) + ($360,000 × 95% × 40%) = $299,400

Q35: What is the total cash payment in December, year 1 for inventory purchases?
Description November Year 1 December Year 1 January Year 2
( Actual ) (Budget ) ( Budget )
Total sales $320,000 $460,000 $240,000
COGS (70%) 224,000 322,000 168,000
Purchases 322,000 168,000
payments
Same month (25%) 80,500 42,000
Second month (75%) ? 241,500
Total payments ? 283,500

Q36: The following information applies to bravo Company:


Month January February March April May June
Purchases $160,000 $160,000 $160,000 $140,000 $140,000 $120,000
Credit sales 100,000 200,000 240,000 300,000 260,000 240,000
Cash collection is 60% in the month of sale, 20% next month and 15% third month the remainder is
uncollectible, cash sales is always expected to be 25% of credit sales also credit terms is 2% cash
discount if payment occurred at the month of sale. Assume also November Total sales were $280,000
and December cash sales were $60,000
Beginning cash balance is $50,000 which represents minimum safety cash
A cash payment equal to 40% of purchases is made at the time of purchase, and 35% is paid in the next
month Purchases for the previous November and December were $150,000 per month. Payroll is 10% of
total sales in the month it occurs, and operating expenses are 20% of the following month's credit sales,
Dep is one fourth (July credit sales were $220,000). Property taxes payments estimated to be $19,500
and $20,000 paid quarterly in January and April respectively. Also assume that any excess cash is
invested at 0.5% monthly and any cash deficit is borrowed at 1% monthly, the interest is to be collected
or paid at the end of the Month. Bravo’s cash budget for the first half of the year would be as follows

Cash budget for the next six months

Description Nov Dec Jan Feb March April May June


Credit sales 224,000 240,000 100,000 200,000 240,000 300,000 260,000 240,000
Cash sales 56,000 60,000 25,000 50,000 60,000 75,000 65,000 60,000
Sales Collections
Cash sales 56,000 60,000 25,000 50,000 60,000 75,000 65,000 60,000
Same month (60% × 131,712 141,120 58,800 117,600 141,120 176,400 152,880 141,120
98%) = 58.8%
Second month (20%) ? 44,800 48,000 20,000 40,000 48,000 60,000 52,000
Third month (15%) ? ? 33,600 36,000 15,000 30,000 36,000 45,000
Total inflow ? ? 165,400 223,600 256,120 329,400 313,880 298,120

Purchases 150,000 150,000 160,000 160,000 160,000 140,000 140,000 120,000


Purchases payments
40% same month 60,000 60,000 64,000 64,000 64,000 56,000 56,000 48,000
35% second month ? 52,500 52,500 56,000 56,000 56,000 49,000 49,000
25% third month ? ? 37,500 37,500 40,000 40,000 40,000 35,000
Total ? ? 154,000 157,500 160,000 152,000 145,000 132,000
Payroll armaments 28,000 30,000 12,500 25,000 30,000 37,500 32,500 30,000
operating expenses 36,000 15,000 30,000 36,000 45,000 39,000 36,000 33,000
Property taxes - - 19,500 - - 20,000 - -
Total cash outflow ? ? 216,000 218,500 235,000 248,500 213,500 195,000

Beg cash balance $50,000 50,000 50,000 50,000 50,000 50,000


Total inflow ? ? 165,400 223,600 256,120 329,400 313,880 298,120
Total cash available 215,400 273,600 306,120 379,400 363,880 348,120
Total cash outflow ? ? 216,000 218,500 235,000 248,500 213,500 195,000
Interest exp./income - (506) (460) (254) 277 780
Net (600) 54,594 70,660 130,646 150,657 153,900
Loans/repayments 50,600 4,594 20,660 80,646 100,657 103,900
Ending cash balance 50,000 50,000 50,000 50,000 50,000 50,000
Loan balance 50,600 46,006 25,346 55,300 155,957 259,857
CR CR CR DR DR DR

Notes

1. Cash discount (2%) is a reduction of cash inflows on the 60% of credit sales.

2. Bad debt expense (5%) is not collected (non-cash expense).

Fact pattern (questions from 37 through 40): super drive. A computer disk storage and back up
company, uses accrual accounting. The company’s statement of financial position for the year ended
November 30 is as follows:
Super drive, Statement of Financial position, November 30
Assets Amount
Cash $52,000
Accounts receivable, net 150,000
Inventory 315,000
Property plant and equipment 1,000,000
Total assets $1,517,000
Liabilities
Accounts payable ($17,880 other payables) $175,000
Common stock (C/S) 900,000
Retained earnings (R.E) 442,000
Total liabilities and shareholders equity $1,517,000

Additional information regarding super drive’s operations includes the following:


♠Sales are budgeted at $520,000 for December and $500,000 for January of the next year.
♠Collections are expected to be 60% in the month of sale and 40% in the month following the sale.
♠80% of the disk drive components are purchased in the month prior to the month of sale, and 20%
are purchased in the month of sale. Purchased components are 40% of the cost of goods sold (Cost
of goods sold is 80% of sales)
♠Payment for the components is made in the month following the purchase.
♠Expected operating expenses during December is 20% of sales of which 25% are noncash expenses
and the remainder is paid during the same month
Description November December January
$150,000
Sales (actual/projected) = $375,000 $520,000 $500,000
40%
Collections
During month 60% 225,000 312,000 300,000
Next month 40% ? 150,000 208,000
Total collections ? $462,000 $508,000
Component's cost (80% ×
$120,000 $166,400 $160,000
40%) =32% of sales
Purchases during month (80% $166,400 × 80% = $160,000 ×
prior to the month of sale) $133,120 80%=$128,000 ?
$160,000
Purchases during month (20% $120,000 × 20% = $166,400 × 20% = ×
in the month of sale) $24,000 $33,280 20% =
$32,000
Total purchases $157,120 $161,280 ?
Purchases payments $157,120 $161,280

Q37: what is the projected A/P balance on Dec, 31, assuming all A/P balance in November, 30 is
paid during December.
Answer: Projected A/P balance on Dec, 31, is 161,280 as follows.
DR. Accounts payable account CR.
Payment of previous month balance 175,000 Beg. A/P Balance 175,000
Ending A/P Bal. 161,280 Purchases during the month $161,280
Total 336,280 Total 336,280

Q38: What is the projected gross profit for the month ending December, 31?
Answer: projected gross profit for the month ending December, 31
= $520,000 × 20% = $104,000.
Q39: What is the projected A/R balance on Dec, 31?
Answer: $208,000 (prove)
Q40: What is the projected Cash balance on Dec, 31?
Answer:
Description Amount
Beginning cash balance (Dec,1) 52,000
Collections during December 462,000
Payments for purchases and other payables (175,000)
Cash operating Expenses 520,000 × 20% × 75% (78,000)
Ending cash balance (Dec,31) 261,000

Comparative B/S
Assets Nov Dec
Cash 52,000 261,000
Accounts receivable, net 150,000 208,000
Inventory 315,000 60,280 (1)
Property plant and equipment 1,000,000 974,000 (2)
Total assets 1,517,000 1,503,280
Liabilities
Accounts payable ($17,880 other payables) 175,000 161,280
Common stock (C/S) 900,000 900,000
Retained earnings (R.E) 442,000 442,000(3)
Total liabilities and shareholders equity 1,517,000 1,503,280

(1) 315,000 + 161,280 – 416,000 = 60,280

(2) 1,000,000 – (520,000 × 20% × 25%) = 974,000

(3) Net income = 0

520,000 × 20% G.P – (520,000 × 20% operating expenses)

Fact pattern (questions from 41 through 43): The Raymar Company is preparing its cash budget for
the months of April and May. The firm has established a $200,000 line of credit with its bank at a 12%
annual rate of interest on which borrowings for cash deficits must be made in $10,000 increments. There
is no outstanding balance on the line of credit loan on April 1. Principal repayments are to be made in any
month in which there is a surplus of cash. Interest is to be paid monthly. If there are no outstanding
balances on the loans, Raymar will invest any cash in excess of its desired end-of-month cash balance in
U.S. Treasury bills. Raymar intends to maintain a minimum balance of $100,000 at the end of each
month by either borrowing for deficits below the minimum balance or investing any excess cash.
Expected monthly collection and disbursement patterns are shown below.

• Collections 50% of the current month’s sales budget and 50% of the previous month’s sales budget.
• Accounts payable disbursements 75% of the current month’s accounts payable budget and 25% of the
previous month’s accounts payable budget.
• All other disbursements occur in the month in which they are budgeted.

Budget Information
Description March April May
Sales $40,000 $50,000 $100,000
Accounts payable 30,000 40,000 40,000
Payroll 60,000 70,000 50,000
Other disbursements 25,000 30,000 10,000

Q41: In April, Raymar’s budget will result in


A. $45,000 in excess cash.
B. A need to borrow $50,000 on its line of credit for the cash deficit.
C. A need to borrow $100,000 on its line of credit for the cash deficit.
D. A need to borrow $90,000 on its line of credit for the cash deficit.
Answer (C) is correct.

Description March April May

Sales $40,000 $50,000 $100,000


Collections -50% same month 20,000 25,000 50,000
Collections -50% from previous month ? 20,000 25,000
Total expected collections ? $45,000 $75,000
Accounts payable 30,000 40,000 40,000
A/P disbursements 75% same month 22,500 30,000 30,000
A/P disbursements 25% from previous month ? 7,500 10,000
Payroll disbursements 60,000 70,000 50,000
Other disbursements 25,000 30,000 10,000
Total disbursements ? $137,500 $100,000

Beginning cash balance 100,000 107,500


Total expected collections ? $45,000 $75,000
Total cash available for use 145,000 182,500
Total disbursements ? $137,500 $100,000
Interest payments - 1,000
Preadjusted ending cash balance 7,500 81,500
Loan borrowings 100,000 20,000
Ending cash balance 107,500 101,500
Loan balance 100,000 120,000

Q42: In May, Raymar will be required to


A. Repay $20,000 principal and pay $1,000 interest.
B. Repay $90,000 principal and pay $100 interest.
C. Pay $900 interest.
D. Borrow an additional $20,000 and pay $1,000 interest.
Answer (D) is correct.
Q43: The cash receipts budget includes:
A. Interest expense. B. Operating supplies C. Loan proceeds. D. Extinguishment of debt.
The correct answer is (C) Loan proceeds are a cash receipt all of the other choices are cash
disbursements
Q44: The treasury analyst for Garth Manufacturing has estimated the cash flows for the first half
of next year (ignoring any short- term borrowings) as follows:
Month January February March April May June
Cash Inflows (millions) $2 2 2 2 4 5
Cash Outflows (millions) 1 4 5 3 2 3

Garth has a line of credit of up to $4 million on which it pays interest monthly at a rate of 1% of
the amount utilized. Garth is expected to have a cash balance of $2 million on January 1 and no
amount utilized on its line of credit. Assuming all cash flows occur at the end of the month, how
much will Garth pay in interest during the first half of the year?
A. $0 B. $61,000 C. $65,000 D. $132,000

Answer (B) is correct.

Description January February March April May June


Beginning cash balance 2,000,000 3,000,000 1,000,000 0 0 0
Cash Inflows 2,000,000 2,000,000 2,000,000 2,000,000 4,000,000 5,000,000
Available cash 4,000,000 5,000,000 3,000,000 2,000,000 4,000,000 5,000,000
Cash Outflows 1,000,000 4,000,000 5,000,000 3,000,000 2,000,000 3,000,000
Interest payments - - 20,000 30,200 10,502
Preadjusted cash balance 3,000,000 1,000,000 (2,000,000) (1,020,000) 1,969,800 1,989,498
Borrowings /repayments - - 2,000,000 1,020,000 (1,969,800) 1,050,200
Ending cash balance 3,000,000 1,000,000 0 0 0 939,298
Loan balance - - 2,000,000 3,020,000 1,050,200 0
Total interest payments = 20,000 +30,200 +10,502 = 60,702

Q45: A firm develops an annual cash budget in order to


A. Support the preparation of its cash flow statement for the annual report.
B. Ascertain which capital expenditure projects are feasible and which capital expenditure projects should
be deferred.
C. Determine the opportunity costs of alternative sales and production strategies.
D. Avoid the opportunity costs of non-invested excess cash and minimize the cost of interim financing.
Answer (A) is incorrect because the cash flow statement is based on actual results, not budgeted figures.
Answer (B) is incorrect because while a cash budget may facilitate decisions regarding deferral of capital
projects; the cash budget does not ascertain which projects are feasible. The cash budget does not ascertain
which capital expenditure projects are feasible or should be deferred. It only shows the cash available for
projects and other activities. The cash budget provides the total liquidity available for projects.
Answer (C) is incorrect because cash budgets do not determine opportunity costs for such matters.
Answer (D) is correct. The cash budget is perhaps the most important part of a company’s budget
program. A cash budget facilitates planning for loans and other financing. Conversely, a firm should plan
how to invest temporary surpluses of cash. A cash budget is particularly valuable in seasonal businesses
in which a few months of revenues must be matched with 12 months of costs. Because a temporary
shortage of cash may drive an otherwise financially sound organization into bankruptcy, proper planning
can prevent financial embarrassment.
Q46: The most direct way to prepare a cash budget for a manufacturing firm is to include
A. Projected sales, credit terms, and net income.
B. Projected net income, depreciation, and goodwill impairment.
C. Projected purchases, percentages of purchases paid, and net income.
D. Projected sales and purchases, percentages of collections, and terms of payments.
Answer (A) is incorrect because net income includes non-cash elements, e.g., goodwill impairment and
depreciation.
Answer (B) is incorrect because net income includes non-cash elements, e.g., goodwill impairment and
depreciation.
Answer (C) is incorrect because collection percentages must be considered, and net income includes non-
cash elements.
Answer (D) is correct. The most direct way of preparing a cash budget requires incorporation of sales
projections and credit terms, collection percentages, estimated purchases and payment terms, and other
cash receipts and disbursements. In other words preparation of the cash budget requires consideration
of both inflows and outflows.

Q47: A firm averages $4,000 in sales per day and is paid, on an average, within 30 days of the sale. After
they receive their invoice, 55% of the customers pay by check, while the remaining 45% pay by credit
card. Approximately how much would the company show in A/R on its balance sheet on any given date?
A. $4,000 B. $120,000 C. $48,000 D. $54,000
Answer (A) is incorrect because $4,000 is only one day’s sales.
Answer (B) is correct. If sales are $4,000 per day, and customers pay in 30 days, 30 days of sales are
outstanding, or $120,000. Whether customers pay by credit card or cash, collection requires 30 days.
Answer (C) is incorrect because invoices are outstanding for 30 days, not 12 days.
Answer (D) is incorrect because $54,000 is based on the 45% of collections via credit card.

Q48: Moor Company’s budgeted sales for the coming year is $40,500,000, of which 80% are expected
to be credit sales at terms of N/30 Moor estimates that a proposed relaxation of credit standards will
increase credit sales by 20% and increase the average collection period from 30 days to 40 days. Based
on a 360-day year, the proposed relaxation of credit standards will result in an expected increase in the
average A/R balance of
A. $540,000 B. $2,700,000 C. $900,000 D. $1,620,000
Answer (D) is correct:

Description Before relaxation After relaxation Difference


Projected sales $40,500,000
Credit sales $40,500,000 × 80% = $32,400,000×120% = $6,480,000
$32,400,000 $38,880,000
Daily credit sales 32,400,000 ÷ 360 = $3,888,000 ÷ 360 = $18,000
$90,000 $108,000
A/R balance $90,000× 30 days = $108,000× 40 days = $1,620,000
$2,700,000 $4,320,000

Q49: Percy publishing is considering a change in its credit terms from N/30 to 2/10, N/30. The
company’s budgeted sales for the coming year are $24,000,000, of which 90% are expected to be made
on credit. If the new credit terms are adopted, Percy estimates that discounts will be taken on 50% of the
credit sales; however, uncollectible accounts will be unchanged. The new credit terms will result in
expected discounts taken in the coming year of
A. $216,000. B. $432,000 C. $240,000 D. $480,000
Answer (A) is correct.
Expected discounts taken = $24,000,000 × 90% × 50% × 2% = $216,000

Q50: In November, a company finalized its budget for the upcoming calendar year. In December, the
decision was made to acquire new equipment in January by trading in old equipment and financing the
amount due by a loan with principal and interest due at the end of three years. Out-of-pocket costs to
operate the machinery would not change. This decision would change which of the company's budgeted
F/S for the upcoming year?
A. The budgeted balance sheet only.
B. Both the budgeted balance sheet and the income statement.
C. The budgeted balance sheet, the income statement, and the statement of cash flows.
D. Both the budgeted income statement and the statement of cash flows.
Answer (C) is correct:

Q51: The Alsner Company's budgeted sales of $220,000 for June, $200,000 for July, $280,000 for
August, $264,000 for September, $244,000 for October, and $300,000 for November.
Approximately 75% of sales are on credit; the remainder is cash sales. Collection experience
indicates that 60% of the budgeted credit sales will be collected the month after the sale, 36% the
second month, and 4% will be uncollectible. Which month has the highest budgeted cash receipts?
A. August. B. September. C. October. D. November.
Answer (D) is correct

Description June July August September October November


budgeted sales ($) $220,000 $200,000 $280,000 $264,000 $244,000 $300,000
Collections
25% Same month 55,000 50,000 70,000 66,000 61,000 75,000
45% Second month 99,000 90,000 126,000 118,800 109,800
27% Third month 59,400 54,000 75,600 71,280
Total Collections ? ? 219,400 246,000 255,400 256,080

Fact pattern (questions from 52 through 53): Paradise Company budgets on an annual basis
for its fiscal year. The following beginning and ending inventory levels (in units) are planned for
the fiscal year of July 1, Year 1, through June 30, Year 2.
Description July 1, Year 1 June 30, Year 2
Raw material* 40,000 50,000
Work-in-process-equivalent 10,000 10,000
Finished goods 80,000 50,000

* Two units of raw material are needed to produce each unit of finished product.

Q52: If Paradise Company plans to sell 480,000 units during the fiscal year; the number of units
it will have to manufacture during the year is
A. 440,000 units. B. 480,000 units. C. 510,000 units. D. 450,000 units.
Answer (D) is correct.
Production quantity budget = 480,000 – (10,000 + 80,000) + (10,000 + 50,000) = 450,000
units

Q53: If 500,000 complete units were to be manufactured during the fiscal year by Paradise
Company, the number of units of raw materials to be purchased is
A. 1,000,000 units. B. 1,020,000 units. C. 1,010,000 units. D.990,000 units.
Answer (C) is correct. Units of raw materials to be purchased = [500,000× 2] - 40,000 + 50,000
= 1,010,000 units
Q54: Karmee Company has been accumulating operating data in order to prepare an annual profit plan
details regarding Karmee’s sales for the first 6 months of the coming year are as follows:

Description January February March April May June


Estimated monthly sales $600,000 650,000 700,000 625,000 720,000 800,000
Type of monthly sale Cash sales 20% Credit sales 80%
Collection pattern for
Month of sale 30% 80% × 30% = 24%
credit sales
One month following sale 40%80% × 40% = 32%
Second month following sale 25%80% × 25% = 20%
Bad debt 5% 80% × 30% = 4%

Karmee’s cost of goods sold averages 40% of the sales value.


Karmee’s objective is to maintain at target inventory equal to 30% of the next month’s sales in
unit. Purchases of merchandise are paid for in the month following the sale. The variable
operating expenses (other than cost of goods sold) for Karmee are 10% of sales and are paid for
in the month following the sale. The annual fixed operating expenses are presented below. All of
these are incurred uniformly throughout the year and paid monthly except for insurance and
property taxes. Insurance is paid quarterly in January, April, July, and October. Property taxes
are paid twice a year in April and October.
Annual fixed operating costs:
Advertising $720,000
Deprecation 420,000
Insurance 180,000
Property taxes 240,000
Salaries 1,080,000
Other fixed operating expenses (paid uniformly) include Advertising and Salaries
= $720,000 +1,080,000 = 1,800,000 ÷ 12 months = $150,000

Required: Prepare a statement showing total monthly collections and payments as well as change in
cash balance during the period from March through June.

Description January February March April May June


Estimated monthly sales $600,000 650,000 700,000 625,000 720,000 800,000
Cash sales collection (20%) 120,000 130,000 140,000 125,000 144,000 160,000
Credit sales collections

Month of sale 24% 144,000 156,000 168,000 150,000 172,800 192,000

One month following sale ? 192,000 208,000 224,000 200,000 230,400


32%
Second month following ? ? 120,000 130,000 140,000 125,000
sale 20%
Total estimated collections ? ? 636,000 629,000 656,800 707,400
COGS (40%) 240,000 260,000 280,000 250,000 288,000 320,000
Beginning inventory 72,000 78,000 84,000 75,000 86,400 96,000
(30%) of next month
Ending inventory (30%) 78,000 84,000 75,000 86,400 96,000 ?
of next month
Monthly purchases 246,000 266,000 271,000 261,400 297,600 ?
Payment for purchases ? 246,000 266,000 271,000 261,400 297,600
Payment for variable ? $60,000 65,000 70,000 62,500 72,000
operating expense
Insurance Payments $45,000 ---- ---- $45,000 ---- ----
Property taxes Payments ---- ---- ---- $120,000 ---- ----
Other fixed operating $150,000 $150,000 $150,000 $150,000 $150,000 $150,000
expenses (paid uniformly)
Total monthly payments ? 456,000 481,000 656,000 473,900 519,600
Change in cash balance 155,000 (27,000) 182,900 187,800

Q55: As part of the master budget process, a merchandising company begins to prepare the cash budget
for the same period. Which of the following additional information will be most useful to management in
preparing this budget?
A. Sales, credit policies, purchasing terms, and planned capital acquisition.
B. Projected revenues, projected expenses, and intended financing activities.
C. Credit policies, projected expenses, and inventory procurement policies.
D. Planned direct material purchases, planned direct labor, and purchasing terms.
Answer (A) is correct

Fact pattern (questions from 56 through 59): Kelly Company is a retail sporting goods store that uses
accrual accounting for its records. Facts regarding Kelly’s operations are as follows:
• Sales are budgeted at $220,000 for December Year one and $200,000 for January Year two.
• Collections are expected to be 60% in the month of sale and 38% in the month following the sale.
• Gross margin is 25% of sales.
• A total of 80% of the merchandise held for resale is purchased in the month prior to the month of sale and
20% is purchased in the month of sale. Payment for merchandise is made in the month following the
purchase.
• Other expected monthly expenses to be paid in cash are $22,600.
• Annual depreciation is $216,000.
Below is Kelly Company’s statement of financial position at November 30, Year one.

Assets Subtotal Total


Cash $22,000
A/R (net of $4,000 allowance) 76,000
Inventory 132,000
P,P,E (net of $680,000 ACC.DEP) 870,000
Total assets $1,100,000
Liabilities and Stockholders’ Equity
Accounts payable $162,000
Common stock 800,000
Retained earnings 138,000
Total liabilities and stockholders’ equity $1,100,000
Q54: Kelly’s budgeted cash collections for December Year one are
A. $208,000 B. $132,000 C. $203,600 D. $212,000
Answer (A) is correct.

Description November December January


Projected and actual gross sales $200,000 $220,000 $200,000
Collections
60% month of sale 120,000 132,000 120,000
38% next month ? 76,000 83,600
Total collections ? 208,000 203,600
COGS 75% 150,000 165,000 150,000
Purchases -80% month prior to sale 132,000 120,000 ?
Purchases -20% in the month of sale 30,000 33,000 30,000
Total purchases 162,000 153,000 ?
Merchandise payments ? 162,000 153,000
Monthly cash expenses $22,600 $22,600 $22,600
Total payments ? 184,600 175,600

Note: During December the following entries would be made


DR: Provision for doubtful accounts (220,000 × 2%) = $4,400
CR: Allowance for doubtful accounts (220,000 × 2%) =$4,400

DR: Allowance for doubtful accounts (200,000 × 2%) = $4,000


CR: A/R (200,000 × 2%) = $4,000

Q55: Kelly’s projected balance in accounts payable on December 31, Year 1, is


A. $162,000 B. $204,000 C. $153,000 D. Some amount other than those given
Answer (C) is correct. A/P balance would represent December purchases of merchandise at $153,000

Q56: Kelly’s projected balance in inventory on December 31, Year one, is


A. $160,000 B. $120,000 C. $153,000 D. $150,000

Answer (B) is correct. Beginning inventory at December, 1 $132,000 + purchases during

Description Amount Amount


Sales $220,000
Less : COGS 165,000
Gross profit 55,000
Less : Operating cash expenses $22,600
Less : Depreciation ($216,000 ÷ 12 month) 18,000
Less: Bad debt expenses (220,000 × 2%) 4,400
EBIT 10,000
December $153,000 – COGS $165,000 = $120,000

Q57: Kelly’s pro forma income (loss) before income taxes for December Year one is
A. $32,400 B. $28,000 C.$10,000 D. Some amount other than those given

Answer (C) is correct.


Pro forma B/S
Assets NOV,30 DEC,31
Cash $22,000 45,400
A/R- gross 80,000 220,000 × 40% = 88,000
Less allowance (4,000) (4,400)
NRV 76,000 83,600
Inventory 132,000 $120,000
P,P,E 1,550,000 1,550,000
Less ACC.DEP $680,000 $680,000 +18,000 = 698,000
Book value of PPE 870,000 852,000
Total assets $1,100,000 1,101,000
Liabilities and Stockholders’ Equity
Accounts payable $162,000 153,000
Common stock 800,000 800,000
Retained earnings 138,000 148,000
Total liabilities and stockholders’ equity $1,100,000 1,101,000

Statement of cash flow – indirect method – December


Net income (accrual) 10,000
Add depreciation expense 18,000
Add provision for doubtful debts 4,400
Add decrease in inventory 12,000
Less increase in gross A/R (8,000)
Less A/R written of from beg Bal (4,000)
Less decrease in A/P (9,000)
Net cash inflow from operations 23,400
Add beginning cash balance $22,000
Ending cash balance 45,400

See Gleim's MCQ's # 168, 200, 202, 203, 205, 207, 237, 238, 240, 241, 243, 244, 245, 255, 256,
257, 258, 259, 260, 261, 262, 263, 264, 265, 266, 268, 269, 270, 272, 273, 274, 275,276,277,278,
286, 287, 288, 289, 290, 291, 292, 293,

Fact pattern: Whitehall Company is a manufacturer of leather goods operating from several regional
divisions treated individually as profit centers. A master operating plan based on divisional
projections for each fiscal year is prepared at corporate headquarters. Whitehall implemented an
incentive plan to reward the performance of key personnel, the plan is designed to avoid “managed
earnings” and recognize careful planning. Whitehall’s income statement for the year ended November
30, 2013, is presented below.

Whitehall Company - Income Statement


For the Year Ended November 30, 2013 (in thousands)
Net sales $16,800
Cost of goods sold $12,600
Selling expense 1,500
Administrative expense 1,800
Interest expense 280
Total expenses 16,180
Income before income taxes 620
Income taxes (40%) 248
Net income 372

The master budget for fiscal year 2014 is based on the following operational directives:

• Increase unit sales five percent.


• Increase selling prices ten percent.
• Increase advertising by $300,000 while holding all other selling and administrative expenses at fiscal
2013 levels.
• Increase production capacity by increasing the average investment in inventory and equipment by
$600,000 while holding all other assets at fiscal 2013 levels.
• Finance the additional assets at an annual interest rate of ten percent and hold all other interest expense
at fiscal 2013 levels.
• Improve the marketability of products carried which will increase the unit COGS by seven percent.
Whitehall’s 2014 effective income tax rate is expected to be 40 percent, the same as in fiscal 2013.

Required:

Prepare a pro forma I/S for Whitehall Company for the fiscal year ending November 30, 2014.

Whitehall Company - Income Statement


For the Year Ended November 30, 2014 (in thousands)
Net sales (16,800 ×105%×110%) $19,404,000
Cost of goods sold (($12,600 × 105% ×107%) $14,156,100
Selling expense (1,500,000 + 300,000) 1,800,000
Administrative expense 1,800,000
Interest expense (280 + 60) 340,000
Total expenses 18,096,100
Income before income taxes 1,307,900
Income taxes (40%) 523,160
Net income 784,740

Fact pattern: Jay manufactures products for networking, video-conferencing equipment. Production
of specialized units is to a large extent, performed under contract, with standard units manufactured
to marketing projections. Maintenance of customer equipment is an important area of customer
satisfaction. Jay’s income statement for the fiscal year ended October 31, Year 1, is presented below.

Jay Income Statement


For the Year Ended October 31, Year 1 ($000 omitted)
Equipment sales $6,000
Maintenance contracts services 1,800
Total net revenue 7,800
Expenses:
Cost of goods sold 4,600
Customer maintenance 1,000
Selling expense 600
Administrative expense 900
Interest expense 150
Total expenses 7,250
Income before income taxes 550
Income taxes (40%) 220
Net income 330

Jay’s management considered the growing videoconferencing market when it proposed the following
actions for fiscal Year 2:
• Increase equipment sales prices by 10%.
• Increase the cost of each unit sold by 3% for needed technology and quality improvements, and
increased variable costs.
• Increase maintenance inventory by $250,000 at the beginning of the year and add two maintenance
technicians at a total cost of $130,000 to cover wages and related travel expenses. These revisions are
intended to improve customer service and response time. The increased inventory will be financed at an
annual interest rate of 12%; no other borrowings or loan reductions are contemplated during fiscal Year
2. All other assets will be held to fiscal Year 1 levels.
• Increase selling expenses by $250,000 but hold administrative expenses at Year 1 levels.
• The effective rate for Year 2 federal and state taxes is expected to be 40%, the same as Year 1.
These actions are expected to increase equipment unit sales by 6%, with a corresponding 6% growth in
maintenance contracts.

Required: Prepare a pro forma income statement for Jay Company for the fiscal year ending
October 31, Year 2.

Jay Income Statement


For the Year Ended October 31, Year 2 ($000 omitted)
Equipment sales 6,996,000
Maintenance contracts services 1,908,000
Total net revenue 8,904,000
Expenses:
Cost of goods sold (4,600 ×106% ×103%) 5,022,280
Customer maintenance (1,000 +130) 1,130,000
Selling expense (600 +250) 850,000
Administrative expense 900,000
Interest expense [(150) + (250 ×12%)] 180,000
Total expenses 8,082,280
Income before income taxes 821,720
Income taxes 328,688
Net income 493,032

ESSAY: Healthful Foods Inc., a manufacturer of breakfast cereals and snack bars, has experienced several
years of steady growth in sales, profits, and dividends while maintaining a relatively low level of debt. The
Board of Directors has adopted a long-run strategy to maximize the value of the shareholders’ investment.
In order to achieve this goal, the Board of directors established the following five-year financial objectives.
• Increase sales by 12 % per year.
• Increase income before taxes by 15 % per year.
• Increase dividends by 12 % per year.
• Maintain long-term debt at a maximum 16 % of assets.
These financial objectives have been attained for the past three years. At the beginning of last year, the
president of Healthful Foods, Andrea Donis, added a fifth financial objective of maintaining COGS at a
maximum of 70 % of sales, and this goal was attained last year.
The budgeting process at Healthful Foods is to be directed toward attaining these goals for the forthcoming
year, a difficult task with the economy in a prolonged recession. In addition, the increased emphasis on
eating healthful foods has driven up the price of ingredients used by the company significantly faster than
the expected rate of inflation.
John Winslow, cost accountant at Healthful Foods, has responsibility for the preparation of the profit plan
for next year. Winslow assured Donis that he could present a budget that achieved all of the financial
objectives. Winslow believed that he could overestimate the ending inventory and reclassify fruit and grain
inspection costs as administrative rather than manufacturing costs to attain the desired objective. Presented
below and in the next column are the actual statements for 2013 and the budgeted statements for 2014 that
Winslow prepared.

Healthful Foods Inc. Income Statement 2013 - 2014


Description Actual- 2013 Budget- 2014
Sales $850,000 $947,750
Variable costs
Cost of goods sold 510,000 574,725
Selling & administrative expenses 90,000 87,500
Contribution margin 250,000 285,525
Fixed costs
Manufacturing 85,000 94,775
Selling & administrative 60,000 70,000
Income before taxes 105,000 120,750

Healthful Foods Inc. Statement of financial position 2013- 2014


Description Actual- 2013 Budget- 2014
Assets
Cash 10 17
Accounts receivable 60 68
Inventory 300 365
Plant & equipment (net) 1,630 1,600
Total assets 2,000 2,050
Liabilities:
Accounts payable $110 $122
Long-term debt 320 308
Shareholders equity:
Common stock 400 400
Retained earnings 1,170 1,220
Total 2,000 2,050

The company paid dividends of $27,720 in 2013, and the expected tax rate for 2014 is 34 percent.

Required:

A. Describe the relationship between strategic planning and budgeting.


B. For each of the five financial objectives established by the Board of Directors and the president of
Healthful Foods Inc., determine whether John Winslow's budget attains these objectives. Support your
conclusion in case by presenting appropriate calculations, and use the following format for your answer.

Objective Attained/not attained Calculations

Solution
A. Strategic planning identifies the overall objectives of an organization and generally considers
the impact of external factors such as competitive forces, market demand, and technological
changes when identifying overall objectives.
Budgeting is the quantitative expression of plans evolving from strategic planning. The time
horizon for budgeting is generally a year, or an operating cycle, and greater attention is focused on
internal factors than external factors.

B. For each of the following financial objectives established by the Board of Directors and president
of Healthful Foods Inc., the calculations to determine whether John Winslow’s budget attains these
objectives are presented in Exhibit 1 below.

Healthful Foods Inc.

Objective Attained/Not Calculations


Attained

1) Increase sales by 12 % per year. Not Attained ($947,750 - $850,000) ÷$850,000 =11.50%

2) Increase income before taxes by 15 % Attained (105,000 -120,750) ÷ 105,000 = 15%


per year.

3) Increase dividends by 12 % per year. Not Attained ($27,720 -$29,695*) ÷27,720 =7.12%

4) Maintain long-term debt at a Attained (308 ÷ 2,050) =15%


maximum 16 % of assets.

5) maintaining cost of goods sold at a Not Attained (574,725 + 94,775) ÷ $947,750) = 70,64%
maximum of 70 % of sales

Expected dividends for the year 2014 = beginning R.E 1,170,000 + net income (120,750 × 66%) –
ending R.E 1,220,000 = $29,695*

Issues that would be considered in evaluating the adequacy of budgeted annual operating income

When the company prepares the first draft of its pro-forma 1/S management must evaluate whether
earnings meet company objectives. This evaluation based on many factors such as:
NI .available.to.C / S .Holders
(1) Desired earnings per share (EPS), where EPS =
Weighted . Average.#.of .C / S
(2) Average earnings for other comparable firms in the industry.
FMV.of .C / S
(3) Desired price-earnings ratio =
EPS
(4) bond covenant restrictions. Debt covenants are clauses in debt contracts that organizations must
meet to be in compliance with lender requirements. Financial forecasts are often used in predicting
whether performance related debt covenants will be met.
(5) Management long - range profit objectives.
NI
(6) Return or investment (ROI), where ROI =
Average.Total. Assets
Net.income.available.to.C / S .holders
(7) Return on stockholder equity where ROE=
Average.common.equity($)
Net. profit
(8) Profit margin on sales =
Net.total.sales
By using these traditional measures if the budgeted income per pro-forma 1/S is inadequate, the budgeting
process must usually begin a-new with different assumptions.
Note: Internal rate of return on investment (IRR) is a method used to evaluate long-term capital projects
but not the budgeted operating income.

Q1: Of the following items, the one item that would not be considered in evaluating the adequacy of the
budgeted annual operating income for a company is
A. Debt to equity ratio B. Price-earnings ratio
C. Earnings per share. D. Industry average for earnings on sales.
Answer (A) is correct: debt to equity ratio is a risk measure of financial leverage
Answer (B) is incorrect because the price-earnings ratio is a financial performance measure and can be
used to measure the adequacy of the budgeted annual operating income.
Answer (C) is incorrect because earning per share represents a financial performance measure and can be
used to measure the adequacy of the budgeted annual operating income.
Answer (D) is incorrect because the industry average for earnings on sales is a financial performance
measure and can be used to measure the adequacy of the budgeted annual operating income

Q2: Of the following items, the one item that would be considered in evaluating the adequacy of
the budgeted annual operating income for a company is
A. Interest coverage ratio B. ROE
C. Current ratio. D. Cash flow from operating activities. E. Internal rate of return.
Answer (A) is incorrect because interest coverage ratio is a risk measure of financial leverage
Answer (B) is correct: ROE is a return measure
Answer (C) is incorrect because current ratio is a liquidity measure
Answer (D) is incorrect because cash flow from operating activities is a liquidity measure
Answer (E) is incorrect because the internal rate of return is used to evaluate investment decisions and
involves the time value of money. IRR represents the discount rate at which the net present value of an
investment is equal to zero. IRR is not used to evaluate the adequacy of budgeted operating income

Q3: Shoo Inc. owns several retail stores. After all initial budget requests were received for the
upcoming year, Shoo’s abbreviated pro forma income statement is as follows.
Sales $46,000,000
Cost of goods sold 20,700,000
Selling and administrative costs 19,800,000
Operating income $ 5,500,000
The COGS and a 5% sales commission are the only variable costs. Shoo’s upper management believes that
the sales manager underestimated projected sales units and wants the sales budget increased such that the
company can achieve its goal of a 15% return on sales. The amount by which sales must increase to achieve
this goal is
A. $4,000,000 B. $3,500,000 C. $1,750,000 D. $1,400,000
Answer (A) is correct:

Description Income statement item Notes


Sales X → 100%
COGS + commission 50% of X 20.70/46 + 5% commission
C.M 50% of X
Fixed operating costs 17.50 should be 35% of X to 19.80- 46 × 5% = 17.50
leave 15% EBIT

Fixed operating costs of 17.50 represents 35% of sales Thus sales = 17.50/ 35% = $50Million
♠The amount by which sales must increase to achieve this goal = $50Million - $46 Million = $4 Million

Q4: What is the significance of financial forecasts in relation to debt covenants?


A. Financial forecasts play no role in determining the probable adherence to debt covenants
B. A debt covenant does not have to be adhered to so long as the company is profitable
C. Financial forecasts provide a basis for creditors to gain comfort that debt covenants will be met
D. Providing a detailed financial forecast assures that debt covenants are being met no matter the results
Answer (C) is correct. Debt covenants are clauses in debt contracts that organizations must meet to be
in compliance with lender requirements. Financial forecasts are often used in predicting whether
performance related debt covenants will be met.

Q5: Netco sales budget for the coming year is as follow.

Item Volume in units Sales price Sales revenue


1 200,000 $50 $10,000,000
2 150,000 10 1,500,000
3 300,000 30 9,000,000
Total sales revenue $20,500 000

Items I and 3 are different models of the same product. Item 2 is a complement to Item 1 Past experience
indicates that the sales volume of item 2 relative to the sales volume of item 1 is fairly constant Netco is
considering a 10% price increase for the coming year for item 1 which will cause sales of item 1 to
decline by 20%. While simultaneously causing sales of item 3 to increase by 5% If Netco institutes the
price increase for item 1. Total sales revenue will decrease by
A. $1,050,000 B. $550,000 C. $850,000 D. $750,000

The correct answer is (A)

Item Volume in Units Sales Price Sales Revenue


1 200,000 × 80%=160,000 $50 ×110% = $55 8,800,000
2 (160,000/4) ×3 =120,000 10 $1,200,000
3 300,000 × 105%=315,000 30 9,450,000
Total sales revenue 19,450,000
$19,450,000 is $1,050,000 less than the 20,500,000 originally projected

Q6: Steers Company has just completed its pro forma financial statements for the coming year.
Relevant information is summarized below.
Projected net income $100,000
Anticipated capital expenditures 50,000
Increase in working capital 25,000
Depreciation expense 15,000
From the information provided above, the increase in Steers’ cash account for the coming year will be:
A. $25,000. B. $40,000 C. $90,000 D. $160,000
correct answer is (B)

Net income $100,000


less Capital expenditures (50,000)
less Increases in working capital (25,000)
Add Depreciation expense 15,000
Increase in cash $ 40,000

Q7: Steers Company has just completed its pro forma financial statements for the coming year. Relevant
information is summarized below.

Assuming that the increase in working capital was the result of a decrease in the accounts
payable balance, how much can Steers use for capital expenditures without rising outside
financing?

A. $100,000 B. $50,000 C. $40,000 D. $90,000

Answer (D) is correct. The amount Steers can use for capital expenditures without having to resort
to outside financing sources is equal to the pro forma amount of cash flow from operations. The pro
forma cash flow from operations is calculated by taking the pro forma net income $100 plus the pro
forma depreciation $15 less the pro forma increase in working capital. $25

Q8: The controller of Nottingham Stores has asked a staff accountant to prepare detailed reports that
summarize the firm’s cash flows for the upcoming accounting period and cash position at the end of the
period. Accordingly, the controller has requested preparation of a cash budget, a pro-forma statement of
cash flows, a detailed listing of cash collections from customers, and a detailed listing of cash payments
for merchandise purchases. Which one of the following correctly identifies the first and last document
to be prepared by the accountant?

Choice First document Last document


A Listing of cash collections Pro-forma statement of cash flows
B Listing of cash collections Cash budget
C Cash budget Either the listing of cash collections or listing of cash
payments, the order of which is unimportant.
D Listing of cash payments Either the pro-forma statement of cash flows or the
cash budget, the order of which is unimportant.

Answer (A) is correct

Q9: Which one of the following is the best use of pro forma financial statements?
A. Enable the treasurer to make appropriate short-term investments.
B. Show prospective investors how they will earn an appropriate rate of return
C. Enable top management to control the activities of subordinate managers
D. Review the company’s financial performance over the past five years
Answer (B) is correct. Prospective investors use financial statements, both historical and pro forma,
to estimate the statement issuer’s future economic performance (cash flows and income) and to
estimate the expected return on an investment in the issuer’s securities. The prospective investor
compares the expected return to his/her required rate of return to see if it is appropriate.

Q10: Pro forma F/S is used within a company for various purposes. They are not used for
A. Comparison with actual results for performance reporting in order to determine employee bonuses
B. Determining whether the company will be in compliance with required covenants on its long-term
debt.
C. "What if" analysis, to forecast the effect of a proposed change
D. Determining the company's future needs for external financing.
Answer (A) is correct: because comparing actual results to planned results for the purpose of
performance reporting is done by comparing the actual to either the flexible budget or the master
budget. A pro forma financial statement may be prepared as a part of the formal planning process that
eventually results in the master budget or the flexible budget. Or pro forma statements may be
prepared after the formal budget for the year has been adopted, if the company is considering an
activity that was not foreseen before the formal budget was adopted. And if a pro forma statement has
been prepared for an activity being considered, actual results will probably be compared with the pro
forma statement in order to determine whether the company's objectives were met. But pro forma
statements are not used for formal performance reporting to determine employee bonuses.
Answer (B) is incorrect because one of the purposes of pro forma F/S is determining whether the
company will be in compliance with required covenants on its long-term debt.
Answer (C) is incorrect because one of the purposes of pro forma financial statements is for doing
"what if" analysis, to forecast the effect of a proposed change such as a price increase that
management anticipates will reduce the demand for the product.
Answer (D) is incorrect because one of the purposes of pro forma financial statements is determining
in advance what the company's future financing needs will be.

Fact pattern: Angelina co. is a manufacturer of notebooks with a comprehensive annual budgeting
process that ends with the preparation of pro forma F/S. All underlying budget schedules have been
completed for the year ending December 31, 2010 and selected data from these schedules are presented
below. Also shown are the pro forma statement of cash receipts and disbursements for the year ending
December 31, 2010 and the pro forma statement of financial position as of December 31, 2009.
Angelina uses the accrual basis of accounting. To facilitate the budgeting process Angelina accumulates
all raw materials, D.L and manufacturing O.H with the exception of depreciation) Selling and
administrative costs in an account called expenses payable. The company’s income tax rate is 40% and
income tax expense is classified as current income taxes payable. Selected data from 2010 budget
schedules:
The majority of sales are on account.

Unit sales Unit price Total revenue


9,500,000 $5.50 $52,250,000
Production units Unit cost Total manufacturing cost
9,640,000 $ 4.75 $ 45,790,000

Raw materials purchases


Item Quantity Unit cost Total purchases
Ring (one per unit) 9,600,000 $0 .80 $7,680,000
Cover (2 per unit) 18,800,000 $0.30 $5,640,000

Direct labor cost


Production hours Cost per hour Total cost
2,410,000 $9 $21,690,000
Each unit requires 15 minutes of direct labor time.

The manufacturing O.H rate is $4.40 per D.L.H ($10,604,000 ÷ 2,410,000 hours).
Variable overhead $5,790,000
Supervisory salaries 1,250,000
Depreciation 724,000
Other fixed cost 2,840,000
Total manufacturing overhead $10,604,000
Selling and administrative expenses
Selling expense $1,875,000
Administrative 3,080,000
Total expense $4,955,000
Each semi-annual mortgage payment consists of interest plus an even principal reduction of
$100,000. Interest payments for 2010 expected to be $250,000
Angelina binders Incorporation
Pro forma Statement of cash receipts and disbursements
For the year ending December 31, 2010 ($ 000 omitted)
Description Subtotal Total
Cash balance 1/1/2010 $565
Cash receipts
Cash sales 5,300
Collection of A/R 46,600
Proceeds from sale 20,000 C/S 420
Total Cash receipts 52,320
Total cash available 52,885
Cash disbursements
Raw materials $13,380
Direct labor 21,640
Manufacturing O.H. 9,650
Selling and administrative expenses 4,980
Income tax Paid 860
Purchase of equipment 1,200
Cash dividends 320
Mortgage payment 450
Total disbursement 52,480
Projected cash balance December, 31,2010 405

Pro forma statement of financial position as of December, 31, 2009


Current assets Current liabilities
Cash 565 Expenses payable 690
A/R 825 Mortgage payable 200
Raw material inventory 301* Income tax payable 356
Finished goods inventory 608** Total Current liabilities 1,246
Total Current assets 2,299 Long term Mortgage payable 2,700
Non - Current assets C/S (300,000 outstanding par $10) 3,000
Land 1,757 APIC 5,400
P.P.E 12,400 9,440 R.E. 1,150
Acc. Dep. (2,960)
Total assets 13,496 Total 13,496

* 65,000 ring assemblies at $0.80 each and 830,000 covers at $0.30 each
** 128,000 units at $ 4.75 each
Required: prepare pro-forma F/S for the year ended dec, 31, 2010
Pro-forma income statement for the year 2010
Description Subtotal Total
Projected sales 9,500 units × $5.50 52,250
Cost of goods manufactured
Beginning raw materials balance (Ring 65×.80 + cover 830 × 0.30) 301
Raw material purchases (Ring 7,680 + Cover 5,640) 13,320
Raw materials available for use 13,621
Ending raw materials inventory(25 ring ×.8 +350 cover ×.3 (125)
Raw materials used ( 9,640 ring × .8 ) + (9,640 cover × 2× $0.3 ) 13,496
Direct labor 2,410 ×$9 21,690
* Manufacturing O.H. $4.4 × 2,410 DLH 10,604
Costs of goods manufactured ( $4.75 ) 45,790
Add beg. Finished goods 128 × 4.75 608
Less ending Finished goods 268 ×$4.75 (1,273)
COGS 45,125
Gross profit 7,125
Selling and administrative exp. 4,955
EBIT 2,170
Less interest expense 250
EBT 1,920
Tax expense (40%) 768
Net income 1,152

Work sheet and pro – forma B/S


Description Beg. Balance Adjustments Ending bat.
DR. CR. DR. CR. DR. CR. DR. CR.
Cash 565 52,320 52,480 46,600 405
A/R 825 52,250 5,300 1,175
Raw material inv. 301 13,320 13,496 125
Finished goods inv. 608 45,790 46,398 1,273
Land 1,757 1,757
P.P.E 12,400 1,200 13,600
Acc. Dep. 2,960 724 (3,684)

Expenses payable 690 49,900 50,095 885


Mortgage payable 200 200
Income tax payable 356 768 860 264
Long-term mortgage 2,700 200 2,500
C/S 3,000 200 3,200
APIC 5,400 220 5,620
R.E 1,150 320 1,152 1,982
14,651 14,651

* Expenses payable = D.M. 7680 + 5640 + D.L 21690 + O.H. 10604 – 724 + S.A.E. 4,955 +
interest 250 = 50,092
* Expenses paid 13,380 + 21,640 + 9,650 + 4,980 + 250 = 49,900
Pro-forma Statement of cash flow –Indirect method
Net income 1,152,000
Add change in ACC.DEP due to DEP EXP 724,000
Less increase in A/R balance (350,000)
Add decrease in inventory –raw materials 176,000
Less increase in inventory – F.G (665,000)
Add increase in expense payable 195,000
Less decrease in income tax payable (92,000)
Net change in cash from operating activities (12,000)
Net cash inflow from operating activities 1,140,000

Purchase of equipment (1,200,000)


Net cash outflow from investing activities (1,200,000)

Cash proceeds from Issuance of 20,000 C/S 420,000


Cash outflow to pay mortgage payable (200,000)
Cash dividends (320,000)
Net cash outflow from financing activates (100,000)
Net change in cash balance from all sources (160,000)
Beginning cash balance 565,000
Ending cash balance 405,000

Pro-forma Statement of cash flow –Direct method


Cash collected from customers (825 + 52,250 -1,175) 51,900
Cash paid for materials, factory workers and factory services 44,695
providers (Beg pay 690 + COGS 45,125- Eng payables 885 +End
inv 125 +1,273 – Beg inv 301 -608-Dep Exp 724
Selling and administrative expenses paid 4,955
Interest expense paid 250
Income taxes paid (356 +768-264) 860
Total cash outflow from operating activities (50,760)
Net cash inflow from operating activities 1,140,000

Purchase of equipment (1,200,000)


Net cash outflow from investing activities (1,200,000)

Cash proceeds from Issuance of 20,000 C/S 420,000


Cash outflow to pay mortgage payable (200,000)
Cash dividends (320,000)
Net cash outflow from financing activates (100,000)
Net change in cash balance from all sources (160,000)
Beginning cash balance 565,000
Ending cash balance 405,000

Essay: Watson Corporation manufactures and sells extended keyboard units to be used with
microcomputers. Robin Halter, budget analyst, coordinated the preparation of the annual budget for the year
ending December 31, 2010. The budget was based on the prior year's sales and production activity. The pro
forma statements of income and COGS are presented below.

Watson Corporation Pro forma Statement of income


For the year ending December 31, 2010 ($000 omitted)
Net sales $25,550
Cost of goods sold (16,565)
Gross profit 8,985
Operating expenses
Marketing and selling expenses $3,200
General and administrative expenses 2,000
Total operating expenses (5,200)
Income from operations before income taxes 3,785
Watson Corporation Pro Forma Statement of Cost of Goods Sold
For the Year Ending December 31, 2010 ($000 omitted)
Direct materials
Beginning materials inventory 1,200
Materials purchases during 2010 11,400
Materials available for use 12,600
Ending materials inventory (1,480)
Direct materials consumed $11,120
Direct labor 980
Factory overhead:
Indirect materials 1,112
General factory overhead 2,800
Total factory overhead: 3,912
Cost of goods manufactured (COGM) 16,012
Beginning finished goods inventory 930
Cost of goods available for sale 16,942
Ending finished goods inventory 377
Cost of goods sold 16,565

On April 10, 2010, Halter met with Walter Collins, vice president of finance, to discuss the first
quarter's results (the period January 1 to March 31, 2010). After their discussion, Collins directed
Halter to reflect the following changes to the budget assumptions in revised pro forma statements.
• The estimated production in units for the fiscal year should be revised from 140,000 to 145,000 units
with the balance of production being scheduled in equal segments over the last nine months of the
year. The actual first quarter's production was 25,000 units.
• The planned inventory for finished goods of 3,300 units at the end of the fiscal year remains
unchanged and will be valued at the average manufacturing cost. The finished goods inventory of
9,300 units on January 1, 2010, had dropped to 9,000 units by March 31, 2010.
• Due to a new labor agreement, the labor rate will increase eight percent effective October 1, 2010,
the beginning of the fourth quarter, instead of the previously anticipated effective date of January 1,
2011, the beginning of the next fiscal year.
• The assumptions remain unchanged for D.M inventory at 16,000 units for the beginning inventory
and 18,500 units for the ending inventory. D.M inventory is valued on a first- in, first-out basis.
During the first quarter, D.M for 27,500 units of output was purchased for $2,200,000. Although D.M
will be purchased evenly for the last nine months, the cost of the D.M will increase by five percent on
July 1, 2010, the beginning of the third quarter.
• Indirect material costs will continue to be projected at ten percent of the cost of D.M consumed.
• One-half of general factory O.H and all of the marketing and general and administrative expenses are
considered fixed.
Required
A. Based on the revised data presented, calculate Watson Corporation’s projected sales for the year
ending December 31, 2010, in 1. Number of units to be sold 2. Dollar volume of net sales.
B. Prepare the pro forma Statement of Costs of Goods Sold for the year ending December 31, 2010.

Solution

♥Budgeted sales before revised data = beginning finished goods inventory 9,300 + production
140,000 units – ending finished goods inventory 3,300 =146,000 units
♥Budgeted selling price before revising data =$25,550,000 ÷146,000 units= $175 per unit

A.1) Based on the revised data presented: Adjusted projected sales for the year ending
December 31, 2010, in number of units to be sold =146,000 units + 5,000 units = 151,000 units

2) Based on the revised data presented: Adjusted projected sales for the year ending December
31, 2010, Dollar volume = 151,000 units' × $175 = $26,425,000

B. Based on the revised data presented, Watson Company's pro forma Statement of Costs of Goods
Sold for the year ending December 31, 2010, is presented below.

Watson Corporation
Revised Pro Forma Statement of Cost of Goods Sold
For the Year Ending December 31, 2010 ($000 omitted)
Direct materials
Beginning materials inventory (16,000 units) $1,200,000
Materials purchases during 2010* 12,120,000
Materials available for use 13,320,000
Ending materials inventory (18,500 units × $80 ×1.05% ) 1,554,000
Direct materials consumed 11,766,000
Direct labor** 1,037,400
Factory overhead
Indirect materials (10% of D.M consumed) 1,176,600
General factory overhead*** $2,850,000
Total factory overhead: 4,026,600
Cost of goods manufactured (COG M) for 145,000 units 16,830,000
Beginning finished goods inventory (9,300 units) 930,000
Cost of goods available for sale 17,760,000
Ending finished goods inventory**** ($379,830)
Cost of goods sold 17,380,170

Revised Pro forma income statement


Projected sales revenue (151,000 units × $175) 26,425,000
COGS 17,380,170
Gross margin 9,044,830
Marketing and selling expenses 3,200,000
General and administrative expenses 2,000,000
Total projected operating expenses 5,200,000
Earnings before tax (EBT) 3,844,830

Supporting calculations

D.M before data revising


Description Per unit cost Total cost
Beginning D.M inventory (16,000 units) $75 $1,200,000
Purchases (142,500 units) $80 $11,400,000
Total material available for use 12,600,000
Materials consumed FIFO (16,000 ×75) + (124,000 × 80) 11,120,000
Ending inventory (18,500 × $80) $80 (1,480,000)

Direct material purchased during first quarter = 27,500 units × $80 = $2,200,000

Quantity of Direct material to be purchased during the next three quarters = D.M to be used in
production 145,000 units + ending D.M inventory 18,500 units – (beginning D.M inventory 16,000
units + D.M purchased during first quarter 27,500 units = 120,000 units

Quarterly D.M purchases =120,000 units ÷ 3 quarter = 40,000 units

Cost of Direct material to be purchased during the next three quarters = 40,000 units × $80 +
(80,000 units ×$80 × 1.05%) =$9,920,000

Cost of materials purchases during 2010 = $2,200,000 + 9,920,000 =$12,120,000*

D.L cost per unit before data revising = $980,000 ÷ 140,000 units = $7 per unit

D.L cost for first quarter = 25,000 units ×$7 per unit =$175,000

D.L cost for second and third quarters = 40,000 units × 2 quarter × $7 per unit = $560,000

D.L cost for last quarter = 40,000 units ×$7 per unit ×1.08% =$302,400

Total D.L cost =$175,000 + $560,000 + $302,400 = $1,037,400**

General factory overhead = (fixed factory O.H 2,800,000 × 50%) + variable factory O.H
(1,400,000 ÷140,000 units) × 145,000 units = $2,850,000***

Manufacturing cost for the year = COGM $16,830,000 + beg F.G $930,000 =$17,760,000 ÷
(145,000 units produced + 9,300 units beg F.G) = $115.10

Ending finished goods inventory = 3,300 units ×$115.10 = $379,830****

Essay: Rein Company, a compressor manufacturer, is developing a budgeted income statement for the
calendar year 2010. The president is generally satisfied with the projected net income for 2009 of $700,000
resulting in EPS figure of $2.80. However, next year he would like EPS to increase to at least $3. Rein
Company employs a standard absorption cost system. Inflation necessitates an annual revision in the
standards as evidenced by an increase in production costs expected in 2010. The total standard
manufacturing cost for 2009 is $72 per unit produced.
Rein expects to sell 100,000 compressors at $110 each in the current year (2009). Forecasts from the sales
department are favorable, and Rein Company is projecting an annual increase of 10% in unit sales in 2010
and 2011. This increase in sales will occur even though a $15 increase in unit selling price will be
implemented in 2010. The selling price increase was absolutely essential to compensate for the increased
production costs and operating expenses. However, management is concerned that any additional sales price
increase would curtail the desired growth in volume.
Standard production costs are developed for the two primary metals used in the compressor (brass and a
steel alloy), the D.L, and manufacturing O.H. The following schedule represents the 2010 standard
quantities and rates for material and labor to produce one compressor.
Brass 4 pounds @ $5.35/pound $21.40
Steel alloy 5 pounds @ $3.16/pound $15.80
Direct labor 4 hours @ $7hour $28
Total prime costs $65.20
The material content of the compressor has been reduced slightly, hopefully without a noticeable decrease in
the quality of the finished product. Improved labor productivity and some increase in automation have
resulted in a decrease in labor hours per unit from 4.40 to 4 However, the significant increases in material
prices and hourly labor rates more than offset any savings from reduced input quantities. The manufacturing
O.H cost per unit schedule has yet to be completed. Preliminary data is as follows:

Overhead items Activity level (units)


100,000 110,000 120,000
Supplies $ 475,000 $ 522,500 $ 570,000
Indirect labor 530,000 583,000 636,000
Utilities 170,000 187,000 204,000
Maintenance 363,000 377,500 392,000
Taxes and insurance 87,000 87,000 87,000
Depreciation 421,000 421,000 421,000
Total overhead S2,046,000 $2,178,000 $2,310,000

The standard O.H rate is based upon DLH and is developed by using the total O.H costs from the above
schedule for the activity level closest to planned production. In developing the standards for the
manufacturing costs the following two assumptions were made.
• The cost of brass is currently selling at $5.65/pound. However, this price is historically high and the
purchasing manager expects the price to drop to the predetermined standard early in 2010.
• Several new employees will be hired for the production line in 2010. The employees will be generally
unskilled. If basic training programs are not effective and improved labor productivity is not experienced,
then the production time per unit of product will increase by 15 minutes over the 2010 standards.
•Rein employs a LIFO inventory system for its finished goods. Rein’s inventory policy for finished goods
is to have 15% of the expected annual unit sales for the coming year in finished goods inventory at the end
of the prior year. The finished goods inventory at December 31, 2009, is expected to consist of 16,500
units at a total carrying cost of $1,006,500.
•Operating expenses are classified as selling, which are variable, and administrative, which are all fixed.
The budgeted selling expenses are expected to average 12% of sales revenue in 2010 which is consistent
with the performance in 2009. The administrative expenses in 2010 are expected to be 20% higher than
the predicted 2009 amount of $907,850.
•Management accepts the cost standards developed by the production and accounting department.
However, they are concerned about the possible effect on net income if the price of brass does not
decrease, and/or the labor efficiency does not improve as expected. Therefore management wants the
budgeted income statement to be prepared using the standards as developed but to consider the worst
possible situation for 2010. Each resulting manufacturing variance should be separately identified and
added to or subtracted from budgeted COGS at standard. Rein is subject to a 45% income tax rate.

REQUIRED:

A. Prepare the budgeted income statement for 2010 for Rein Company as specified by management.
(Round all calculations to the nearest dollar)
B. Review the 2010 budgeted income statement prepared for Rein Company and discusses whether the
president’s objectives can be achieved.

Preliminary calculations
1. Number of C/S = $700,000 ÷ $2.8 = 250,000 C/S

2. Units to be produced during 2010

= [110,000 to be sold + ending inventory (110,000 units × 110% × 15%)] – (beginning


inventory (110,000 units ×15%) = 111,650 units of F.G

3. Using high-low point to segregate mixed costs to its components

Factory O.H item Activity level Fixed Variable


110,000
Supplies $ 522,500 - $ 522,500
Indirect labor 583,000 - 583,000
Utilities 187,000 - 187,000
Maintenance 377,500 218,000 159,500
Taxes and insurance 87,000 87,000 -
Depreciation 421,000 421,000 -
Total overhead $2,178,000 726,000 1,452,000
O.H rate per unit (based on 110,000 units) $6.6 $13.20
variable O.H rate per DLH $13.20/4 =$3.30
Predetermined single factory O.H allocation rate $2,178,000 ÷
(4 DLH ×
110,000 units)
= $4.95

4. per unit total standard manufacturing costs

Per unit total standard manufacturing costs (1)


Description Calculation Subtotal Total
D.M - brasses 4 units × $5.35 $21.4
D.M- steel 5 units × $3.16 $15.8
Total per unit D.M $37.20
D.L 4 DLH × $7 $28
Factory O.H ($2,178,000 ÷ 440,000 DLH) × 4 DLH $19.80
Total per unit standard manufacturing costs $85

Rein Company budgeted income statement


For the Year Ended December 31, 2010
Projected Sales (100,000 units × 110% × $125 13,750,000
Projected COGS (110,000 units × $85) 9,350,000
Gross profit 4,400,000
Total variances (1) $410,589 U
Adjusted Gross profit 3,989,411
Selling expense (13,750,000 ×12%) 1,650,000
Administrative expenses (907,850×120%) 1,089,420
Total operating expenses 2,739,420
Income before taxes 1,249,991
Income tax expense (45%) 562,496
Net income 687,495
Number of C/S 250,000
EPS $2.75

Variances (1)
Cost item Calculation Amount
D.M brasses ; price $0.30 × 4 brasses × 111,650 units $133,980 U
variance
D.L – efficiency variance 1/4 DLH × $7× 111,650 units $195,388 U
Variable Factory O.H $3.3 V.O.H per DLH×0.25 DLH = $0.825 $92,111 U
efficiency variance $0.825×111,650 units = $92,111
Fixed overhead volume ($726,000 ÷110,000 units) = $6.6 fixed O.H per unit $10,890 F
variance (111,650 units -110,000 units) ×$6.6 fixed O.H per
unit = $10,890
Total variances $410,589 U

B. Based upon the results of the 2010 budgeted income statement, the president’s objective can't be
achieved. A review of the statement highlights the following circumstances.
• 2010 income statement prepared using the worst situation gives a net income of $687,495 and EPS of
$2.75 which is a decrease from the 2009 income of $700,000 and EPS of $2.80. These budgeted figures are
considerably below the president’s objective of $750,000 net income and $3 EPS.
• If the unfavorable variances do not occur, net income will increase by $231,813 after taxes ($421,479 ×
55%) resulting in an increase in EPS of $0 .927, giving a total EPS of $3.677 which is well above the
president’s objective.
• Manufacturing costs were 65.50% of the selling price ($72 / $110) in 2009 and 71% of the selling price in
2010 after manufacturing cost increases ($88.775/$125). Administrative expenses increased 20% in 2010.
Therefore, the 13.60% sales price increase in 2010 was not sufficient to cover the increases in
manufacturing cost and increases in administrative expense.

Essay: As part of its cost control program, Tracer Company uses a standard cost system for all
manufactured items. The standard cost for each item is established at the beginning of the fiscal year, and
the standards are not revised until the beginning of the next fiscal year. Changes in costs, caused during the
year by changes in material or labor inputs or by changes in the manufacturing process, are recognized as
they occur by the inclusion of planned variances in Tracers monthly operating budgets.
Presented below is the labor standard that was established for one of Tracer’s products, effective January 1,
2010, the beginning of the fiscal year.
Assembler A labor (5 hours @ $10/hour) $50
Assembler B labor (3 hours @ $11 /hour) 33
Machinist labor {2 hours @ $15/hour) 30
Standard cost per 100 units 113
The standard was based on the labor being performed by a team consisting of five persons with assembler
A skills, three persons with assembler B skills, and two persons with machinist skills; this team
represents the most efficient use of the company's skilled employees. The standard also assumed that the
quality of material that had been used in prior years would be available for the coming year.
For the first seven months of the fiscal year, actual manufacturing costs at Tracer have been within the
standards established. However, the company has received a significant increase in orders, and there is
insufficient number of skilled workers available to meet the increased production. Therefore, beginning in
august, 2010, the production teams will consist of eight persons with assembler a skills, one person with
assembler B skills, and one person with machinist skills. The reorganized teams will work more slowly
than the normal teams; and, as a result, only 80 units will be produced in the same time period that 100 units
would normally be produced. Faulty work has never been a cause for units to be rejected in the final
inspection process, and it is not expected to be a cause for rejection with the reorganized teams.
Furthermore, Tracer has been notified by its material supplier that a lower quality material will be supplied
beginning July 1. Normally, one unit of raw material is required for each good unit produced, and no units
are lost due to defective material. Tracer estimates that 6 percent of the units manufactured after July 1 will
be rejected in the final inspection process due to defective material.
Required
A. Determine the number of units of lower quality material that Tracer Company must enter into production
in order to produce 35,720 good finished units.
B. Without prejudice to your answer in requirement A, assume that Tracer Company must manufacture a
total of 50,000 units in august to have sufficient good units to fill the orders received.
1) Determine how many hours of each class of labor will be needed to manufacture a total of 50,000 units in
august.
2) Determine the amount that should be included in Tracers august operating budget for the planned labor
variance caused by the reorganization of the labor teams and the lower quality material, and indicate how
much of the planned variance can be attributed to (a) the change in material and (b) the reorganization of the
labor teams. Be sure to support your answer with appropriate calculations.
Solution
A. Tracer Company must put 38,000 units of lower quality material into production in order to produce
35,720 good finished units, calculated as follows.
Material units required = [(35,720 good finished units) ÷ (1 - Rejection rate)]
= [(35,720 good finished units) ÷ (0.94)] = 38,000 input units

B. 1) Number of reorganized teams needed to produce 50,000 units = [50,000 units ÷ 80 unit
per team] = 625 teams
Each team takes 10 hours to produce 80 units, thus total hours needed to produce the 50,000
units = 625 teams × 10 hours = 6,250 hours
Tracer Company must utilize 5,000 assembler A hours, 625 assembler B hours, and 625 machinist
hours calculated as follows:
• New team = 8 assembler A, 1 assembler B, 1 machinist
Description % Time per 50,000 units*
Assembler A 80% 5,000
Assembler B 10% 625
Machinist 10% 625
Total 100% 6,250 DLH
2) Tracer Company should include an additional $13,140 in its operating budget for the planned labor
variance. This variance consists of $3,390 for the change in material and $9,750 for the labor change caused
by the reduced efficiency of the new team, calculated as follows.
1. D.L Cost to produce 50,000 units based on old team = [50,000 units ÷100 units per team] × $113=
$56,500
2. D.L Cost to produce 50,000 units based on new inefficient team = [50,000 units ÷ 80 units per team] ×
$106* = $66,250
* [(8 hours × $10) + (1 hour × $11) + (1 hour × $15)] = $106
3. Variance due to reduced efficiency of the new team = $66,250 - $56,500 = $9,750 U
4. Variance in D.L cost due to change in material is determined as follows:
A. Number of defective units = 50,000 units × 6% = 3,000 defective units
B. Number of teams required to produce 3,000 units based on old efficient team = (3,000 units ÷ 100
unit per team) = 30 teams
C. Cost of teams = 30 teams × $113 = $3,390 U
5. Total additional planned labor variance = $9,750 + $3,390 = $13,140 U

Q: A Polk retailer is developing cash and other budget information for July august and September.
At June 30 Polk had cash of $6,600 accounts receivable of $524,000 inventories of $371,280 and
accounts payable of $159,666. The budget is to be based on the following assumptions.

Sales

Each month’s sales are billed on the last day of the month, Customers are allowed a 2% discount if payment
is made within 10 days after the billing date. Receivables are booked gross. 65% of the billings are collected
within the discount period. 20% are collected by the end of the month. 10% are collected by the end of the
second month. And 5% prove uncollectible.

Purchases
-Each month’s ending inventory in units is equal to 120% of the next month’s units of sales. The
cost of each unit of inventory is $25.
-Selling, General, and administrative expenses, of which $3,000 is depreciation, are equal to 20%
of the current month’s sales.
-60% of all purchases of materials and selling, general, and administrative expenses are paid in the
month of purchase and the remainder in the following month.
Actual and projected sales are as follows

Description May June July August September October


Sales – units 10,600 10,900 10,700 10,200 10,800 11,000
Sales – Dollars 424,000 436,000 428,000 408,000 432,000 440,000

Schedule of cash collection and disbursement

Description May June July August September October


Sales – units 10,600 10,900 10,700 10,200 10,800 11,000
Sales – Dollars 424,000 436,000 428,000 408,000 432,000 440,000
Collection
Second month (65% × 98%) 354,888 364,932 358,236 341,496 361,584
+ 20% = 83.7%
Third month 10% 42,400 43,600 42,800 40,800
Total collections 407,332 401,836 384,296 402,384
Month’s sales – units 10,600 10,900 10,700 10,200 10,800 11,000
Beg inv.-units -120% of the 12,720 13,080 12,840 12,240 12,960 13,200
next month sales-units
End inv. -units 13,080 12,840 12,240 12,960 13,200 ?
Quantity to be purchased- 10,960 10,660 10,100 10,920 11,040 ?
units
Cost of purchases 274,000 266,500 252,500 273,000 276,000 ?

S, A EXP (20% of sales 81,800 84,200 82,600 78,600 83,400 85,000


minus $3,000 )
Total costs and EXP 355,800 350,700 335,100 351,600 359,400 ?
Payments during month 213,480 210,420 201,060 210,960 215,640 ?
60%
Payments next month 40% ? 142,320 140,280 134,040 140,640 143,760
Total payments 352,740 341,340 345,000 356,280 ?

Essay: CrossMan Corporation, a rapidly expanding crossbow distributor to retail outlets, is in the process
of formulating plans for 2010. Joan Caldwell, director of marketing, has completed her 2010 forecast and
is confident that sales estimates will be met or exceeded. The following sales figures show the growth
expected and will provide the planning basis for other corporate departments.

Month Forecasted Sales Month Forecasted Sales


January $1,800,000 July $3,000,000
February 2,000,000 August 3,000,000
March 1,800,000 September 3,200,000
April 2,200,000 October 3,200,000
May 2,500,000 November 3,000,000
June 2,800,000 December 3,400,000

George Brownell, assistant controller, has been given the responsibility for formulating the cash flow
projection, a critical element during a period of rapid expansion. The following information will be
used in preparing the cash analysis.
• CrossMan has experienced an excellent record in accounts receivable collection and expects this trend to
continue. Sixty percent of billings are collected in the month after the sale and 40 percent in the second
month after the sale. Uncollectible accounts are nominal and will not be considered in the analysis.
• The purchase of the crossbows is CrossMan’s largest expenditure; the cost of these items equals 50
percent of sales. Sixty percent of the crossbows are received one month prior to sale and 40 percent are
received during the month of sale.
• Prior experience shows that 80 percent of accounts payable are paid by CrossMan one month after
receipt of the purchased crossbows, and the remaining 20 percent are paid the second month after receipt.
• Hourly wages, including fringe benefits, are a factor of sales volume and are equal to 20 percent of the
current month's sales. These wages are paid in the month incurred.
• General and administrative expenses are projected to be $2,640,000 for 2010. The composition of the
expenses is given below. All of these expenses are incurred uniformly throughout the year except the
property taxes. Property taxes are paid in four equal installments in the last month of each quarter.

Description Amount
Salaries $480,000
Promotion 660,000
Property taxes 240,000
Insurance 360,000
Utilities 300,000
Depreciation 600,000
Total $2,640,000

• Income tax payments are made by CrossMan in the first month of each quarter based on the income for
the prior quarter. CrossMan's income tax rate is 40 percent. CrossMan’s net income for the first quarter of
2010 is projected to be $612,000.
• CrossMan has a corporate policy of maintaining an end-of-month cash balance of $100,000. Cash is
invested or borrowed monthly, as necessary, to maintain this balance.
• CrossMan uses a calendar year reporting period.

Required:
A. Prepare a Pro Forma income statement for CrossMan Corporation, by quarter,
B. Prepare a Pro Forma Schedule of Cash Receipts and Disbursements for CrossMan Corporation,
by month, for the second quarter of 2010. Be sure that all receipts, disbursements, and
borrowing/investing amounts are presented on a monthly basis. Ignore the interest expense and/or
interest income associated with the borrowing/ investing activities
C. Discuss why cash budgeting is particularly important for a rapidly expanding company such as
CrossMan Corporation.

Solution:
A. Pro Forma income statement for CrossMan Corporation, by quarter,

Crossman Corporation Pro Forma income statement


for the year 2010
Description Q1 Q2 Q3 Q4 Total
Projected sales 5,600,000 7,500,000 9,200,000 9,600,000 31,900,000
COGS (50%) 2,800,000 3,750,000 4,600,000 4,800,000 15,950,000
Gross profit 2,800,000 3,750,000 4,600,000 4,800,000 15,950,000
Operating expenses
Hourly wages (20% of 1,120,000 1,500,000 1,840,000 1,920,000 6,380,000
sales
Property taxes 60,000 60,000 60,000 60,000 240,000
Salaries 120,000 120,000 120,000 120,000 $480,000
Promotion 165,000 165,000 165,000 165,000 660,000
Insurance 90,000 90,000 90,000 90,000 360,000
Utilities 75,000 75,000 75,000 75,000 300,000
Depreciation 150,000 150,000 150,000 150,000 600,000
Total operating expenses 1,780,000 2,160,000 2,500,000 2,580,000 9,020,000
EBIT 1,020,000 1,590,000 2,100,000 2,220,000 6,930,000
Income taxes (40%) 408,000 636,000 840,000 888,000 2,772,000
Projected net income 612,000 954,000 1,260,000 1,332,000 4,158,000

B. A Pro Forma Schedule of Cash Receipts and Disbursements for CrossMan Corporation, by
month, for the second quarter of 2010, is presented below. (For Pro Forma Schedule purposes,
interest expenses and/or interest income associated with the borrowing/investing activities have not
been considered.)

CrossMan corporation pro forma schedule of cash receipts and disbursements


Second quarter 2010 (in thousands)
Description April May June
Beginning cash balance 100,000 100,000 100,000
A/R collections
Collections from February sales 800,000
Collections from March sales 1,080,000 720,000
Collections from April sales 1,320,000 880,000
Collections from May sales 1,500,000
Total 1,880,000 2,040,000 2,380,000
Total cash available 1,980,000 2,140,000 2,480,000
Cash payments for purchases* 1,004,000 1,156,000 1,310,000
Hourly wages 20% of sales 440,000 500,000 560,000
Property taxes - - 60,000
Other monthly cash operating exp. 150,000 150,000 150,000
Income taxes 408,000
Total expected cash outflows 2,002,000 1,806,000 2,080,000
Cash balance (22,000) 334,000 400,000
Cash deficit-borrowed 122,000
Cash surplus-invested 234,000 300,000
Ending cash balance 100,000 100,000 100,000

Supporting calculation of Cash payments for purchases*


Description February March April May June July
Cost of purchases (50%) 1,000,000 900,000 1,100,000 1,250,000 1,400,000 1,500,000
Purchases -60% 540,000 660,000 750,000 840,000 900,000 900,000
Purchases -40% 400,000 360,000 440,000 500,000 560,000 -
Total purchases 940,000 1,020,000 1,190,000 1,340,000 1,460,000
Payments for material purchases
80% second month 752,000 816,000 952,000 1,072,000 1,168,000
20% second month 188,000 204,000 238,000 268,000
Cash payments for purchases* 1,004,000 1,156,000 1,310,000

See Gleim's MCQ's # 162, 180, 181,182,183,184,185,186,187, 228,229, 230,231,232,233,234,


246, 247,248,249,250,251, 252,253,254, 279, 280, 281, 282,283,284,285,

Creating pro forma F/S using the % of sales method

- % of sales method is a short-cut simple approach to prepare pro forma F/S

- It assumes that the relationship between I/S and B/S items and sales revenue remains constant, which means
that they grow proportionally with sales growth. Any activity that is needed to directly support the operations
that will generate the specific sales level is assumed to grow proportionately with sales. On the I/S, these items
include COGS and selling and administrative expense (S&A); on the B/S, it includes current assets, net fixed
assets, A/P, and accruals. But none of the financing activities, namely, N/P (short-term borrowing), LTD, and
owners’ equity are assumed to grow proportionately with sales.

Example: Heavenly Furniture Company uses the % of sales method to forecast its pro forma F/S. Based on
historical F/S, the following relationships can be established between several items and sales:
Description Current year
Sales $100,000
COGS (80%) 80,000
Gross Margin 20,000
S&A Expenses (5%) 5,000
EBIT (Operating Income) 15,000
Interest expense 1,800
EBT 13,200
Income tax expense (35%) 4,620
Net income $8,580
EPS (based on 12,000 WA# of C/S) $0.715
Dividends (40%) $3,432
Addition to retained earnings $5,148

Assets Current Year


Cash and cash equivalents 3% 3,000
Receivables 18% 18,000
Inventories 25% 25,000
Total current assets 46,000
Net fixed assets 35% 35,000
Total assets 81,000
Liabilities and equity
Accounts payable 12% 12,000
Accruals 8% 8,000
Notes payable 5,000
Total current liabilities 25,000
Long term debt 20,000
Total liabilities 45,000
Common stock 20,000
Retained earnings 16,000
Total equity 36,000
Total liabilities and equity 81,000

Heavenly Furniture is projecting a sales revenue growth of 18% in the upcoming year. The company plans
on maintaining its dividend policy of paying out 40% of its net income as dividends. It is currently paying
8% interest on its notes payable and 10% on its long-term debt. Interest expense is calculated based on its
beginning-of-period debt balance. Additional external funding to support the projected sales growth would
be via notes payables
Using the % of sales method
1. Create the pro forma income statement
2. Create the pro forma balance sheet.
3. Create the pro forma statement of cash flows.

Creating a Pro forma income Statement

Current Year Upcoming Year


Description
Sales revenue $100,000 (100,000 ×1.18%) $118,000
COGS 80,000 (118,000 ×80 %) 94,400
Gross Margin 20,000 23,600
S&A Expenses 5,000. (118,000 ×5 %).5,900
EBIT (Operating Income) 15,000 17,700
Interest expense 1,800 (5,000 × 8 %) + (20,000 ×10 %) 2,400
EBT 13,200 15,300
Income tax expense (35%) 4,620 5,355
Net Income $8,580 $9,945
EPS $0.72 ($9,945÷12,000 C/S) $0.83
Dividends (40%) $3,432 ($9,945×40 %) $3,978
Addition to retained earnings $5,148 $5,967

Creating a Pro forma balance sheet

Assets Current Year Upcoming Year


Cash and cash equivalents 3,000 (118,000 ×3 %) 3,540
Receivables 18,000 (118,000 ×18 %) 21,240
Inventories 25,000 (118,000 ×25 %) 29,500
Total current assets 46,000 54,280
Net fixed assets 35,000 (118,000 ×35 %) 41,300
Total assets 81,000 95,580
Liabilities and equity
Accounts payable 12,000 (118,000 ×12%) 14,160
Accruals 8,000 (118,000 ×8%) 9,440
Notes payable 5,000 (5,000 + 5,013**) 10,013
Total current liabilities 25,000 33,613
Long term debt 20,000 20,000
Total liabilities 45,000 53,613
Common stock 20,000 20,000
Retained earnings 16,000 (16,000 +$5,967) 21,967
Total equity 36,000 41,967
Total liabilities and equity 81,000 95,580

** Needed external financing via N/P = $18,000 × (81% - 20%) - $5,967 = $5,013

**Heavenly Furniture Company has several options to finance the $5,013 of external funding it needs. The
company can raise the funds using N/P, LTD, C/S, or any combination of the three. However, if the company
is unwilling to seek that much external funding, it can lower its external funding need by increasing the
amount of internal funding (i.e., R.E) available by altering its dividend policy. The company can lower its
dividend payout ratio (e.g., from 40% to 30%) to reduce the amount of dividends paid out and to increase the
amount of earnings retained.

Creating a pro forma statement of cash flows

Net income $9,945


Increase in Receivables (3,240)
Increase in Inventories (4,500)
Increase in Payables 2,160
Increase in Accruals 1,440
Net Cash inflow from operating activities $5,805

New assets acquisitions (cash outflow on investing activities) (6,300)

Increase in Notes payables $5,013


Dividends (3,978)
Net Cash inflow from financing activities $1,035
Net change in cash flow $540
Beginning cash 3,000
Ending cash $3,540

♥Once a company creates its pro forma F/S, the statements need to be analyzed to determine if the
company is meeting its pre-determined financial targets. This can be accomplished by calculating a
variety of financial ratios and comparing them to pre-determined targets and industry averages.
These calculations can help answer questions such as:
• is the company’s leverage (as measured by its debt ratio) within an acceptable range?
• Is its return on equity (ROE) acceptable in relation to the industry average?

Current Upcoming
Ratio Year Year

Current ratio [C.A/C.L] Liquidity measure 1.84:1 1.615:1▼


Quick ratio (Acid test ratio) [Quick assets /C.L] Liquidity measure 0.84:1 0.737:1▼
Return on asset [NI/ Average total assets] (ROA) Return measure 10.60% 10.40%▼
Return on equity [NI/ Average total C.E] (ROE) Return measure 23.83 % 23.70%▼
Gross profit margin (G.P/Sales) Return measure 20% 20%
Operating profit margin (EBIT/Sales) Return measure 15% 15%
Net profit margin (NI/Sales) Return measure 8.58% 8.43%▼
Debt ratio (Total debt/Total assets) Risk measure 55.56% 56.09%▲
Time interest earned ratio (interest coverage ratio) (EBIT/I) 8.3333 times 7.375 times▼
Risk measure
Basic EPS (NI- current year P/S) ÷WA# of C/S) Return measure $0.715 $0.83▲

Comments

1. (LIQUIDITY) in order to support a projected 18% sales growth, Heavenly Furniture Company is
raising $5,013 of external funding via N/P. Because the company is depending on short-term borrowing
for its external funding needs, its liquidity position will deteriorate as the current ratio and quick ratio
are expected to decrease in the upcoming year.

2. (CREDITOR'S RISK) Because the company is relying on N/P to rise its needed external funding, it is
important that it pay close attention to its debt ratio and interest coverage (in terms of time interest earned)
to ensure that it has not violated its debt covenants. The company’s debt ratio is expected to increase from
55.56% to 56.09% and its interest coverage ratio is expected to decrease from 8.333 to 7.3750. Both of these
changes are relatively minor, so the company should be able to meet its debt covenant.
3. (RETURN MEASURES) its profit position, in terms of return on asset (ROA), return on equity
(ROE), and net profit margin are also expected to decrease in the upcoming year.

4. EPS are expected to increase from $0.715 to $0.83 (about 16% increases) as a result of the sales growth
that is funded by additional debt.

Q1: B.G. Barns, Inc. uses the % of sales method to forecast its pro forma F/S. Based on historical F/S, the
following relationships can be established between several items and sales:
COGS is 65% of sales
Selling and administrative (S&A) Expense is 15% of sales
Cash and equivalents is 6% of sales
Accounts receivable is 8% of sales
Inventories are 24% of sales
Net fixed assets are 32% of sales
Accounts payable is 15% of sales
Accruals are 10% of sales
B.G. Barns is projecting a sales revenue growth of 13% in the upcoming year. The company currently has
20,000 shares of C/S outstanding par value $1 and it plans on maintaining its dividend policy of paying
out 30% of its net income as dividends. It is currently paying 6% interest on its N/P ($3,000 note) and 8%
on its LTD ($17,500 bond). The company has a 40% tax rate.
Based on the above information, assume current year sales are $125,000, any additional financing needs is
to be arranged at year end through N/P while any excess is used to redeem N/P first and a portion of LTD.

Creating a pro forma income statement

Current year Upcoming year


Description
Sales revenue $125,000 125,000 ×113%= $141,250
COGS 65% 81,250 $141,250 × 65% = 91,812.50
Gross Margin 43,750 49,437.50
S&A Expenses 15% 18,750 $141,250 × 15% =21,187.50
EBIT 25,000 28,250
(3,000 × 6%) + (17,500 × 8%) = (3,000 × 6%) + (17,500 × 8%)
Interest expense
1,580 = 1,580
EBT 23,420 26,670
Income tax expense (40%) 9,368 10,668
Net Income 14,052 16,002
EPS (20,000 C/S) 0.7026 0.80
Dividends (30%) 4,215.60 4,800.60
Addition to R.E 9,836.40 11,201.40

Creating a pro forma balance sheet

Assets Current Year Upcoming Year


Cash and cash equivalents 6% $7,500 $8,475
Receivables 8% 10,000 11,300
Inventories 24% 30,000 33,900
Total current assets 47,500 53,675
Net fixed assets 32% 40,000 45,200
Total assets 87,500 98,875
Liabilities and equity
Accounts payable 15% 18,750 21,187.50
Accruals 10% 12,500 14,125
Notes payable 6% interest rate 3,000 0
Total current liabilities 34,250 35,312.50
Long term debt 8% Coupon. Rate 17,500 17,500 - 888.90 =16,611.10
Total liabilities 51,750 51,923.60
Common stock 20,000 C/S 20,000 20,000
Retained earnings 15,750 26,951.40
Total equity 35,750 46,951.40
Total liabilities and equity 87,500 98,875

** Excess = $16,250 × (70% - 25%) - $11,201.40= $3,888.90 used to redeem N/P and partially
reduce LTD

Fact pattern: The balance sheet and income statement of the Grow 'n' Glow Manufacturing
Company during the past year are as follows (000 omitted):

Assets Liabilities
Cash $9,700 A/P 3% $3,000
A/R 15,300 N/P (interest 5%) 10,000
Inventory 18,500 Accrued liabilities 6% 6,000
Total current assets 43.50% 43,500 Total current liabilities 19,000
Held to maturity securities (earn 4%) 45,600 LTD (interest 7%) 35,600
Net fixed assets 32,200 Total liabilities 54,600
Total long term assets 77,800 Equity
C/S 10,000
APIC 30,000
R.E 26,700
Total equity 66,700
Total assets 121,300 Total 121,300

Income statement
Net sales $100,000
COGS 66,200
Gross profit 33,800
Selling expenses 14,600
General and administrative expenses 11,200
Operating income 6,200
Net interest expense 1,200
Net income before tax 5,000
Income tax expense (35%) 1,750
Net income 3,250

The company paid dividends during the past year of $975. During the past year, fixed assets were
being used at 85% of capacity. In all other respects, the company was operating at full capacity.
Assuming the company's dividend policy is that dividends will grow at a rate of 4% per year, the
past year's interest rate on debt was 5% on short-term debt and 7% on long-term debt. The held-to-
maturity securities earn 4% return and are not expected to change next year.

Q2: By what percentage could next year's a sales increase over the past year's sales without the
company needing to increase its fixed assets?
A. 15% B. 27.4% C. 67.8% D. 17.6%
$100,000
Answer (D) is correct: -$100,000 = $17,647
85%
$17 ,647
=17.646%
$100 ,000
Therefore, sales could grow at 17.60% before the company would need to purchase additional
fixed assets.

Q3: A profitable firm that is experiencing rapid sales growth will find that its need for external
financing will
A. Increase, because in order to increase sales, the firm must decrease its prices, which will lead to
decreased profits and decreased cash.
B. Decrease, because the higher sales will lead to higher profits, and the added profits will provide more
cash.
C. Decrease, because more inventories will be sold and the decrease in inventory will generate
additional cash.
D. Increase, because of its need for additional investment in working capital and fixed assets to support
the increased sales.
Answer (A) is incorrect because a firm in a high-growth stage does not necessarily need to decrease its
prices in order to keep sales growing, so this is an incorrect assumption.
Answer (B) is incorrect because higher sales may lead to higher profits; but increased cash from higher
profits is not the only effect the higher sales will have on cash.
Answer (C) is incorrect because while it is true that more inventories will be sold when sales increase,
more inventories will also need to be manufactured or purchased, not only to replace the inventory that
has been sold but also to have enough inventory on hand to support the increased sales. So the increased
cash generated by increased inventory sales will be offset by the need to acquire more inventories.
Answer (D) is correct: the higher the firm's rate of growth in sales, the greater will be its need for
additional financing. When sales increase, firms usually need to purchase more assets to support the
increased level of sales. More inventories will need to be purchased, and additional equipment will be
needed to expand manufacturing.
Additional employees will be required to operate the new equipment and also to make and process the
additional sales and to provide customer service after the sale. Some of the required funding can be
provided by increases in profits and increases in accounts payable and accruals (called "spontaneous
liabilities"); but not all of the required funding can be provided in that way. The company will need to
Rise additional external financing either in the form of borrowed funds or in the form of new equity.

Q4: A company's need for external financing depends on several factors. A factor that does not affect
the company's need for external financing is
A. The company's retention ratio B. The company's profit margin
C. Rapid sales growth. D. The company's spontaneous assets-to-sales ratio.
Answer (A) is incorrect because a company's retention ratio is a factor that affects the company's need
for external financing. The retention ratio is how much of the company's net income it does not pay out
in dividends and thus retains in the company. The retention ratio is calculated as (net income - dividends
paid) ÷ Net income. Companies that pay out less of their net income in dividends have a higher retention
ratio. The higher the retention ratio, the less need the company will have for external financing.
Companies with smaller retention ratios will have a greater need for additional funds.
Answer (B) is incorrect because a company's profit margin is a factor that affects the company's need for
external financing. The higher the company's profit margin is, the more net income will be available to
fund increases in assets, and the less need the company will have for external financing.
Answer (C) is incorrect because a rapid sales growth is a factor that affects a company's need for
external financing. Rapid sales growth requires increases in assets, and increases in assets generate the
need for external financing. The higher the growth rate in sales, the greater will be the need for
additional financing.
Answer (D) is correct: It is the company's spontaneous liabilities-to-sales ratio that affects its need for
external financing. A company with a higher spontaneous liabilities-to-sales ratio will have less need for
external financing, because it can finance more of its working capital needs by using increases in A/P
and accrued liabilities.

Q5: Increases in sales generally cause spontaneous increases in some liability and net worth lines on the
balance sheet. The liability and net worth items that increase spontaneously with increases in sales
include all of the following except
A. Notes payable. B. Retained earnings.
C. Accrued salaries and wages. D. Accounts payable.
Answer (A) is correct: Borrowed funds such as N/P do not increase spontaneously with increases in
sales. The firm must make an effort to cause its borrowings to increase, by making intentional
arrangements to borrow the funds.
Answer (B) is incorrect because R.E will increase spontaneously with increases in sales because as sales
increase, net income also generally increases. The increase in net income will lead to a higher balance in
the R.E account in the equity section of the balance sheet.
Answer (C) is incorrect because accrued salaries and wages will increase spontaneously with increases
in sales because as sales increase, additional employees will need to be hired to support the increased
sales. The increase in staff will naturally lead to a higher level of accrued salaries and wages.
Answer (D) is incorrect because A/P will increase spontaneously with increases in sales because as sales
increase, so will purchases of direct materials (for a manufacturer) or of inventory (for a reseller). These
additional purchases will naturally lead to a higher level of A/P.

Q6: Which of the following events will cause a company's requirements for external financing to
increase?
I. The dividend payout ratio increases.
II. The company changes its credit terms, increasing the time it gives customers to pay.
III. The company negotiates a lower price and longer terms with a major supplier.
IV. The retention ratio increases.
V. Increased competition forces the company to lower its prices.
A. II and V. B. III and IV. C. I, II, and V. D. II, IV and V.
Answer (A) is incorrect because these will cause the company's requirements for external financing
to increase, but they are not the only events from among those listed that will have that effect.
Answer (B) is incorrect because negotiating a lower price and longer terms with a major supplier and
increasing its retention ratio will both cause the company's requirements for external financing to
decrease, not increase.
Answer (C) is correct: These events will all cause the company's requirements for external
financing to increase. When the dividend payout ratio increases, it means the company is paying out
more of its net income in dividends and so it will have less R.E and cash available. When the
company changes its credit terms to increase the time it gives customers to pay, this will cause A/R
to increase and collections and cash to decrease. Lower prices will decrease the profit margin and
will decrease collections from sales, which will result in less cash available.
Answer (D) is incorrect because increasing the retention ratio will cause the company's requirements
for external financing to decrease, not increase.

Q7: A company had $500,000 of sales for the year just ended and is projecting sales of $600,000
for the coming year. For every $1 increase in sales, 38 cents of additional financing is required
for the purchase of additional assets. The projected profit margin is 20%, and 60% of profits will
be retained for reinvestment in the company. The amount of additional external financing needed
by the company in the coming year is
A. $26,000 B. $0 C. $38,000 D. $110,000 E. $64,000
Answer (B) is correct: Because of the expected increase in sales of $100,000, the company will need an
additional $38,000 of financing. The company reinvests 60 of the profits of the company. Profits are
expected to be $120,000 (20% of the $600,000 of sales). Of this profit, 60%, or $72,000, will be
reinvested and is available for financing. Since the amount that will be available from inside the
company is larger than the amount needed, the company will not need any additional external financing.

Q8: A firm's balance sheet as of December 31 is shown below. The firm's sales for the year were
$1,000,000,000 and its after-tax margin on sales was 5%. Sales are expected to increase next year to
$1,300,000,000, and it plans to distribute 50% of its net profits to stockholders. Based on the % of sales
method, how much external financing need assuming the amount of funds that must be obtained
externally is financed through issuance of bonds.
Assets ($ millions) Liabilities ($ millions)
Cash $50 Accounts payable $30
Receivables 130 Accrued taxes & wages 40
Inventories 150 Mortgage bonds 130
Total current assets 330 C/S 250
Net fixed assets 220 R.E 100
Total 550 Total 550

♥Total assets must increase by $0.55 for each $1 increase in sales if assets are to be maintained at their
present % of sales ($550 million of total assets ÷ $1,000 million of sales). Accordingly total increase in
amount of assets needed to achieve the target sales = $300 increase in sales × $0.55 = $165 million

♥Because A/P and accruals account are 7% of current sales, ($70 million ÷$1,000 million) they should
finance the increased sales by $21 million ($300 million sales increase × $.07) these items rise
spontaneously with sales because higher sales require more purchases, accruals of wages and taxes, etc.

♥The remaining amount = $165 million - $21 million = $144 million must be generated in other ways,
such as by non- trade debt and equity.

♥Given an after-tax profit margin on sales of 5%, and 50% distribution rate thus the increase in R.E that
can finance portion of the remainder = $1,300 million in sales × 5% ×50% undistributed= $32.50 million

♥Hence, the external funds required = $144- $32.5 = $111.5 million


Total. Assets  Current.Liabilities
♥External funds required = Expected increase in sales × -
Current.Sales.Level
increase in R.E
$550  $70
♥External funds required = $300× - $1,300 million in sales × 5% ×50% = $111.5 million
$1,000

Pro-forma balance sheet


Cash 5% $50 1,300 × 5% = $65
Receivables 13% 130 1,300 × 13% =169
Inventories 15% 150 1,300 × 15% =195
Total current assets 33% 330 1,300 × 33% = 429
Net fixed assets 22% 220 1,300 × 22% =286
Total assets 550 715
Liabilities and equity
Accounts payable 3% $30 1,300 × 3% = 39
Accrued taxes & wages 4% 40 1,300 × 4% = 52
Total current liabilities 7% 70 1,300 × 7% = 91
Mortgage bonds 130 130 +111.5 = 241.5
Total liabilities 200 332.5
C/S 250 250
R.E 100 100 +32.5 = 132.5
Total equity 350 382.5
Total liabilities and equity 550 715

Q9: A firm's capital intensity ratio is


A. Its total long-term debt plus equity divided by total assets.
B. Its common equity divided by total liabilities.
C. Its assets that increase when sales increase divided by sales.
D. Its shareholders' equity divided by total assets.
Answers (A, B and D) are incorrect because the capital intensity ratio is the amount of assets required
per dollar of sales. The capital intensity ratio of a firm affects its capital requirements. A company with a
high assets-to-sales ratio will require more assets for a given increase in sales and therefore will have a
greater need for external financing than a company with a lower assets-to-sales ratio.
Answer (C) is correct: The capital intensity ratio is the amount of assets required per dollar of sales. It is
assets that increase when sales increase divided by sales. The capital intensity ratio of a firm affects its
capital requirements. A company with a high assets-to-sales ratio will require more assets for a given
increase in sales and therefore will have a greater need for external financing than a company with a
lower assets-to-sales ratio.

Underlying assumption for using the %-of-sales technique to develop pro forma F/S
1. Most balance sheet accounts are tied to sales (Except financing sources)
2. Assets are efficiently used i.e. Current level of all assets is optimal for the current sales level,
Thus assets-to- sales ratios are constant. This is based on the assumption that The firm is
operating at full capacity and no unnecessary amounts of current assets or fixed assets are held at
the current sales level, Thus, a greater amount of assets will be required for a greater amount of
sales (e.g.. receivables , inventories and cash requirements etc.).
3. Technological considerations enables that fixed assets can be added in small, non-discrete
amounts because, if fixed assets must be added in large, discrete amounts, the ratios are not
constant.
4. No Economies or diseconomies of scale exist in the use of assets. Because, if economies or
diseconomies of scale exist in the use of assets, the ratios of assets to sales are not constant
5. Actual sales-to-asset ratios for the firm in any given period will not vary with different stages
of the economic cycle. Because, if ratios vary with economic cycles, the %-of-sales method is
inappropriate
Benefits of pro forma F/S

Pro forma F/S helps a company plan for the future, and it supports three major functions.
1. Assess whether the anticipated performance is in line with its established targets.
2. Anticipate the amount of funding needed to achieve forecasted sales growth.
3. Estimate the effects of changes in assumptions of key numbers by performing sensitivity analysis
(i.e., what-if analysis). Sensitivity analysis helps to identify potential conditions that could lead to major
problems for the company. This enables the company to plan for appropriate actions in case such an
event should occur. In addition, sensitivity analysis also provides the company with the opportunity to
analyze the impact of changing its operating plans.

Using a contribution margin format that segregates costs in all categories into variable component
and fixed component allows the budget to be used for internal performance measurement and to
help show where costs can be controlled.

EXAMPLE: The following information relates to Clyde Corporation which produced and sold 50,000
units during a recent accounting period
Sales $650,000
Manufacturing costs
Fixed 210,000
Variable 140,000
Selling and administrative costs
Fixed 100,000
Variable 45,000
Income tax rate 40%
In the next accounting period, if production and sales are expected to be 40,000 units the company should
anticipate a per-unit contribution margin of
A. $13.30 B. $7.98 C. $3.10 D. $0.55 E. $9.30
Answer (E) is correct: U.C.M = per unit selling price $13 – ($2.80 per unit variable
manufacturing cost + $0.90 per unit variable non-manufacturing cost) = $9.30
Contribution income statement
Description Per unit Last period Next period
50,000 units 40,000 units
Sales $13 $650,000 520,000
Less variable manufacturing costs 2.80 140,000 112,000
Manufacturing C.M 10.20 510,000 408,000
Less variable Selling and administrative costs 0.90 45,000 36,000
C.M 9.30 465,000 372,000
Less Fixed Manufacturing costs 210,000 210,000
Less Fixed Selling and administrative costs 100,000 100,000
EBIT 155,000 62,000

Q: What is the most effective approach to a budgeting process?


A. Combination approach B. Non-qualitative approach
C. Participative approach D. Authoritative approach
Answer (A) is correct: In an authoritative budget (top-down budget), top management sets everything
from strategic goals down to the individual items of the budget for each department and expects lower
managers and employees to adhere to the budget and meet the goals. In a participative budget (bottom-
up or self-imposed budget), managers at all levels and certain key employees cooperate to set budgets
for their areas, and top management usually retains final approval. The ideal process combines the
features of each and falls somewhere between these methods.

Pro forma F/S in service companies


Essay: Don Masters and two of his colleagues are considering opining a law office that would make
inexpensive legal services available to those who could not otherwise afford these services. The intent is to
provide easy access for their clients by having the office open 360 day per year, 16 hours each day from 7:00
am to 11:00 p.m. the office would be staffed by a lawyer, paralegal, legal secretary, and clerk-receptionist for
each of the two 8-hour shift. To determine the feasibility of the project, Masters hired a marketing consultant
to assist with market projections. The result of this study show that if the firm spend $0.50 million on
advertising the first year, the number of the new clients expected each day would have the following
probability distribution.
Number of new clients per day 20 30 55 85
Probability 0.10 0.30 0.40 0.20

Masters and his associates believe these numbers are reasonable and are prepared to spend the $500,000 on
advertising. Other information about the operation of the office is given below. The only charge to each new
client would be $30 for the initial consultation. All cases that warranted further legal work would be accepted
on a contingency basis with the firm earning 30% of any favorable judgment. Master estimates the 20% of
new client consultation will result in favorable statements judgments $2,000 each. It is not expected that there
will be repeat clients during the first year of operations. The hourly wages of the staff are projected to be $25
for the lawyer, $20 for the paralegal, $15 for the legal secretary, and $10 for the clerk-receptionist. Fringe
benefit expense will be 40% of the wage paid. A total of 400 hours of overtime is expected for the year; this
will be divided equally between the legal secretary and the clerk-receptionist positions. Overtime will be paid
at one and half times the regular wage and the fringe benefit expense will apply to the full wage. Masters
has located 6,000 square feet of suitable office space which rents for $28 per square foot annually. Associated
expenses will be $22,000 for property insurance and $32,000 for utilities. It will be necessary for the group to
purchase malpractice insurance, which is expected to cost $180,000 annually. The initial investment in office
equipment will be $60,000; this equipment has an estimated useful life of 4 year. The cost of office supplies
has been estimated to be $4 per expected new client consultation.
Required: prepare the pro-forma I/S for the first year of operations and determine even-point
Solution:
Cost structure  First year
Description Fixed Variable cost per client
Advertising $500,000
Salaries, overtime and fringe benefits $574,980
Rent (6,000 × $28) $168,000
Property insurance $22,000
Utilities $32,000
Malpractice insurance $180,000
Dep. Exp. $60,000 ÷ 4 year $15,000
Office supplies $4 × 50 client × 360 day $4
Total $1,491,980 $4 × 50 × 360 = $72,000

Supporting calculations
1. Expected number of clients per day based on probability distribution
= [(20 × 10%) + (30 × 30%) + (55 ×40%) + (85 × 20%)] = 50 client
Note: Expected number of clients per day based on most likely event would be 55 clients (less
accurate than expected value)
2. Expected revenues = [$30 × 50 client × 360 day] + [50 client × 360 day × 20% × $2,000 ×
30%] = $2,700,000
3. Average revenue per client = $2,700,000 ÷ [50 client × 360 day] = $150
4. Wages, Salaries and fringe benefits
- Lawyer $25 × 16 hours × 360 day = $144,000
- Paralegal $20 × 16 hours × 360 day = $115,200
- Secretary [$15 × 16 hours × 360 day] + [200 × $15 × 1.5 time] = $ 90,900
- Clerk [$10 × 16 hours × 360 day] + [200 × $10 × 1.5 time] = $ 60,600
Total salaries and overtime $410,700

5. Fringe benefit = $410,700 × 40% = $164,280

$1491980
Break-even point = = 10,219 clients each client has an average
Average..revenue..$ 150  V .C.$4
revenue of $150
Contribution margin pro-forma income statement
Revenues Subtotal Total
Initial consultations 540,000
Favorable Lawsuits 2,160,000
Total projected revenues 2,700,000
Total variable operating expenses (72,000)
Total contribution margin 2,628,000
Total fixed operating expenses 1,491,980
Projected before tax operating income 1,136,020

Essay: A Racquet Club (RC) is a sports facility that offers tennis, racquet ball and other physical fitness
facilities to its members. RC owns and operates a large club with 2,000 members in a metropolitan area. The
club has experienced cash flow problems over the last five years, especially during the summer months when
both court use and new membership sales are low. Temporary bank loans have been obtained to cover the
summer shortages. The owners have decided to take action to improve RC’s net cash flow position. They
have asked the club’s financial manager to prepare a projected cash budget based on a proposed revised fee
structure. The proposal would increase membership fees and replace the hourly tennis and racquet ball
court fees with a quarterly charge that would allow unlimited usage of the courts. The new rates would
remain competitive when compared to the rates of other clubs in the area. Although there will be some
members who do not renew because of the increase in price; management believes that the offer of unlimited
court time will increase membership by 10%. The original and proposed fee structure is shown below, along
with the current membership distribution. The membership distribution is assumed to remain unchanged. All
members would be required to pay the quarterly court charges.

Original and proposed fee structure

Membership Old-annual New-annual Quarterly court Membership


Category membership fees membership fees charges distribution

Individual 250 $300 $50 60%

Student 150 180 40 10%

Family 500 600 90 30%

Projected membership payment activity

Quarter New Renewed members (renewal Court usage time in hours


members for the conning year) Prime Regular
1 100 700 5,000 7,000
2 70 330 2,000 4,000
3 50 150 1,000 2,000
4 200 600 5,000 7,000

Fixed costs are $157,500 per quarter, including a quarterly depreciation charge of $24,500. Variable
costs are estimated at $15 per hour of total court usage time.

REQUIRED:

Assume that the change to the new pricing structure will be implemented starting from third quarter.
Assume also that any renewals and new members will be due at the beginning of the quarter and
membership structure is as follows

Description Q1 Q2 Q3 Q4

Membership at the beginning of the quarter 2,080 2,020 2,000 2,020

Renewals 700 330 150 600

New 100 70 50 200

Withdraws-at the Beg of the quarter 160 90 30 150

Membership at the end of the quarter 2,020 2,000 2,020 2,070

A. Prepare RC’s cash budget for the third quarter. Assume the opening cash balance is $186,000, Include
supporting calculations where appropriate.
B. Prepare pro-forma income statement for the third quarter
C. How would sensitivity analysis help RC management in the decision-making process?
D. Identify at least four factors that RC should consider before implementing this decision.
Answer
RC Cash Budget Proposed Third Quarter (only)
Beginning cash balance $186,000
Third quarter cash receipts 198,820
Third quarter cash expenditures 178,000
Ending cash balance 206,820

Supporting calculations

1. Cash inflow from memberships

Cash collected during the third quarter for membership consists of:
A. Cash collected from new members (50 member) and
B. Members who renew their annual membership for the coming year (150 members)

Members % Allocation of Annual Cash collections from annual


category members Membership Fees memberships
Individual 60% 200 × 60%=120 $300 $36,000
Student 10% 200 × 10%=20 180 $3,600
Family 30% 200 × 30%= 60 600 $36,000
Total 100% 200× 100%= 200 75,600

2. Cash inflow from quarterly court charges

All members would be required to pay the quarterly court charges (2,000 current members +
50 new members – 30 withdrawals = 2,020 members

Members % Allocation of members Quarterly Court Cash collections from


category Charges Quarterly Court Charges
Individual 60% 2,020 × 60%=1,212 $50 $60,600
Student 10% 2,020 × 10% = 202 40 $8,080
Family 30% 2,020 × 30%= 606 90 $54,540
Total 100% 2,020 × 100% = 2,020 123,220

Total estimated cash collections during the third quarter


1. Cash collections from annual memberships = 75,600
2. Cash collections from Quarterly Court Charges =123,220
Total $198,820
Cash expenditures
Fixed costs 157,500
Less depreciation (24,500)
Variable costs (1,000 hours +2,000 hours) × $15 = 45,000
Total cash costs 178,000

B. Pro-forma income statement for the third quarter


1. Active members at the beginning of the quarter members who paid at the original membership
Fees (2,000 members – renewals 150- member – withdrawals 30) = 1,820 members

Members % Allocation of Annual Quarterly accrual basis


category members Membership Fees Membership revenues
Individual 60% 1,820 × 60% = 1,092 × $250 = $68,250
1,092 $273,000
Student 10% 1,820 × 10% = 182×$150 = $6,875
182 $27,300
Family 30% 1,820 × 30%= 546 ×$500 = $68,250
546 $273,000
Total 100% 1,820 × 100%= 573,300 143,325
1,820

2. The renewals (150 member) and new members (50) will pay at the proposed new annual
Membership Fees

Members % Allocation of Annual Quarterly accrual basis


category members Membership Fees Membership revenues
Individual 60% 200 × 60% = 120 × $300= $36,000 $9,000
120
Student 10% 200 × 10% = 20 20 ×$180 = $3,600 $900
Family 30% 200 × 30% = 60 60 ×$600 = $36,000 $9,000
Total 100% 200 × 100% = 75,600 18,900
200

3. Quarterly Court Charges based on proposed structure would constitute collections from
2,000 members at the beginning -30 member's withdrawal + 50 new members =2.020 member

Members category % Allocation of Quarterly court Quarterly court


members charges charges
Individual 60% 2.020 × 60% =1,212 $50 $60,600
Student 10% 2.020 × 10% = 202 40 $8,080
Family 30% 2.020 × 30% = 606 90 $54,540
Total 100% 2,020 × 100% = 123,220
2,020

4. Pro-forma income statement for the third quarter

Description Amount
Quarterly accrual basis membership revenues(143,325 +18,900) 162,225
Quarterly Court Charges 123,220
Total revenue 285,445
Less fixed cost (157,500)
Variable cost (45,000)
Operating income 82,945

C. Sensitivity analysis would help RC management by testing the assumed projections and seeing how
sensitive the cash flows are to changes in the number of members or the distribution of members.
D. Other factors that RC should consider include:
• Communication strategy to current members (how this price change is perceived by current members).
• Market acceptance of the new pricing strategy.
• Cost associated with the change.
• Timing of the change.
• The effect on the mix of membership class.
• The anticipated rate of return for excess cash and the costs of borrowing funds.
• The reliability of the projections.
• The capacity of the tennis and racquet ball courts.
• Price elasticity for memberships in similar clubs.
• The reaction of the competition.
• Quality of its facilities and staff.
• Cost of advertising/communicating this price change.

Sensitivity analysis (what-if analyses)

Since the company is dealing with the future when projecting its pro forma F/S, there is always a chance
that these initial assumptions will change. What if the projected sales growth is 20% rather than 18%?
What if the cost of production has increased and the COGS is actually 85% of sales rather than 80% of
sales?

The Company can analyze these alternatives by performing a series of what-if analyses. This involves
systematically changing one of the assumptions and analyzing the impact that these changes have on
the pro forma statements and on the selected financial ratios.

What-if analysis; changes one assumption (or input variable) at a time; it is also known as sensitivity
analysis.

Example: Susan Hines has developed an estimate of the earnings per share for her firm for the next year
using the following parameters.
Sales $20 million
Cost of goods sold 70% of sales
General & administrative expenses $300,000
Selling expense $100,000 plus 10% of sales
Debt outstanding $5 million @ 8% interest rate
Effective tax rate 35%
Common shares outstanding 2 million
She is now interested in the sensitivity of EPS to sales forecast changes. A 10% sales increase would
increase EPS by

A. 7 cents per share B.10.40 cents per share C.13 cents per share D. 20 cents per share

Answer (C) is correct

Description Original Forecast changes.


Sales revenue 20,000,000 22,000,000
Cost of goods sold 14,000,000 15,400,000
Gross profit 6,000,000 6,600,000
Selling expense 2,100,000 2,300,000
General & administrative expenses 300,0000 300,000
EBIT 3,600,000 4,000,000
Interest expense 400,000 400,000
EBT 3,200,000 3,600,000
Income tax expense 35% 1,120,000 1,260,000
Net income 2,080,000 2,340,000
# of C/S outstanding 2,000,000 2,000,000
EPS $1.04 $1.17
Increase in EPS $1.17- $1.04 = $0.13
% change in EPS =12.50 %

Overall, sensitivity analysis encourages a company to manage by exception. Performing sensitivity analysis
by changing different input variables in the planning model (called parameters which are independent
variables) allows the company to determine which sets of input variables have the greatest impact on items
of interest, called outcome measure (which is dependent variables for example EPS) and which sets of
input variables have negligible impacts on the items of interest. This allows the company to focus its
attention and effort on the most critical assumptions.
♠Sensitivity analysis is an appropriate technique to analyze the alternatives by using expected inputs and
altering them before a decision is made, a trial- and-error method may be adopted, usually in a computer
model, to calculate the sensitivity of the solution (variability of outcomes) to changes in a variable
♠Sensitivity analysis tries to identify what source of uncertainty weighs more on the study's conclusions
♠Sensitivity analysis is a way to predict the outcome of a decision if a situation turns out to be different
compared to the key prediction(s).
♠Sensitivity analysis can be useful to computer modelers for the following purposes
1. Support decision making or the development of recommendations for decision makers (e.g. testing the
robustness of a result)
2. Enhancing communication from modelers to decision makers (e.g. by making recommendations more
credible, understandable, compelling or persuasive)
3. Increased understanding or quantification of the system (e.g. understanding relationships between input
and output variables)
4. Model development (e.g. searching for errors in the model).

Benefits and limitations of sensitivity analysis

Sensitivity analysis shows managers how susceptible the outcomes of decisions are to changes in any
parameter or estimate. Managers may be faced with a situation in which any slight variance in an
estimate will result in a large change in the outcome or, a large variance in an estimate may not result in a
noticeable change in the outcome. In addition, the change in one estimate or parameter could have an
impact on another. If managers are not aware of this mutual relationship and make only one change, the
results of their sensitivity analysis will not be reliable.

Q: When performing a sensitivity analysis, if the selling price per unit is increased, then the
A. Per unit fixed administrative costs will increase.
B. Per unit direct materials purchase price will increase.
C. Total volume of sales will increase.
D. Total costs for sales commissions and other non-manufacturing variable costs will increase.
Answer (D) is correct

Sensitivity analysis can be used to estimate the impact on the outcome of decisions of occurrences that
may or may not happen. For example: What if workers strike and hold up production? What will be the
impact on the bottom line? Another example, in a present value analysis, a manager might first calculate the
net present value (NPV) or internal rate of return (IRR) assuming that a new asset has a 10-year life. The
NPV or IRR can then be recalculated using a 5-year life to determine how sensitive the result is to the
change in the assumption.

Sensitivity analysis helps analysts determine how changes in the probabilities for states of nature or changes
in potential payoffs, which may be based on subjective assessments, affect recommended decision
alternatives.

Sensitivity analysis helps to evaluate the risk associated with decisions. When the result will change
drastically if the originally predicted data are not achieved the result is called to be sensitive to this parameter

Q1: __________ is the process of varying key estimates to identify those estimates that are the most critical to
a decision.
A. The graph method B. A sensitivity analysis
C. The degree of operating leverage D. Sales mix
Answer (B) is correct

Q2: Which of the following best describes sensitivity analysis?


A. Sensitivity analysis determines the value of the mean square error.
B. Sensitivity analysis measures consumers' sensitivity to changes in product weights.
C. Sensitivity analysis helps determine how changes in probabilities will change expected outcomes.
D. Sensitivity analysis is a marketing tool that tests consumers' sensitivity to market conditions.
The correct answer is (C). Sensitivity analysis helps analysts determine how changes in the probabilities
for states of nature or changes in potential payoffs, which may be based on subjective assessments, affect
recommended decision alternatives.

Q3: A widely used approach that managers use to recognize uncertainty about individual items and to
obtain an immediate financial estimate of the consequences of possible prediction errors is
A. Expected value analysis. B. Learning curve analysis.
C. Sensitivity analysis. D. Regression analysis.
Answer (A) is incorrect because expected value is the probabilistically weighted average of the outcomes
of an action.
Answer (B) is incorrect because learning curve analysis quantifies how labor costs decline as employees
learn their jobs through repetition.
Answer (C) is correct. Sensitivity analysis determines how a result varies with changes in a given
variable or parameter in a mathematical decision model. For example, in a present value analysis, a
manager might first calculate the net present value or internal rate of return assuming that a new asset has
a 10-year life. The NPV or IRR can then be recalculated using a 5-year life to determine how sensitive the
result is to the change in the assumption.
Answer (D) is incorrect because regression, or least squares, analysis determines the average change in the
dependent variable given a unit change in one or more independent variables.

Q4: Through the use of decision models, managers thoroughly analyze many alternatives and decide on the
best alternative for the company. Often, the actual results achieved from a particular decision are not what
were expected when the decision was made. In addition, an alternative that was not selected would have
actually been the best decision for the company. The appropriate technique to analyze the alternatives by
using expected inputs and altering them before a decision is made is
A. Expected value analysis. B. Linear programming.
C. Program Evaluation Review Technique (PERT). D. Sensitivity analysis.
Answer (A) is incorrect because expected value analysis is used to determine an anticipated return or cost
based upon probabilities of events and their related outcomes.
Answer (B) is incorrect because linear programming optimizes a function given certain constraints.
Answer (C) is incorrect because PERT is a network technique used to plan and control large projects.
Answer (D) is correct. Sensitivity modeling can be used to determine the outcome of a variety of decisions.
A trial- and-error method may be adopted, usually in a computer model, to calculate the sensitivity of the
solution (variability of outcomes) to changes in a variable.

Q5: The process of evaluating the effect of changes in variables such as sales price or wage rates on the
optimum solution in a linear programming application is called
A. Iterative analysis. B. Regression analysis.
C. Sensitivity analysis. D. Matrix analysis.
Answer (A) is incorrect because both the simplex method and iterative analysis are ways of working a linear
programming problem by hand.
Answer (B) is incorrect because regression analysis is a means of finding the relationship between two or
more variables.
Answer (C) is correct. Sensitivity analysis is a process to determine how sensitive the final result (solution)
is to changes in variables. It is often used in capital budgeting decisions to incorporate various levels of risk.
Answer (D) is incorrect because matrix analysis is another mathematical tool used to solve a variety of
problem types.

Q6: __________ utilizes a “what-if” technique that examines how results will change if the originally
predicted data changes.
A. A sales forecast B. A sensitivity analysis
C. A pro forma financial statements D. The statement of cash flows
Answer (B) is correct

Q7: In an economic order quantity (EOQ) model, both the costs per order and the holding costs are
estimates. If those estimates are varied to determine how much the changes affect the optimal EOQ, such
analysis would be called a:
A. Forecasting model. B. Sensitivity analysis.
C. PERT/CPM analysis. D. Decision analysis.
Answer (A) is incorrect because forecasting models involve projecting data over time or developing
regression models when time series data are not available.
Answer (B) is correct: An EOQ sensitivity analysis involves varying the holding costs per unit and/or the
order costs to determine how much the change affect the optimal EOQ.
Answer (C) is incorrect because a PERT/CPM analysis involves project scheduling.
Answer (D) is incorrect because decision analysis involves selecting the best option from alternatives.

Q8: A cost-volume-profit model developed in a dynamic environment determined that the estimated
parameters used might vary between limits. Subsequent testing of the model with respect to all possible
values of the estimated parameters is termed
A. Statistical estimation. B. Statistical hypothesis testing.
C. A sensitivity analysis. D. A time-series study.
Answer (A) is incorrect because statistical estimation involves estimated parameters.
Answer (B) is incorrect because statistical hypothesis testing refers to the testing of different propositions.
Answer (C) is correct: Sensitivity analysis is a method of studying the effects (sensitivity) of changes in
one or more variables on the results.
Answer (D) is incorrect because Time-series study is used in forecasting data over a specified period.
Q9: Sensitivity analysis in linear programming is used to
A. Determine the degree that the constraints vary.
B. Develop the technological matrix.
C. Test the accuracy of the parameters.
D. Determine how the optimal solution would react to changes in parameters.
Answer (A) is incorrect because sensitivity analysis determines the degree of change in the expected result
if the constraints vary.
Answer (B) is incorrect because the technological matrix is not developed using sensitivity analysis. The
technological matrix is an input of a model and is designed by engineers.
Answer (C) is incorrect because sensitivity analysis does not test the accuracy of the parameters. It rather
determines the degree of change in the expected result if the parameters vary.
Answer (D) is correct: Sensitivity analysis encompasses a variety of methods used to determine how an
expected result will change if factors that were involved in predicting an amount change.

Fact pattern: divisions A and B are autonomous divisions of a corporation. They have no beginning or
ending inventories, and the number of units produced is equal to the number of units sold. Following is
financial information relating to the two divisions.

Description Divisions A Divisions A


Sales $150,000 $400,000
Other revenue 10,000 15,000
Direct materials 30,000 65,000
Direct labor 20,000 40,000
Variable factory overhead 5,000 15,000
Fixed factory overhead 25,000 55,000
Variable selling and administrative expense 15,000 30,000
Fixed selling and administrative expense 35,000 60,000
Central corporate expenses (allocated) 12,000 20,000

Q1: What is the contribution margin of Division B?


A. $235,000 B. $265,000 C. $150,000 D. $205,000

Answer (B) is correct:

Description Divisions A Divisions A


Total revenue = Sales + Other revenue $160,000 $415,000
Direct materials 30,000 65,000
Direct labor 20,000 40,000
Variable factory overhead 5,000 15,000
Variable selling and administrative expense 15,000 30,000
T.C.M $90,000 $265,000

Q2: What is the total contribution to corporate profits generated by Division A before allocation of
central corporate expenses?
A. $20,000 B. $18,000 C. $30,000 D. $80,000

Answer (C) is correct: Department A's contribution to the corporate profits is calculated as all revenues
minus all expenses except the allocated central corporate expenses
= T.C.M $90,000 – (fixed O.H $25,000 + Fixed non-manufacturing $35,000) = $30,000

True or false

1. If budgeted amounts change; the kaizen approach can be used to examine changes in the budgeted
results
Answer is false: If budgeted amounts change, sensitivity (what-if) analysis can be used to examine
changes in the budgeted results

2. Sensitivity (what-if) analysis incorporates continuous improvement into budgeted amounts.


Answer is false: Kaizen budgeting incorporates continuous improvement into budgeted amounts.

3. Sensitivity analysis is a “what-if” technique that managers use to examine how a result will change
if the originally predicted data are not achieved or if an underlying assumption changes.
Answer is true

4. Sensitivity analysis helps to evaluate the risk associated with decisions.


Answer is true: When the result will change drastically if the originally predicted data are not achieved
the result is called to be sensitive to this parameter

See Gleim's MCQ's # 114,

9- Pro forma (or budgeted) Income Statement

Various pieces of the operating budget developed earlier are used to put together the
pro forma (or budgeted) income statement, which shows what the profits for the
company will be at the end of the year if the company meets its budget and if its
assumptions prove to be correct. When budgeted income falls short of the goal,
management knows it must take corrective action. The budget is revised to account for
these actions. A budgeted income statement is therefore a benchmark to use in
evaluating progress.

Once a company completes the various pieces of the operating budget and creates
the pro forma (or budgeted) income statement, it will next develop the necessary
financial budgets to identify the assets and capital finance (both debt and equity)
needed to support the operation. These financial budgets include the capital
expenditure budget, the cash budget, the pro forma (or budgeted) balance sheet,
and the pro forma (or budgeted) statement of cash flows.

12- Pro forma (Budgeted) Balance Sheet

A pro forma balance sheet (also known as budget balance sheet or statement of
financial position) illustrates how operations should affect the company’s assets,
liabilities, and stockholders’ equity. The budgeted balance sheet is usually the last item
prepared in a master budget and is based in part on the budgeted balance sheet at the
end of the current period. The effects of operations for the budget period are added to
the data in the prior balance sheet.

13- Pro forma (Budgeted) Statement of Cash flows

A company’s pro forma statement of cash flows represents its projected sources and
uses of funds. Using information from the income statement and balance sheet, it
groups the company’s cash flows into one of three activities: operating, investing, and
financing activities.

Reasons for Budgeting

There are four main reasons a company creates a budget: planning, communication,
monitoring, and evaluation.

1- Planning

One of the major benefits of budgeting is that it forces the organization to examine the
future. Expectations must be established for income, expenses, personnel needs, future
growth (or contraction), etc. Planning allows for the input of ideas from multiple
sources within the organization, and allows for input from different viewpoints. The
planning process may generate new ideas for the organization’s direction, or it may
provide insight into better ways to achieve goals that have already been established.
The budget process provides a framework to achieve the goals of the organization.
Without the framework of a budget, individual managers would be improvising
decisions without the direction and coordination provided by a budget. Without a
budget, the organization would be operating in a reactive manner, rather than in a
proactive manner.

2- Communication and Coordination

Budgeting also promotes communication and coordination of efforts within the


organization. The different parts of the organization (production, marketing, materials
management, etc.) must communicate their plans and needs to each other during the
budget process so that all can evaluate the effect that the plans and needs of others have
on their own plans and needs. Each part of the organization must coordinate its
activities to attain the budgeted goals and objectives. For example, if new products are
to be developed, funds must be provided for development, materials will have to be
purchased to produce the products, marketing and sales must have sufficient resources
to promote and sell the products, and shipping and distribution may need additional
space to store the products or additional resources to distribute them. Budgeting also
allows the organization to communicate its goals to everyone in the organization,
including those not involved in the budget process. Budgeting sets the stage for
everyone in the organization to work toward the goals of the organization (goal
congruence).

3- Monitoring
The budget sets standards, or performance indicators, by which managers can monitor
the organization’s progress in meeting its goals. By comparing the actual results for a
period to the budgeted results for that period, managers can see whether the
organization is on track to achieving its goals. Breaking down the organization’s master
budget to divisional and departmental levels allows each level of the organization to be
evaluated. The organization as a whole may be meeting its goals, while individual
divisions and departments are failing. The difference between the actual results and the
expected results is called a variance. A negative, or unfavorable, variance may indicate
a need to take corrective action. Positive, or favorable, variances can reflect
opportunities to make adjustments to take advantage of the conditions creating the
variance, that is, if sales are up, then perhaps production should be increased.

4- Evaluation

Budgets also serve as guides or instruments for employee evaluations. Once the budget
is set and managers have been advised of their responsibilities in relation to budget
performance, they can be held responsible for their portion of the budget. By comparing
actual results to the budget for a given period, a manager’s performance can be
evaluated. Having negative or unfavorable results does not necessarily mean that a
manager is not performing well, but it does provide an indication that a specific part of
the business should be focused on, in order to determine the root cause of the
unfavorable variance. Likewise, positive, or favorable, results do not necessarily mean
that a manager is performing in an exceptional way. Performance evaluations allow an
organization to motivate employees by rewarding them for good performance in a
number of ways, such as through performance-based bonuses, and/or by including
performance evaluations in the decision process for future compensation or promotion
decisions.

• However, early budgeting steps can be used to refine the strategic direction of the
company because they use more current information.

• Forecasts often have lower accountability than a budget, are not usually approved by
management, and are not often formally analyzed against variances. For instance, a
manager may create a forecast for direct materials needed in next week’s production to
be sure that there is adequate inventory. However, budgets must use the forward-
looking information from more comprehensive forecasts. Therefore, the forecasts
directly used in the budgeting process, such as the sales forecast, must be kept
accountable.
• The budget should be used to alleviate potential bottlenecks and to allocate resources
to those areas that will use the funds most efficiently and effectively.
• The budget must contain technically correct and reasonably accurate numbers and
facts.
• Management (including top management) must fully endorse the budget - they must
accept responsibility for reaching the budget goals.
• The budget must be perceived by employees as a planning, communication, and
coordinating tool, and not as a pressure or blame device.
• The budget must be characterized as a motivating tool to help employees work toward
organizational goals.
• The budget must be seen as an internal control device, and internal-use budgets
should base evaluations on controllable or discretionary costs.
•A higher authority than the team who developed the budget must review and approve
the budget.
•The final budget should not be easily changed, but it must be flexible enough to be
useful. Budgets should compel planning, promote communication and coordination,
and provide performance criteria. The budget process must balance input from those
who will need to follow the budget against a thorough and fair review of the budget by
upper management.

Whether the organization and its budget are very simple or highly complex, the
characteristics of a successful budget process include: the budget period, the
participants in the budget process, the basic steps in budgeting, and the use of cost
standards.

You might also like